You are on page 1of 113

1) WEALTH TRANSFER UPON DEATH: THE FUNDAMENTALS a) Introduction i) The Right to Inherit and the Right to Convey (1)

While not a natural right, the strict beginnings of inheritance made heirs disobedient and headstrong, defrauded creditors of their just debts, and prevented many provident fathers from dividing or charging their estates as the exigencies of their families required. (a) This introduced pretty generally the right of disposing of ones property, or a part of it, by testament; that is, by written or oral instructions properly witnessed and authenticated, according to the pleasure of the deceased, which we emphatically style his will. (i) Wills, testaments, rights of inheritance and successions are all of them creatures of the civil or municipal laws, and accordingly are in all respects regulated by them. (2) John Locke, disagreed. (a) The inheritance is a natural right born into man as a strong desire of propagating their kind, and continuing themselves in their Posterity, and this gives children a title, to share in property of their parents, and a right to inherit their possessions. b) The Basic Policies i) The Problem of the Dead Hand: The cold and numbing influence of the Dead Hand (1) Restatement (Third) of Property: Wills and Other Donative Transfers (2003) (a) Sec. 10.1 Donors Intention Determines the Meaning of a Donative Document and Is Given Effect to the Maximum Extent Allowed by Law (i) Among the rules of law that prohibit or restrict freedom of disposition in certain instances are those relating to spousal rights, creditors rights, unreasonable restraints on alienation or marriage, provisions promoting separation or divorce, impermissible racial or other categorical restrictions, provisions encouraging illegal activity, and the rules against perpetuities and accumulations. ii) Shapira v. Union National Bank, 315 N.E.2d 825 (1974) (1) Material Facts: Father left his inheritance to his son on the condition that he marry a jewish girl whose both parents are jewish. (2) Issue: Can a will specify that the heirs to the inheritance marry a person of a specific religion? (3) Holding: Yes. (4) Reasoning: In Ohio a testator may fully disinherit his children This would seem to demonstrate that, from a constitutional standpoint, a testator may restrict a childs inheritance. The will articulates a partial restraint upon marriage. If it were a full restraint, or an unreasonable restraint, then it would be unconstitutional. The great weight of authority contends that gifts conditioned on the restriction of marriage to a particular religious class or faith is reasonable. (5) Notes: What if he had tried and failed to marry a Jewish girl by the time the Will required? If his father was alive he may come to reason that the son should be allowed 1

more time. However, since he is dead, incapable of reason or capable of suffering the consequences. We should thus apply the cy pres approach to private trust cases, unless the testator expressly rejects a power of judicial modification. (a) Senator Joseph Lieberman allowed his cousins who did not marry jewish women to restore their shares after their wives converted to Judaism. (b) Restatement (Second) of Propert: Donative Transfers Sec. 6.2, cmt. A (1983). (i) This provision provides that a restraint unreasonably limits the transferees opportunity to marry if a marriage permitted by the restraint is not likely to occur. The likelihood of marriage is a factual one, to be determined from the circumstances of the particular case. (c) A will or trust provision is ordinarily invalid if it is intended or tends to encourage disruption of a family relationship. i.e. not allowing a surviving spouse to remarry unless the purpose is to provide support. (d) Restatement (Third) of Trusts Sec. 29(c), (2003) invalidates trusts that are contrary to public policy. (i) Conditions that encourage the beneficiaries to pursue an education, conditions that provide what might be termed moral incentives, and conditions designed to encourage the beneficiaries to have a productive career are generally enforceable. c) Probate and Nonprobate Property i) Probate property is property that passes through probate under the decedents will or by intestacy ii) Nonprobate property is property that passes outside of probate under an instrument other than a will (1) For example: Joint tenancy property, both real and personal, life insurance, contracts with payable-on-death provisions, interests in a trust, etc. (2) Distribution of nonprobate property does not involve a court proceeding, but is made in accordance with the terms of the controlling contract or trust or deed. iii) Probate Terminology and History (1) When a person dies and probate is necessary, a personal representative will oversee the winding up of the decedents affairs. (a) This person is a fiduciary who inventories and collects the property of the decedent; manages and protects the property during the administration of the decedents estate; processes the claims of creditors and tax collectors; and distributes the property to those entitled. (b) The court that supervises the administration of the probate estate is usually referred to as a probate court. (c) If the person dies testate and in the will names the fiduciary person, that person is called the executor. If there is no executor then the court will name someone who is called an administrator.

(i) The administrator is usually selected from a statutory list of person who are to be given preference typically in the following order: surviving spouse, children, parents, siblings, creditors. 1. Going through probate means that an estate is administered through one of the probate courts. 2) PROVIDING FOR THE FAMILY a) Rights of a Spouse i) Marital property systems (1) Separate property (a) Husband and wife own separately all property each acquires (except those items one spouse has agreed to put into joint ownership with the other). (2) Community property (a) After marriage, husband and wife own all acquisitions from earnings after marriage in equal, undivided shares unless otherwise agreed to. (i) There are many different variations of the two above systems, but represent the difference of thinking about spousal rights. (ii) Generally, if one spouse dies they may dispose of their half of property through a will as they choose. 1. Note: Community property disposition of property varies widely based on which community property state you are in. ii) Social Security (1) Social Security system pays retirement benefits to a worker and the workers surviving spouse. (a) Note: The taxes you pay now are going toward paying current individuals benefits. (b) Workers have no power to transfer their right to benefits to any other person. (c) A Divorced former spouse of the worker has a right to benefits if the marriage lasted for ten years or longer. (i) Note: If both spouses work, they can only take social security for the larger amount but not both incentive to have a stay at home spouse? (2) Employee Pension Plans (a) Most private pension plans are governed by the federal Employee Retirement Income Security Act of 1974 (ERISA), which preempts inconsistent state laws. As amended in 1984, ERISA requires that the spouse of an employee haave survivorship rights if the employee predeceases the spouse. (i) The purpose to provide a treat of income to surviving spouses. (ii) If the employee dies before retirement, the surviving spouse may be entitled to a preretirement survivor annuity. 1. Waiver or a spouses right is discouraged hence premarital agreements cannot waive ERISA-covered pension rights 2. Problem: A man remarried after his wife died and collected employee pension. He died and the court held that the funds should go to his new wife instead of his children, regardless of his first wifes intention that it be 3

left to the children. The reason was to support ERISAs object to protect surviving spouses. This preempted state community property rights. (3) Homestead (a) Most states have a homestead law that is designed to secure the family home to the surviving spouse of minor children, free of the claims of the decedents creditors. (i) Some states require a homestead must be established by the decedent during life, usually by filling out a declaration of homestead in some public office. (ii) Other states allow a probate court to set aside a homestead. (iii) This right is often given in addition to any other rights the surviving spouse has in the decedents estate. (4) Personal Property Set-Aside (a) Surviving spouse (and sometimes of minor children) are allowed to receive tangible personal property of the decedent to a certain value. (UPC Sec. 2-402 (1990 rev. 2008) sets the limit at $15,000.00, subject to the cost of living adjustment in Sec. 1109. (i) These, like homestead, are exempt from creditors claims, and usually include household furniture and clothing, but may also include a car and farm animals. 1. While there are several conditions, if met, the decedent has no power to deprive the surviving spouse of the exempt items (5) Family Allowance (a) Every state has a statute authorizing the probate court to allow an allowance for maintenance and support of the surviving spouse (and often of dependent children). (i) The allowance may be limited to a fixed period (typically one year), or while the will is contested or for the entire period of administration. 1. As with homestead and personal property set-aside, any family allowance is in addition to whatever other interest pass to the surviving spouse. a. UPC Sec. 2-404 (1990): a reasonable allowance cannot continue past a year if the estate is inadequate to pay creditors b. UPC Sec. 2-405 (1990): the personal representative to determine the family allowance up to a stated limit without court order by subject to judicial review. (6) Dower and Curtesy (a) Dower entitles the widow to a life estate in one-third of her husbands qualifying land (i) The right attaches the moment the husband acquires title to land or upon marriage, whichever is later. 1. This means that selling the property to another was subject to the wifes dower which cannot be cut off without her consent. 2. This has been abolished in the great majority of states since property interest is no longer as valuable as it once was. It functions really only as a practical matter to require both spouses signature in the sale of land.

(ii) A curtesy is the same as a dower for man, except that he had a full ownership in the property. Again this has been abolished in most states and survives only as a label to name the support interest to the husband which is the same as the wife. 1. Note: The husband had to have had a child with the now-deceased wife. He had the right even if no child survived. iii) The spousal elective share (1) All but one of the separate property states gives the surviving spouse, by statute, an elective share. (a) This is usually 1/3 of the decedents property that would be considered probate (life insurance is nonprobate and would thus have to be directly given) (2) The spouse can take under the decedents will or the spouse can renounce the will and take a fractional share of the decedents estate. There has to be a will in order for there to be an elective share option for the surviving spouse (3) The rationale for the elective share (a) The primary policy justification for the elective share is that the surviving spouse contributed to the decedents acquisition of wealth and deserves to have a portion of it. (i) This implements the partnership theory of marriage. (b) Another policy reason is that to provide the surviving spouse with adequate support. (i) A partnership theory militates toward awarding the surviving spouse one-half of the decedents property acquired during the marriage, whereas the support theory often justifies a smaller share but would apply it to all the decedents property. 1. Note, in most states and under UPC Sec.2-212(a) if the wife dies before exercising her right to elective share the proceeds go the heirs of her husband (assuming they are not the same). (c) Caution: There is no subject with more statutory variation than the elective share depending on whether a state views the proceeds under partnership theory, support theory or a mix of the two depending on the circumstances. (i) UPC Aticle II, Part 2 1. Elective Share of Surviving Spouse a. Partnership Theory i. Under this approach, the economic rights of each spouse are seen as deriving from an unspoken marital bargain under which the partners agree that each is to enjoy a half interest in the fruits of the marriage, i.e., in the property nominally acquired by and titled in the sole name of either partner during the marriage. ii. Simply stated, the system of equitable distribution views marriage as essentially a shared enterprise or joint undertaking in the nature of a partnership to which both spouses contribute directly and

indirectly, financially and nonfinancially the fruits of which are distributable at divorce iii. At death, elective-share law in the common-law states has not caught up to the partnership theory a surviving spouse may claim a one-third share of the decedents estate not the 50 percent share of the couples combined assets that the partnership theory would imply. (d) Note: The surviving spouse may waive the elective or forced share under UPC Sec. 2-213 This would usually occur if the surviving spouse wanted the wills instructions to be performed as the decedent wished, even if the surviving spouse would have had a large portion of the estate. (i) Another reason some surviving spouses decline the elective share is to prevent a new spouse and his or her heirs from getting a portion of the decedents estate this is not an issue with community property states because (ii) Note: Further, the implications of community states and separate property states laws require careful planning for the couple on the move. (4) Property subject to the elective share (a) The original elective share allowed 1/3 of the decedents estate. (i) This was understood to be the probate estate with increasing importance of nonprobate modes of transfer 1. Note: You could thus leave all of your money in a trust to your heirs while giving your entire probate estate to your spouse, amounting little to nothing, effectively disinheriting them. (b) Statutory Schemes (i) Dissatisfied with vague tests laid down by courts, many states have enacted statutes providing objective criteria for determining what nonprobate transfers are subject to the elective share. 1. They favor instead a list of specified nonprobate transfers that are added to the probate estate to constitute a net estate or an elective estate or an augmented estate against which the surviving spouses elective share is applied. a. New York i. The New York statute now gives the surviving spouse $50,000.00 or one-third of the decedents net estate, whichever is greater, plus a personal property set-aside. - In New York, the decedents net estate that is subject to the elective share includes the probate estate and the following nonprobate transfers: o (1) gifts causa mortis (gifts of tangible personal property contingent on death;

(2) gifts made within one year before death, except gifts not exceeding the amount of the gift tax annual exclusion ($13,000.00 in 2009); o (3) savings account (Totten) trusts; o (4) joint bank accounts, to the extent of the decedents contribution; o (5) joint tenancies and tenancies by the entireties, to the extent of the decedents contribution; o (6) property payable on death to a person other than the decedent; o (7) lifetime transfers in which the decedent retained possession or life income or a power to revoke or a power to consume, invade, or dispose of the principal; o (8) pension plans or the like; and o (9) any property over which the decedent had a general power of appointment enabling him to appoint the property to whomever he pleases ii. The amount of the elective share is reduced by deducting the value of any interest, other than a life estate, that passes from the decedent to the surviving spouse by intestacy, by will, or by will substitute. b. Delaware i. It defines the property subject to the elective share as all property includible in the decedents gross estate under the federal estate tax, whether or not an estate tax return is filed. - If a nonprobate transfer is taxable at death (as are revocable trusts, POD contracts, and joint tenancies), the surviving spouse can reach it. ii. This approach has the advantage of incorporating into elective share law the inclusion of principles of federal estate tax law, which evolved out of long experience with decedents trying to avoid estate tax by lifetime transfers. - The estate tax law concentrates only on whether an economic benefit is transferred from the decedent to another person. (c) UPC Sec. 2-202 (1969) Inspired by the New York statute the UPC introduced the augmented estate (the probate estate augmented with certain nonprobate transfers) (i) The surviving spouse is entitled to one-third of the augmented estate. The augmented estate includes the probate estate and the following nonprobate and inter vivos transfers made without consideration at any time during the marriage:

1. Any transfer under which the decedent retains the right to possession or income from property; 2. Any transfer which the decedent can revoke or invade or dispose of the principal for his own benefit; 3. Any transfer in joint tenancy with someone other than the spouse; 4. Any transfer made within two years before death exceeding $3,000.00 per done per year ($3,000.00 was, at the time, the maximum amount exempt from the federal gift tax under annual exclusion; it is now $13,000.00); and 5. Property given to the surviving spouse during life, including a life estate in a trust, and property received by the spouse at death derived from the decedent, such as life insurance and pensions. (ii) The purpose of 1 -4 was to prevent a decedent from defeating his spouses right to a share. The purpose of 5 is to prevent a spouse who has been well provided for by lifetime or nonprobate transfers from electing against the will and claiming more than a fair share. (iii) To be equitable, the 1969 UPC includes gifts to the spouse in the decedents augmented estate, crediting them against the elective share to which the surviving spouse is entitled. 1. This approach remains in effect in many states, and the augmented estate also influenced many states to change their elective share systems even if they did not adopt the UPC (d) The 1990 UPC completely redesigned the elective share and the augmented estate so that it achieved results closer to those of a community property system, which the revisers took to be the desideratum. (i) The central idea of the 1990 UPC elective share is to add up all the property of both spouses and split according to a percentage based on the length of the marriage. 1. The revisers believed that this would result in treating spouses in common law jurisdictions in roughly the same way they are treated in community property jurisdictions (assuming that the spouses have only community property and each spouse owns half of their community property assets). 2. The 1990 UPC also adds to the augmented estate many transfers made before marriage, as well as transfers during marriage, where the decedent retained substantial control of the property. a. In this respect, the 1990 UPC resembles the Internal Revenue Code, which subjects to estate taxation property transferred by the decedent during life over which the decedent retained substantial control as well as property subject to a general power of appointment given the decedent by others i. The purpose of the augmented estate in the 1990 version is not to protect against fraud on the widows share but to bring the electiveshare law into line with the partnership theory of marriage, 8

tempered by a minimum support obligation in the form of $50,000.00 supplemental elective-share amount. ii. The official comment explains that the purpose is to protect longterm marriage surviving spouses where most of the property was in the decedents name or eliminate the entitlement of a surviving spouse where most of the assets were in his or her name. It also protects against the entitlement of a surviving spouse in a shortterm marriage, in which neither spouse contributed much. (e) UPC Sec. 2-202 (1990) Elective Share (i) (a) Elective-Share Amount. The surviving spouse of a decedent who dies domiciled in this State has a right of election, under the limitations and conditions sated in this Part, to take an elective-share amount equal to 50 percent of the value of the marital-property portion of the augmented estate. (f) UPC Sec. 2-203. Composition of the Augmented Estate; Marital-Property Portion (i) (a)*T+he value of the augmented estate.consists of the sum of the values of all property, whether real or personal, movable or immovable, tangible or intangible, wherever situated, that constitute: 1. The decedents net probate estate (defined in Sec. 2-204); 2. The decedents nonprobate transfers to others *defined in Sec. 2-205]; 3. The decedents nonprobate transfers to the surviving spouse *defined in Sec. 2-206]; and 4. The surviving spouses property and nonprobate transfers to others [defined in Sec. 2-207] (ii) (b) The value of the marital-property portion of the augmented estate consists of the sum of the values of the four components of the augmented estate as determined under subsection (a) multiplied by the following percentage: If the decedent and the spouse were The percentage is: married to each other: Less than a year 3% 1 year but less than 2 years 6% 2 years but less than 3 years 12% 3 years but less than 4 years 18% 4 years but less than 5 years 24% 5 years but less than 6 years 30% 6 years but less than 7 years 36% 7 years but less than 8 years 42% 8 years but less than 9 years 48% 9 years but less than 10 years 54% 10 years but less than 11 60% 11 but less than 12 68% 12 but less than 13 76% 13 but less than 14 84% 14 but less than 15 92% 9

15 years or more 100% (g) In funding the elective share amount, the UPC credits the surviving spouse with nonpropate transfers to the surviving spouse and marital assets that are already owned by the surviving spouse. UPC Sec. 2-209 (1990). (h) Note: Community property systems do not include property brought to the marriage or acquired by gift or inheritance, which is the separate property of the acquiring spouse (so long as it is kept separate). This property can be disposed of any way the decedent likes, the surviving spouse has no claim to it. (i) The 1990 UPC includes all property of both spouses so no inheritance or property kept from before marriage is out of the elective share augmented estate. 1. The 1990 UPC provisions on elective shares has been implemented in onefifth of the states, mainly in the Great Plains. (5) The Estate Tax (Marital Deduction) and the Dependency of Women (a) Congress solved the estate tax problem (community property law that takes advantage of large tax breaks not available to separate property law) by giving the husband an estate tax marital deduction, up to 50 percent of the value of his estate, for property left to his surviving wife in a form comparable to the outright ownership that a wife would have under community property. (i) However, this was an issue given that husbands felt that this would put their wives in a position of not being able to manage their wealth (as opposed to devising a trust for them). 1. The answer is that if the husband gave his wife a life estate (support) with the power to appoint the property to anyone she wished at her death (equivalent to complete ownership at her death), this arrangement would still qualify for the marital deduction (since this is viewed as giving a fee simple). (b) In 1982, the federal government allowed interspousal transfers to not be taxed at all, provided that the donor spouse gives the donee spouse at least a life estate in the property. (i) The above work-around is no longer needed for this tax break (ii) This current marital deduction requires only that the donor spouse create a trust giving his surviving spouse support for life to avoid transfer taxation (called QTIP trust). (6) Same-Sex Marriage and Domestic Partners (a) Unmarried surviving partners, even those in a relationship that approximates marriage, have none of these benefits. I.e. social security, pension rights, elective share all of which qualify for certain federal estate and gift tax marital deductions. (b) However, there is a growing trend toward recognizing same-sex marriage and civil unions, thus allowing these important beneficial legal and economic consequences. (7) An incompetent surviving spouse

10

(i) Under the support theory, if the incompetent surviving spouse does not need the property for her support, the representative should be barred from getting the forced share. (ii) Under a partnership theory, the representative should be entitled to do so because the forced share represents the surviving spouses portion of the marital partnership property, which ultimately should pass to the surviving spouse heirs or devisees. (b) In re Estate of Cross (i) Material Facts: An estate was left to a son and none to the elderly wife, who suffered from Alzheimers. A court appointed intervener took the forced share allowable and the son sued that they could not or should not do that against the estates wishes. (ii) Issue: Should a court be able to take a forced share for the incompetent surviving spouse? (iii) Holding: Yes. (iv) Reasoning: Under Ohio statute the court has latitude to determine whether or not to take a forced share based on a variety of circumstances. These include the age, probable life expectancy, physical and mental condition, and present and reasonably anticipated future needs of the surviving spouse and whether the taking is necessary to provide adequate support for the surviving spouse during his life expectancy. 1. Here, if the probate court had not taken against the will, under Ohio statute, the surviving spouse would not have been eligible for assisted living under Medicare since it requires that individuals utilize all potential income. Not doing so would in effect be deemed as an individual attempting to make themselves eligible for public assistance when in fact they are not. (v) Notes: 1. A minority of states hold that the guardian should elect to take against the will if it is to the surviving spouses economic benefit, calculated mathematically. The majority of states hold, as did the court in Cross, that all the surrounding facts and circumstances should be taken into consideration by the probate court. a. The majority approach allows the guardian to determine whether the surviving spouse would have wanted to abide by the decedents will and take into account the decedents estate plan. 2. UPC Sec. 2-203 (1969): A court order action against a will is only after finding that exercise is necessary to prove adequate support for the protected person during his probably life expectancy. a. Support Theory 3. UPC Sec. 2-212 (1990 rev. 2008): if a representative elects the elective share for an incompetent surviving spouse, the portion of the elective share that

11

exceeds what the dcedent spouse provided for the survivor must be placed in a custodial trust for the benefit of the surviving spouse. a. The trustee is given the power to expend income and principal for the surviving spouses support, but upon the psouses death the trustee must transfer the property to the residuary devisees under the will of the predeceased spouse of to the predeceased spouses heirs. i. The purpose is to assure that that part of the elective share is devoted to the personal economic benefit and needs of a surviving spouse, but not to the economic benefit of the surviving spouses heirs or devisees. b. The right of election may only be exercised by the surviving spouse or a representative of the surviving spouse during the surviving spouses life. 4. Creditors and Medicaid Eligibility a. The majority rule is that ordinary creditors of the surviving spouse cannot compel the surviving spouse to take a forced share. i. However, given Medicaid requires the value of elective share be counted toward the surviving spouses available resources the need to preserve benefits eligibility compels the surviving spouse to take the forced share 5. Abandonment a. In a minority of states the elective share is denied to individuals who abandoned or refused to support the deceased spouse. i. Proving abandonment can be difficult. ii. Moreover, the issue of whether elective share is allowable to divorced and separated couples differs from state to state and property theories. (i.e. in most community property states hold that earnings are still community property after separation.) iv) Waiver; prenuptial agreements. (1) Waiver: (a) The prototypical waiver occurs in a premarital agreement (sometimes called a prenuptial or antenuptial agreement). (i) All separate property states will enforce a waiver of the right of election by premarital agreement, and most will also enforce a waiver agreed to during the marriage (a postnuptial agreement). 1. Note: Many states still feel that this is not an arms-length bargain or even if they do, that it is still inequitable. (b) In just over half the states, the enforceability of a prenuptial agreement is governed by the UPAA (1983) (i) The party opposing the enforcement of a premarital agreement must prove that 1. It was not voluntary, or

12

2. Was unconscionable when executed and the party opposing enforcement did not have fair and reasonable disclosure of the other partys property and finances. (c) UPC (1990, as amended 1993) Sec. 2-213. Waiver of Right to Elect and of Other Rights (i) This provision adopts the enforcement standard of the UPAA and extends it to both prenuptial and postnuptial agreements. 1. (a) The right of election of a surviving spouse and the rights of the surviving spouse to homestead allowance, exempt property, and family allowance, or any of them, may be waived, wholly or partially, before or after marriage, by a written contract, agreement, or waiver signed by the surviving spouse. 2. (b) A surviving spouses waiver is not enforceable if the surviving spouse proves that: a. (1) he [or she] did not execute the waiver voluntarily; or b. (2) the waiver was unconscionable when it was executed and, before execution of the waiver, he [or she]: i. Was not provided a fair and reasonable disclosure of the property or financial obligations of the decedent; ii. Did not voluntarily and expressly waive, in writing, any right to disclosure of the property or financial obligations of the decedent beyond this disclosure provided; and iii. Did not have or reasonably could have had, an adequate knowledge of the property or financial obligations of the decedent. 3. (c) an issue of unconscionablility of a waiver is for decision by the court as a matter of law 4. (d) Unless it provides to the contrary, a waiver of all rights, or equivalent language, in the property or estate of a present or prospective spouse or a complete property settlement entered into after or in anticipation of separation or divorce is a waiver of all rights of elective share, homestead allowance, exempt property, and family allowance by each spouse in the property of the other and a renunciation by each of all benefits that would otherwise pass to him [or her] from the other by intestate succession or by virtue of any will executed before the waiver or property settlement. (d) Notes: (i) Fair and reasonable disclosure: 1. To avoid a subsequent challenge based on inadequate disclosure, it is advisable to prepare a schedule of each partys assets, including a good fair evaluation of each asset, and to attach the schedule to the agreement. (ii) Independent counsel: 1. Due to Barry Bonds prenuptial situation, the California legislature amended its enactment of the UPAA to require independent counsel (or a written, knowing waiver) and seven days notice before signing. 13

a. An increasing number of jurisdictions now require independent counsel, or the opportunity for independent counsel, as a condition for the enforcement of a premarital agreement. i. Note, however, that lack of independent counsel by itself is not enough to invalidate a premarital agreement, but it is a relevant factor in assessing voluntariness and adequacy of disclosure. (iii) Postnuptial versus prenuptial agreements. 1. Postnuptial agreements are looked at with closer scrutiny and in some states not enforced at all. b) Disclaimer i) When an heir or devisee declines to take the property, that refusal is called a disclaimer. ii) Disclaimers allow for post-mortem estate planning. (1) The most common motivations for disclaimer are to reduce taxes or to keep property from creditors. (a) At common law this could not be done, since refusal of property would lead to the next heir because there needed to be someone who was liable for the feudal obligations a reason once valid but of no importance today. (i) If an heir renounced his inheritance and the common law rule applied, the situation was treated as though the heir had received the intestate share and then made a taxable gift to the person who took by reason of the renunciation. (b) By Contrast, if a devisee disclaimed a testamentary gift, there were no gift tax consequences. (i) Almost all states have enacted disclaimer legislation that provides that the disclaimant is treated as having died before the decedent or before the time of distribution. 1. This fiction allows the decedents family to undertake post-mortem estate planning. iii) Simultaneous Death (1) Uniform Simultaneous Death Act (a) The original USDA (1940 rev. 1953), drafted to answer simultaneous death, provided that if there is no sufficient evidence of the order of deaths, the beneficiary is deem to have predeceased the donor. (i) Thus, neither inherits from the other (b) If two joint-tenants die simultaneously, one-half of the property is distributed as if one person survived and the other half in the same way. (i) This is the same for property held in tenancy by the entirety or community property. (c) Life insurance is distributed as though the insured survived the beneficiary. (d) The act was revised in 1991 because of ghastly cases involving the issue of sufficient evidence of the order of the deaths.

14

(e) Note: The simultaneous death problem arises more often in intestacy than elsewhere because well-drafted instruments typically require a beneficiary to survive the donor by a stated period of time (often 30 or 60 days). (i) Also, husbands and wives often travel together and are commonly each others primary beneficiary, the typical simultaneous death case involves spouses. (2) USDA 1991 (a) 120-Hour Rule (i) UPC Sec.2-104 and 2-702 (1990, rev. 2008) provide that an heir or devisee or life insurance beneficiary who fails to survive by 120 hours (5 days) is deemed to have predeceased the decedent. 1. The USDA was amended in 1991 to conform with the above provisions. iv) Slayer Statutes (1) UPC Sec. 2-803 bars the killer from succeeding to nonprobate as well as probate property. It also provides that a wrongful acquisition of property must be treated in accordance with the equitable principle that a killer cannot profit from his wrong. (a) The usual view is that the killer has predeceased the victim. (b) Under UPC Sec. 2-803 the killer is treated as having disclaimed the property, and under the UPC disclaimer statute, Sec. 2-1106 (2002, rev. 2006), the disclaimant is treated as having died immediately before the time of distribution. (i) In Estate of Covert a man who shot his wife and then killed himself, the court held that Edward could not take from his wifes estate. However, because Edwards devisees were innocent of Edwards crime, the court allowed them to take form the wifes estate. 1. Some states (Cal., RI, and Virginia) extend the bar by statue to the killers descendants. Others limit it by case law. (2) Is criminal conviction required? (a) UPC Sec.2-803 a final criminal conviction of a felonious and intentional killing is conclusive. (i) If the person is found guilty by a perponderonce of the evidence then they are barred. 1. Where a killer commits suicide, the killers estate may still be barred under this section. (ii) This seems to follow the majority view. 1. Note: A plea of guilty to a lesser crime than specified in the slayer statute did not prevent the killer from being barred in a civil proceeding. In re Estate of Cotton. 2. If the suspected killer appeals a judgment, there is a prima facie case of felonious and intentional killing, putting the burden on him to overcome it. In re Peterson. 3) THE LAWYERS OBLIGATIONS: TORT AND CONTRACT THEORIES a) Duties to Intended Beneficiaries i) Simpson v. Calivas, Supreme Court of New Hampshire, 1994 15

(1) Material Facts: A will stated that the wife would get a life estate in the homestead and everything else (residuary) would go to the son. The remaining portion of the land held the family business. The wife however argued that the will meant that she would receive all of the land, not just the home. The probate court ruled that it meant all the land. The son then buys out the mother for $400,000.00. The son then sues the lawyer for breach of duty of reasonable care. A theory in tort of: (a) Negligence: (i) Duty (ii) Breach (iii) Cause (iv) Harm (b) The son also stated a cause of action in breach of Contract: (i) Establish that there was an offer, acceptance, consideration, and breach? 1. Perhaps not between the son and the attorney, but between the decedent and the attorney a. The attorneys defense is that the Son was not in privity of the contract (2) Issue: (a) Does an attorney who drafts a testators will owes a duty of reasonable care to intended beneficiaries? (b) Does the 3rd party beneficiary have a right to breach of contract? (3) Holding: (a) There is such a duty (b) The task of the probate court is a limited one: to determine the intent of the testator as expressed in the language of the will. If it is ambiguous, extrinsic evidence may be presented to clarify the meaning of the will. (4) Reasoning: (a) In general, the scope of such a duty is limited to those in privity of contract with each other. However, we extend this duty where the harm to those persons not in privity is apparent. The overwhelming majority of courts have ruled that duty runs from an attorney to an intended beneficiary of a will, and we agree. (5) Notes: ii) Conflicts of Interest (1) Hotz v. Minyard, 304 S.C. 225 (1991) (a) Material Facts: Father asked attorney to lie to his daughter about the contents of his will and the attorney complied. The daughter sued the attorney on breach of fiduciary duty. (b) Issue: Does an attorney breach his fiduciary duty to his client by lying about the contents of a will for that client to his daughter, whom is also his client? (c) Holding: Yes (d) Reasoning: A fiduciary relationship exists when one has a special confidence in another so that the latter, in equity and good conscience, is bound to act in good faith. An attorney/client relationship is by nature a fiduciary one. While the attorney 16

was not the plaintiffs attorney regarding the contents of her fathers will, the attorney did have an ongoing attorney/client relationship with Judy and there was evidence that she had a special confidence in him. The attorney had a duty to deal in good faith and not actively misrepresent the first will to his client. (e) Notes: In such cases it is important to discuss with them at the outset possible conflicts of interests and the ground rules for sharing information. iii) Introduction to Intestacy: An Estate Plan by Default (1) Dying without a will leaves a person intestate. (a) This law governs the distribution of an intestate decedents probate property. These are the default rules that lawyers plan around. (i) A will, in addition to identifying who will take the decedents probate property, wills can designate guardians for minor children, identify a trustworthy individual or trust company to administer the estate, reduce probate costs by waiving a required bond (or surety on a bond), and achieve tax savings. 1. Even so, about half of adults do not have a will. a. This is due to variety of reasons: fear of confronting mortality, time and cost, etc. (b) The state governs intestate by statute of descent and distribution. (i) If the will only has part of the property disposed of, than it is called partial intestacy in which the part of the probate estate not disposed of by the will passes by intestacy. 1. Generally speaking, the law of the state where the decedent domiciled at death governs the disposition of personal property, and the law of the state where the decedents real property is located governs the disposition of real property. (2) UPC Sec. 2-101 (1990). Intestate Estate (a) Any part of a decedents estate not effectively disposed of by will passes by intestate succession to the decedents heirs as prescribed in this Code, except as modified by the decedents will (b) A decedent by will may expressly exclude or limit the right of an individual or class to succeed to property of the decedent passing by intestate succession. If that individual or a member of a class survives the decedent, the share of the decedents intestate estate to which that individual or class would have succeeded passes as if that individual or each member of that class had disclaimed his [or her] intestate share. iv) Intestate Share of a Surviving Spouse (1) The intestate share of a surviving spouse (a) UPC 2-102 (1969). Share of the Spouse. (i) The intestate share of the surviving spouse is: 1. (1) if there is no surviving issue or parent of the decedent, the entire intestate estate;

17

2. (2) if there is no surviving issue but the decedent is survived by a parent or parents, the first [$50,000], plus one-half of the balance of the intestate estate; 3. (3) if there are surviving issue all of whom are issue of the surviving spouse also, the first [$50,000], plus one-half of the balance of the intestate estate; 4. (4) if there are surviving issue one or more of whom are not issue of the surviving spouse, one-half of the intestate estate. (b) UPC 2-102 (1990). Share of Spouse (i) The intestate share of a decedents surviving spouse is 1. The entire intestate estate if: a. No surviving descendant or parent of the decedent survives; or b. All of the decedents surviving descendants are also descendants of the surviving spouse and there is no other descendant of the surviving spouse who survives the decedent; 2. The first [$300,000.00], plus three-fourths of any balance of the intestate estate, if no descendent of the decedent survives the decedent, but a parent of the decedent survives the decedent; 3. The first [$225,000.00], plus one-half of any balance of the intestate estate, if all of the decedents surviving descendants are also descendant of the surviving spouse and the surviving spouse has one or more surviving descendants who are not descendants of the decedent; 4. The first ($150,000.00), plus one-half of any balance of the intestate estate, if one or more of the decedents surviving descendants are not descendants of the surviving spouse (c) Share of Surviving Spouse (i) Under most current state laws, the surviving spouse usually receives at least one-half share of the decedents estate, an increase from the one-quarter or one-third of the estate that was typical a half century ago. (ii) UPC Sec.2-102(1) 1. If the surviving spouse has the same descendants of the decedent than she gets everything 2. If there are no descendants, half of the states and the UPC, provide that the estate is shared with the decedents parents, if any. (iii) The reason for these provisions is the idea of partnership theory. (2) Domestic Partners and intestate succession (a) This is the same as surviving spouse in so much as the state in which they domicile recognizes same-sex marriage/civil unions. (3) Rights of a spouse omitted from a premarital will (a) Most states override a premarital will in this way and give the surviving spouse an intestate portion of the will and follow the wills contents otherwise. (b) UPC Sec. 2-301 (1969) Omitted Spouse

18

(i) (a) If a testator fails to provide by will for his surviving spouse who married the testator after the execution of the will, the omitted spouse shall receive the same share of the estate he would have received if the decedent left no will unless it appears from the will that the omission was intentional or the testator provided for the spouse by transfer outside the will and the intent that the transfer be in lieu of a testamentary provision is shown by statements of the testator or from the amount of the transfer or other evidence. (ii) (b) In satisfying a share provided by this section, the devises made by the will abate as provided in Section 3-902. (c) UPC Sec. 2-301 (1990). Entitlement of Spouse; Premarital Will 1. (a) if a testators surviving spouse married the testator after the testator executed his [or her] will, the surviving spouse is entitled to receive, as an intestate share, no less than the value of the share of the estate he [or she] would have received if the testator had died intestate as to that portion of the testators estate, if any, that is neither devised to a child of the testator who was born before the testator married the surviving spouse and who is not a child of the surviving spouse nor devised to a descendant of such a child or passes under sections 2-603 or 2-604 to such a child or to a descendant of such a child, unless: a. (1) it appears form the will or other evidence that the will was made in contemplation of the testators marriage to the surviving spouse; b. (2) The will expresses the intention that it is to be effective notwithstanding any subsequent marriage; or c. (3) The testator provided for the spouse by transfer outside the will and the intent that the transfer be in lieu of a testamentary provision is shown by the testators statements or is reasonably inferred from the amount of the transfer or other evidence. 2. (b) In satisfying the share provided by this section, devises made by the will to the testators surviving spouse, if any, are applied first, and other devises, other than devise to a child of the testator who was born before the testator married the surviving spouse and who is not a child of the surviving spouse or a devisee or substitute gift under sections 3-603 or 2-604 to a descendant of such a child, abate as provided in section 3-902. v) Right of Descendants (1) The intestate share of surviving descendants (a) Sec. 2-103 (1990). Share of Heirs Other Than Surviving Spouse (i) (a) Any part of the intestate estate not passing to a decedent's surviving spouse under Section 2-102, or the entire intestate estate if there is no surviving spouse, passes in the following order to the individuals who survive the decedent: 1. (1) to the decedent's descendants by representation;

19

2. (2) if there is no surviving descendant, to the decedent's parents equally if both survive, or to the surviving parent if only one survives; 3. (3) if there is no surviving descendant or parent, to the descendants of the decedent's parents or either of them by representation; 4. (4) if there is no surviving descendant, parent, or descendant of a parent, but the decedent is survived on both the paternal and maternal sides by one or more grandparents or descendants of grandparents: a. (A) half to the decedent's paternal grandparents equally if both survive, to the surviving paternal grandparent if only one survives, or to the descendants of the decedent's paternal grandparents or either of them if both are deceased, the descendants taking by representation; and b. (B) half to the decedent's maternal grandparents equally if both survive, to the surviving maternal grandparent if only one survives, or to the descendants of the decedent's maternal grandparents or either of them if both are deceased, the descendants taking by representation; 5. (5) if there is no surviving descendant, parent, or descendant of a parent, but the decedent is survived by one or more grandparents or descendants of grandparents on the paternal but not the maternal side, or on the maternal but not the paternal side, to the decedent's relatives on the side with one or more surviving members in the manner described in paragraph (4). (ii) (b) If there is no taker under subsection (a), but the decedent has: 1. (1) one deceased spouse who has one or more descendants who survive the decedent, the estate or part thereof passes to that spouse's descendants by representation; or 2. (2) more than one deceased spouse who has one or more descendants who survive the decedent, an equal share of the estate or part thereof passes to each set of descendants by representation. (2) Taking by representation: per stirpes distribution (a) English Per Stirpes (i) Representation at each level (b) Modern Per Stirpes, or Per Capita with Representation (i) Start at the closest generation and divide per capita then continue with normal per stirpes representation. (c) UPC (1990) Sec. 2-106. Shares of Descendants (Per Capita at each Generation) (i) (a) [Definitions.] In this section: 1. (1) Deceased descendant, deceased parent, or deceased grandparent means a descendant, parent, or grandparent who either predeceased the decedent or is deemed to have predeceased the decedent under Section 2104.

20

2. (2) Surviving descendant means a descendant who neither predeceased the decedent nor is deemed to have predeceased the decedent under Section 2-104. (ii) (b) [Decedent's Descendants.] If, under Section 2-103(a)(1), a decedent's intestate estate or a part thereof passes by representation to the decedent's descendants, the estate or part thereof is divided into as many equal shares as there are (i) surviving descendants in the generation nearest to the decedent which contains one or more surviving descendants and (ii) deceased descendants in the same generation who left surviving descendants, if any. Each surviving descendant in the nearest generation is allocated one share. The remaining shares, if any, are combined and then divided in the same manner among the surviving descendants of the deceased descendants as if the surviving descendants who were allocated a share and their surviving descendants had predeceased the decedent. (iii) (c) [Descendants of Parents or Grandparents.] If, under Section 2-103(a)(3) or (4), a decedent's intestate estate or a part thereof passes by representation to the descendants of the decedent's deceased parents or either of them or to the descendants of the decedent's deceased paternal or maternal grandparents or either of them, the estate or part thereof is divided into as many equal shares as there are (i) surviving descendants in the generation nearest the deceased parents or either of them, or the deceased grandparents or either of them, that contains one or more surviving descendants and (ii) deceased descendants in the same generation who left surviving descendants, if any. Each surviving descendant in the nearest generation is allocated one share. The remaining shares, if any, are combined and then divided in the same manner among the surviving descendants of the deceased descendants as if the surviving descendants who were allocated a share and their surviving descendants had predeceased the decedent. (3) Rights of issue omitted from a will (a) Protection from intentional Omission (i) The Domestic Approach 1. In all states but Louisiana, there is no requirement that a testator leave any property whatsoever to a child. a. Even, so, doing so virtually invites a will contest based on testamentary capacity, undue influence, and fraud as examples. b. Moreover, in cases involving the disinheritance of children, judges and juries are usually sympathetic to the children thus many practicing lawyers will persuade their clients to an out-of-court settlement with the disinherited child. c. Notes: i. Does this power provide leverage for parents and grandparents to have their children take care of them in their old age? 21

ii. In Louisiana, a forced share for a child under 23, the mentally infirm and the disabled, can only be revoked due to a small number of exceptions (see page 521). (b) Protection from Unintentional Omission (i) Pretermission statutes prevent the unintentional disinheritance of descendants. (ii) Gray v. Gray, 947 So. 2d 1045 (2006) 1. Material Facts: A husband divorced his wife and set up a trust for the son of that marriage upon that divorce. However, his will at the time was that all of his belongings should go to that wife. Upon divorce, this was no longer applicable. The son, sought to take a portion of his fathers estate upon his death, even though he was omitted from the will. 2. Issue: Does Alabamas statute require that the son be given the intestate share of his fathers property where he was provided a trust during his lifetime, before his fathers death, and is otherwise omitted from the will? 3. Holding: No. 4. Reasoning: Alabamas statute is clear in that the exception for omitted children to receive inheritance is not applicable to those children who received funds through a trust while their parent was alive 5. Notes: a. The dissent strongly disagrees in that the wills intent was to give the inheritance to the sons then mother who would then pass it on to his son moreover, the statutes purpose is to contemplate this very circumstance. (iii) UPC Sec. 2-302 (1969). Pretermitted Children 1. (a) If a testator fails to provide in his will for any of his children born or adopted after the execution of his will, the omitted child receives a share in the estate equal in value to that which he would have received in the testator had died intestate unless: a. (1) it appears from the will that the omission was intentional; or b. (2) when the will was executed the testator had one or more children and devised substantially all his estate to the other parent of the omitted child; or c. (3) the testator provided for the child by transfer outside the will and the intent that the transfer be in lieu of a testamentary provision is shown by statements of the testator or from the amount of the transfer or other evidence. 2. (b) If at the time of the execution of the will the testator fails to provide in his will for a living child solely because he believes the child to be dead, the child receives a share in the estate equal in value to that which he would have received if the testator had died intestate. 3. (c) In satisfying a share provided by this section, the devises made by the will abate as provided in Section 3-902. 22

(iv) UPC (1990) Sec. 2-302. Omitted Children 1. (a) Except as provided in subsection (b), if a testator fails to provide in his [or her] will for any of his [or her] children born or adopted after the execution of the will, the omitted after-born or after-adopted child receives a share in the estate as follows: a. (1) If the testator had no child living when he [or she] executed the will, an omitted after-born or after-adopted child receives a share in the estate equal in value to that which the child would have received had the testator died intestate, unless the will devised all or substantially all of the estate to the other parent of the omitted child and that other parent survives the testator and is entitled to take under the will. b. (2) If the testator had one or more children living when he [or she] executed the will, and the will devised property or an interest in property to one or more of the then-living children, an omitted afterborn or after-adopted child is entitled to share in the testator's estate as follows: i. (A) The portion of the testator's estate in which the omitted afterborn or after-adopted child is entitled to share is limited to devises made to the testator's then-living children under the will. ii. (B) The omitted after-born or after-adopted child is entitled to receive the share of the testator's estate, as limited in subparagraph (A), that the child would have received had the testator included all omitted after-born and after-adopted children with the children to whom devises were made under the will and had given an equal share of the estate to each child. iii. (C) To the extent feasible, the interest granted an omitted afterborn or after-adopted child under this section must be of the same character, whether equitable or legal, present or future, as that devised to the testator's then-living children under the will. iv. (D) In satisfying a share provided by this paragraph, devises to the testator's children who were living when the will was executed abate ratably. In abating the devises of the then-living children, the court shall preserve to the maximum extent possible the character of the testamentary plan adopted by the testator. 2. (b) Neither subsection (a)(1) nor subsection (a)(2) applies if: a. (1) it appears from the will that the omission was intentional; or b. (2) the testator provided for the omitted after-born or after-adopted child by transfer outside the will and the intent that the transfer be in lieu of a testamentary provision is shown by the testator's statements or is reasonably inferred from the amount of the transfer or other evidence.

23

3. (c) If at the time of execution of the will the testator fails to provide in his [or her] will for a living child solely because he [or she] believes the child to be dead, the child is entitled to share in the estate as if the child were an omitted after-born or after-adopted child. 4. (d) In satisfying a share provided by subsection (a)(1), devises made by the will abate under Section 3-902. (v) Missouri Type Pretermitted Heir Statutes 1. It must appear from the will itself that the omission of a child or other heir was intentional. Extrinsic evidence of intent is not admissible. (vi) Massachusetts Type Pretermitted Heir Statutes 1. The child takes unless it appears that such omission was intentional and not occasioned by any mistake. Thus, extrinsic evidence is admitted to show the presence or absence of intent to disinherit. (vii) Pretermitted Child statutes follow one of two patterns 1. Like UPC 2-302, they protect only children born or adopted after execution of the will 2. Other statutes operate in favor of children alive when the will was executed as well as after born children a. The failure to name all of the testators living children under this stance invites a challenge under the pretermitted child statute i. This issue is more acute in a minority of states that apply the statute to not only children, but descendants as well ii. Some take the stance that disinheriting ones initial descendants would also follow that their children are also disinherited while others do not. vi) The intestate Share of Ancestors and Collaterals (1) Shares of Ancestors and collaterals (a) In the case that there is no spouse or parents, a decedents heirs will be more remote ancestors or collateral kindred (i) All persons related by blood to the decedent but who are not descendants or ancestors are called collateral kindred. 1. First line Collaterals a. Descendants of the decedents parents, other than the decedent and the decedents descendants 2. Second-line Collaterals a. Descendants of the decedents grandparents, other than the decedents parents and their descendants 3. Etc. See table on page 93 (b) Parentelic System (See page 93 for flow chart) (i) The intstate estate passes to grandparents and their descendants, and if noe to great grandparents and their descendants, and if none to the great-great-grand

24

parents and their descendants, and so on down the line descended from an ancestor until an heir is found. (c) Degree-of-relationship System (See page 93 for Degrees) (i) The intestate estate passes to the closet of kin, counting degrees kinship 1. To ascertain the degree of relationship of the decedent to the claimant you count the steps (counting one for each generation) up from the decedent to the nearest common ancestor of the decedent and the claimant, and then you count the steps down to the claimant from the common ancestor. a. The total number of steps is the degree of relationship i. The potential heir with the smallest number inherits the entire estate. (d) There are many variations of these two systems spread across many states. (e) The number of collateral kindred is immense (i) We have two hundred and seventy millions of kindred in the fifteenth degree 1. Should we allow intestate succession to these so-called laughing-heirs? a. At least half the states have done so, stopping the succession at the grand-parents heirs b. UPC 2-103(a), see Rights of Descendants c. UPC as revised in 2008 have created a new class of heirs known as step children (f) If there are no survivors entitled to take under the intestacy statute, the estate escheats to the state (i) Escheats of substantial estates are rare 1. Also, heir searching firms offer to provide information about whether a person is an heir to a great estate for a fee. (2) UPC Sec. 2-103 (1990). (See Rights of Descendants) (3) UPC Sec. 2-105 (1990). (a) If there is no taker under the provisions of this [article], the intestate estate passes to the state. vii) Additional issues involving children (1) Adopted children (a) UPC Sec. 2-116 (1990). Effect of Parent-Child Relationship (i) Except as otherwise provided in Section 2-119(b) through (e), if a parent-child relationship exists or is established under this [subpart], the parent is a parent of the child and the child is a child of the parent for the purpose of intestate succession. (b) UPC Sec. 2-118(a) (1990). Adoptee and Adoptees Adoptive Parent or Parents. (i) (a) [Parent-Child Relationship Between Adoptee and Adoptive Parent or Parents.] A parent-child relationship exists between an adoptee and the adoptee's adoptive parent or parents. (c) UPC Sec. 2-119 (1990). Adoptee and Adoptees genetic Parents.

25

(i) (a) [Parent-Child Relationship Between Adoptee and Genetic Parents.] Except as otherwise provided in subsections (b) through (e), a parent-child relationship does not exist between an adoptee and the adoptee's genetic parents. (ii) (b) [Stepchild Adopted by Stepparent.] A parent-child relationship exists between an individual who is adopted by the spouse of either genetic parent and: 1. (1) the genetic parent whose spouse adopted the individual; and 2. (2) the other genetic parent, but only for the purpose of the right of the adoptee or a descendant of the adoptee to inherit from or through the other genetic parent. (iii) (c) [Individual Adopted by Relative of Genetic Parent.] A parent-child relationship exists between both genetic parents and an individual who is adopted by a relative of a genetic parent, or by the spouse or surviving spouse of a relative of a genetic parent, but only for the purpose of the right of the adoptee or a descendant of the adoptee to inherit from or through either genetic parent. (iv) (d) [Individual Adopted after Death of Both Genetic Parents.] A parent-child relationship exists between both genetic parents and an individual who is adopted after the death of both genetic parents, but only for the purpose of the right of the adoptee or a descendant of the adoptee to inherit through either genetic parent. (v) (e) [Child of Assisted Reproduction or Gestational Child Who Is Subsequently Adopted.] If, after a parent-child relationship is established between a child of assisted reproduction and a parent or parents under Section 2-120 or between a gestational child and a parent or parents under Section 2-121, the child is adopted by another or others, the child's parent or parents under Section 2-120 or 2-121 are treated as the child's genetic parent or parents for the purpose of this section. (d) Hall v. Vallandingham, 540 A2d 1162 (1988) (i) Material Facts: Four childrens father died and the wife remarried. The new husband adopted the children. Many years later, their natural uncle died and they argued that they were entitled to their natural fathers representative share. (ii) Issue: Do adopted children have the representative right to their families estate through intestate succession? (iii) Holding: No. (iv) Reasoning: Maryland statutes, read together, disallow situations such as this where people could effectively get dual inheritance. (v) Notes: 1. This holding doesnt truly prevent dual inheritance if the timing of deaths is slightly different than the children could effectively dual inherit.

26

2. This varies widely from state to state some allow dual inheritance outright while others, including UPC Sec. 2114(b) (1990), an adopted child can inherit from natural and adoptive parents if the adoptive parent is a stepparent. a. There are many statutory variations on these themes. 3. Note: Under the 2008 amendments to the UPC, the threshold question is if there is a child-parent relationship. a. If such a relationship exists, the parent is a parent of the child and the child is a child of the parent for the purpose of intestate succession by, from, or through the parent or the child (Sec. 2-116). b. Regarding adoption, a parent child relationship exists between an adopted child and the adoptive parent (Sec. 2-118(a)), but not between an adopted child and the childs genetic parents (Sec. 2-119(a)), the latter subject to several exceptions (see Sec. 2-119 above). 4. Adult Adoption. a. Most intestacy statutes draw no distinction between the adoption of a minor and the adoption. (Some states do not allow the adoption of a lover) i. The reason for adopting a lover would to prevent contests to the will (or if you were in a homosexual relationship and the state did not recognize the right) 5. Adoption and the interpretation of wills and trusts. a. In most states, a minor adopted by A is presumptively included in a gift by T to the children, issue, descendants, or heirs of A. (2) Posthumous children (a) Uniform parentage Act Sec. 204 (2000, rev. 2002) establishes a rebuttable presumption that a child born to a woman within 300 days after the death of her husband is a child of that husband. (3) Nonmarital children (a) The law allows inheritance from the mother (b) The law for inheritance of the father vary (i) Most allow this inheritance by evidence of the subsequent marriage of the parents, by acknowledgment by the father, by adjudication during the life of the father, or by clear and convincing proof after his death. (4) Advancements (a) At common law, any lifetime gift by the decedent to a child was considered an advancement in effect, a prepayment of the childs intestate share. (i) To avoid the application of this doctrine a child had the burden of proof that this was actually just an absolute gift that was not counted toward the childs share of the estate. 1. The rationale is that parents want equal distribution to all children, and the only way to do this is to take into account the lifetime gifts given to children in determining the amount of the equal shares. 27

2. When a child predeceases the parent, the amount of the advancement is deducted from the shares of the childs descendants if other children of the parent survive. (ii) If a gift is treated as an advancement it is accounted for in distributing the decedents estate by bringing it into a hotchpot. 1. If a child receives a $10,000.0 advancement, this is amount is put back into the estate when determining how much of the estate that child is entitled to a. i.e. Daughter has advancement of 10k and her father dies leaving 50k for his three children. The 10k is added making 60k and each child receives their 20k except the daughter who only gets 10k since she already received 10k in advancement i. Note: If she received the amount in property as an advancement, it would not be added since we know the parent wanted that child to have that property. (iii) Note: Many states have reversed the common law presumption of advancement. In fact, many states, including the UPC Sec. 2-109(a) (1990), go further requiring that amounts meant to be an advancement must be set forth as such in writing. (b) UPC Sec. 2-109. Advancements (i) (a) If an individual dies intestate as to all or a portion of his [or her] estate, property the decedent gave during the decedent's lifetime to an individual who, at the decedent's death, is an heir is treated as an advancement against the heir's intestate share only if (i) the decedent declared in a contemporaneous writing or the heir acknowledged in writing that the gift is an advancement, or (ii) the decedent's contemporaneous writing or the heir's written acknowledgment otherwise indicates that the gift is to be taken into account in computing the division and distribution of the decedent's intestate estate. (ii) (b) For purposes of subsection (a), property advanced is valued as of the time the heir came into possession or enjoyment of the property or as of the time of the decedent's death, whichever first occurs. (iii) (c) If the recipient of the property fails to survive the decedent, the property is not taken into account in computing the division and distribution of the decedent's intestate estate, unless the decedent's contemporaneous writing provides otherwise. (c) UPC Sec. 2-109(c) changes the common law rule if the recipient does not survive the decedent in that case, the advancement is not taken into account in determining the share of the recipients descendants (i) Requiring a rule for writing of advancements all by eliminates them from the law of intestate succession which avoids contentious litigation between family members. (5) Managing a minors property 28

(a) Guardianship and Conservatorship of Minors (i) A minor has neither the legal capacity to manage property nor the legal power to make most choices about how and where to live. 1. Clients with young children should be advised to provide for the possibility that their children might be orphaned a possibility that must be confronted by rich and poor clients alike. (ii) Guardian of the Person 1. A guardian of the person has responsibility for the minor childs custody and care. If both parents die the court will appoint a guardian, usually from among the nearest relatives this person may not be whom the parents want 2. Accordingly, one of the principal reasons for making a will is to make sure that a minor child is raised by a guardian who is willing and one whom the parents want to take care of the child in the case that both die before that child reaches adulthood. Gaurdianship of a minor is covered under UPC Sec. 5-201 through 5-210 (not assigned) (iii) Property Management Options 1. This deals with taking care of the minor childs property several alternatives for property management are available a. Guardianship of property i. A guardian of property is responsible for handing over the property when the child reaches 18 and can only make decisions under strict court supervision ii. Since each trip incurs attorneys fees and costs, and is somewhat like going through continuous probate until the child reaches the age of the majority, it should be avoided b. Conservatorship i. Appointment and supervision by the court is still required, buy the conservator has far more flexible powers than a guardian, and only one trip to the courthouse annually for an accounting may be necessary. ii. This permits a more streamlined administration of the estate, allowing a higher net return on the assets, more flexibility in investments, and a greater chance of meeting the financial needs of the child, both while a minor and on termination of the conservatorship. See UPC Sec. 5-431 (not assigned) c. Custodianship i. A custodian is a person who is given property to hold for the benefit of a minor under the UTMA (1983) or the UGMA (1956). ii. The creation is quite simple: Most banks, brokers and other financial institutions have forms that can be filled out. Also, a will can include a facility of payment clause whereby assets will be 29

distributed outright to a minor may be paid instead directly to a custodian or even the parent or guardian of a minor iii. The custodian has the right to manage the property and to reinvest it. However, the custodian is a fiduciary and is subject to the standard of care that would be observed by a prudent person dealing with property of another. UTMA Sec. 12(b). iv. The Custodian is not under the supervision of the court, and no accounting is necessary, but an interested party may require it if he wishes. v. The amounts must be generally small - $5,000.00 per year (UPC Sec. 5-104 (1998) and payments to custodians over $10,000.00 need court approval when a large amount is involved a trust is preferable. d. Trusteeship i. A trust is the most flexible of all property arrangements. The donor can tailor the trust specifically to family circumstances and the donors particular desires. ii. A trust can postpone possession until the donor thinks the child is competent to manage the property. iii. Even when a person has not children, most well-designed estate plans provide for a contingent trust in the event that there is a minor beneficiary, perhaps because a named adult beneficiary predeceases the testator. iv. There is thus no such thing as a simple will in a good law firm you must account for the possibility of minors and the best way to do so is through a trust. 4) FORMALITIES OF WILLS: CAPACITY, EXECUTION, COMPONENT PARTS, AND REVOCATION a) Contest of Wills. i) In re Estate of Getty, 85 Cal. App. 3d 755 (1978) (1) Material Facts: A man instituted his 21st codicil of his will naming certain person executors and trustees of the will. The granddaughter, a contingent trustee for all trusts under the will, contested the final codicil under capacity and undue influence doctrines. The issue was whether she was an interested person under the probate code to have standing to contest the will. (2) Issue: Does a contingent trustee have standing under the probate code to challenge a wills specific codicil created in this case? (3) Holding: No. (4) Reasoning: An interested person under the probate code is someone who would be impaired or defeated by the probate of the will, or benefitted by setting it aside. The granddaughter is none of these things. Legal interests which stem from management of the corpus, are not rights to the estate res that constitute sufficient pecuniary interest

30

to give standing to the omitted trustees to contest the codicil without reference to the effect of the subsequent codicil on the beneficiary. (5) Notes: (a) Why was there not standing? (i) There was no changing of the interest of the amounts going to the museum in fact they would receive more under the current will interpretation (b) The courts position of the fees associated with the execution of the will were not positive they thought this was little work that did not warrant such outrageous fees (i) Difficult executor issues involve a person with many tiny assets, not one large lump sum going to a single beneficiary (c) Policy Issue: If the testators wishes are the most important thing to consider when executing a will, why does the court not allow standing for what the person actually wanted? Dont we allow testators to do what they want with almost no limitations? (d) This was challenged by the granddaughter for the purpose of doing what she thought her grandfather actually wanted to happen with his will. ii) Planning for and Avoiding a Will Contest (1) The most common grounds for a will contest are lack of capacity and undue influence (a) Reasons why this is more common than in England or other European Countries (i) The U.S. potential winnings will likely outweigh the statutory amount required by law. England has a statutory minimum (ii) There is usually no cost for litigation unless you win (iii) Civil law allows for evidence of a decedents exceptional capacity to be kept with the will. (2) A contest is more likely to succeed in a jury trial, where they are sympathetic, as opposed to a bench trial. (3) Warning signs: (a) An eccentric client takes radical departures from previous plans (b) Testator has multiple or blended families arising from multiple marriages (c) Imposed conditions on bequest that would make a beneficiary bristle (d) If the testator makes a disposition to a person or group unpopular with the family (e) Unnatural disposition (i) Omission of close family member or an unexplainable distinction among family members of equal relation. 1. A omitted child would read the will as a rejection by the parent and would seek to contest (4) Strategies: (a) As a lawyer, make the client write out what he wants and respond with all the possible implications. Then draft the will as the client wishes, keeping both letters with the will so that there is evidence that the testator knew what he was getting into before writing the will.

31

(b) Use of video or stenographer (for the extremely elderly, since they could look badly on video) (c) Hold a family meeting at which the testator explains his rationale (d) Arrange a professional examination (e) Use disinterested witnesses who will present the soundness of the will on testimony (i) Also, have the testator discuss the will in detail and his rationale for it (f) Suggest a no-contest clause (i) However, this will not really work unless the testator is willing to make a substantial bequest to the potential contestant (g) Instead of transferring through a will, they could do so through an inter-vivos trust. (i) However, these too can be contested on the same grounds as a will 1. Though, as a practical matter, it is more difficult to contest a trust 2. Moreover, most states allow this to be done in secret, so those who may contest the trust may be too late to do so. 3. Also, almost all trust contests are tried before a judge who is less likely to overturn it (h) Inter-vivos gift (i) While this can be contested, potentials beneficiaries would not want to contest it and possibly upset the donor who can still write them out of their will (i) Could always cut a check to each potential contestant. (i) Upon cashing the check they could open themselves up to vigorous crossexamination however this looks, and truly is, a lawyers gimmick. (j) Notes: (i) Some have suggested mediation to keep the contest out of the public eye, try to keep family members speaking to each other and lesson litigation costs 1. Should courts order these? Should you include a clause in the will to make sure that any contest goes to mediation b) Mental Capacity and Undue Influence. i) Mental capacity in general. (1) The test of Mental Capacity (a) The test for mental capacity is minimal. To be competent to make a will, the testator must be an adult (age 18 or older) and must be capable of knowing and understanding in a general way; 1. The nature and extent of his or her property, 2. The natural objects of his or her bounty, and 3. The disposition that he or she is making of that property, and must be capable of 4. Relating these elements to one an 5. other and forming an orderly desire regarding the disposition of the property a. Restatement (Third) of Property Sec. 8.1(b) (2003). (ii) The test is one of capability, not actual knowledge. 32

1. The testator does not need to be of average intelligence, as this would incapacitate half the population (b) In re Estate of Washburn, 690 A.2d 1024 (1997) (i) Material Facts: Woman with developing Alzheimers changes her will within three weeks to remover the original beneficiary and gives her estate to her care giver. (ii) Issue: Did the testator have sufficient capacity at the time of making the will in order to give it to her care giver? (iii) Holding: No (iv) Reasoning: Sufficient witnesses, experts and evidence was given to show that the testator was not of sound mind at the time of creating the will. She did not remember certain family members and was unaware of her property. (v) Notes: 1. There is perhaps a whiff of undue influence here. 2. The challenger here was the niece, and the motive was for money. 3. Note that proponent had to prove that she was not incompetent in this case. 4. Practical Happenings of a Will: a. Will goes to court b. They look at will and make sure that it is O.K. by State Law c. Then it is probated unless there is a challenge i. Someone has to challenge ii. They usually have the burden of proving, usually, that the will doesnt work. iii. Washburn and Wilson have different standards of proof. (c) Wilson v. Lane, 614 S.E.2d 88 (2005) (i) Material Facts: An older womans last will and testament was challenged due to the fact that she was suffering from early signs of dementia, had several eccentric habits, was forgetful and called the fire-department at one time to put out a fire that didnt exist. There was also evidence that she had a strong fear of water. The will that she executed included several of her blood relatives and one caregiver the amounts were not specified. (ii) Issue: Does the evidence above, taken as a whole, render the decedent incapable of writing a valid will? (iii) Holding: No. (iv) Reasoning: A person is mentally capable to make a will if she has sufficient intellect to enable [her] to have a decided and rational desire as to the disposition of her property. The law does not withhold from the aged, the feeble, the weak-minded, the capricious, the notionate, the right to make a will, provided such person has a decided and rational desire as to the disposition of his property.

33

1. Eccentric habits and absurd beliefs do not establish testamentary incapacity. All that is required to sustain the will is proof that the decedent was capable of forming a certain rational desire with respect to the disposition of her assets. 2. An expert stated that she appeared to have early stages of Alzheimers but did not show how it would have made her unable to form a rational desire regarding the disposition of her assets. 3. Finally, even if the decedent had an inability to live alone at the time the will was executed, that fact bears no relation to her ability to form a rational desire regarding the disposition of her assets. (v) Notes: 1. Dissent: A court must allow the issue of testamentary capacity to go to the jury when there is a genuine conflict in the evidence regarding the testators state of mind. We should have decided the case based on the juries interpretation of the evidence and understanding of the facts not overturned that principle. 2. Evidentiary Burdens: Putting the burden of proof (or burden of persuasion) on the proponent is the minority view, as it was in Washburn. a. The prevailing rule, as in Wilson and Breedon v. Stone (page 171), is that once the proponent adduces prima facie evidence of due execution, the party contesting the will on the grounds of lack of capacity as the burden of persuasion. i. Restatement (Third) of Property: Wills and Other Donative Transfers Sec. 8.1, cmt. F (2003); ii. UPC Sec. 3-407 (1990) 3. Both Washburn and Wilson had very similar facts however the outcome was different a. This may have been due to the contents of the will (in the latter case, the caregiver was viewed as part of the family) and also whom the burden of persuasion is ultimately rested on. b. Also, these cases used different standards of proof. 4. Professional Responsibility: A lawyer may not draft a will for a person the lawyer believes to be incompetent, but the lawyer may rely on her own judgment of the clients capacity. a. However, because of the importance of testamentary freedom, the lawyer may properly assist clients whose testamentary capacity appears to be borderline. b. In any case, the lawyer should take steps to preserve evidence regarding the clients testamentary capacity. *ACTEC Commentaries on the Model Rules of Professional Conduct 132 (4th ed. 2006)]

34

5. Ante-mortem Probate: Arkansas, North Dakota, and Ohio have statutes that allow probate of a will during the testators life. All beneficiaries become parties to the action when this occurs. a. This procedure is rarely invoked (d) Capacity Thresholds (i) The capacity standards for contracts and gifts are designed to protect the incompetent contractor or donor from suffering economic loss during lifetime, which might result in impoverishment. (ii) Restatement (Third) of Property: Wills and Other Donative Transfers Sec. 8.1(c) (2003) makes this higher standard explicit: To make an irrevocable lifetime gift, not only must on have capacity to make a will, but one must also be capable of understanding the effect that the gift may have on the future financial security of the donor and of anyone who may be dependent on the donor. 1. For a dead person, this worry of impoverishment is a less important consideration. Hence the capacity to make a will is lower than that of contract or an irrevocable lifetime gift. a. The modern view is that this lower capacity standard also applies to the making of a revocable trust or other will substitute that is revocable until death. (iii) In Lee v. Lee, the court reasoned that a person under a conservatorship is without the necessary contractual power to execute a deed, but such a person may nonetheless write a valid will if the trial court finds, as the trial court did here that the will was written during a lucid interval. (iv) Interestingly, legal capacity to make a will requires a greater mental competency than is required for marriage. 1. Since a spouse has a right to the decedents property, isnt this similar to making a will? (e) Why Require Mental Capacity? (i) It assures a sane person that the disposition he desires will be carried out even if he later becomes insane and makes another will. (ii) The law also requires mental capacity to protect the decedents family. (iii) Moreover, the public acceptance of laws rests upon a belief that legal institutions, including inheritance, are legitimate, and legitimacy cannot exist unless decisions are reasoned. 1. On this view, it is important that the succession to property be perceives as a responsible, reasoned act, according the survivors their just deserts. ii) Insane delusion. (1) A Person may have sufficient mental capacity generally to execute a will but be suffering from an insane delusion so as to cause a will to fail for lack of testamentary capacity nonetheless. If an insane delusion is shown, but the delusion did not affect the dispositions, then the will stands.

35

(a) Therefore, much of the insane delusion deals with causation; typically the testators insane delusion involves a false belief about a member of the testators family. (i) This is a legal, not a psychiatric, concept. 1. A delusion is a false conception of reality. An insane delusion is the testator adheres against all evidence and reason to the contrary. a. Thus there is a line between insane delusion and mistake. Under traditional law (although it is changing) a will cannot be changed due to mistake. b. If there is any factual basis at all for the testators delusion, it is not deemed insane. c. However, the majority view is that a delusion is insane even if there is some factual basis for it if a rational person in the testators situation could not have drawn the conclusion reached by the testator. (Restatement (Third) of Property: Wills and Other Donative Transfers Sec. 8.1, cmt. S (2003)). (2) In re Strittmater, 53 A.2d 205, (1947) (a) Material Facts: Testator left her estate to the National Womens Party. Her only relatives, who had not seen her in several years, contested the will on account of her perceived split personality insane delusion that men were evil. Much of the evidence came from a single doctor the decedents regular doctor and from passages she had written in books kept within her home. (b) Issue: Is this enough to render her will invalid? (c) Holding: Yes. (d) Reasoning: Her insane delusions about males are what led her to leave her estate to this party, not a reasoned decision to support the group. (e) Notes: This court is very biased, and it stands to reason that this would not come out in the same way had it been reviewed by a modern court. (f) Kostic v. Chaplin (2007) (i) The court struck down one of the largest gifts to a political party when the testator believed that the head of that party could save the world from a satanic plot. He had cut his sons from the will, since he believed they, along with other family members, were out to kill him based on some satanic plot. 1. Note: The man was very smart read and spoke five different languages as well as having a love of all academia. (3) Breeden v. Stone, 992 P.2d 1167 (200) (a) Material Facts: Decedent created a holygraphic (hand written) will giving all his assets to his friend. His family contested the will on the grounds that he did not have the requisite testamentary capacity to properly write a will in the way he truly intended. The decedent was a heavy cocaine and alcohol abuser who had just been charged for killing another person in a car crash incident. His moods were alternately euphoric, fearful, and depressed and there was evidence of that he

36

excessively worried about threats against himself and his dog from government agents, friends, and others. Decedent, after writing the will, committed suicide. (b) Issue: Did the decedent have the proper testamentary capacity to draft his own holographic will and thus have it enforced in probate court? (c) Holding: Yes. (d) Reasoning: Experts concluded that he had the requisite motor skills to write his will. He had also told his friends several weeks before committing suicide that he did not intend on leaving anything to his family. Moreover, his insane delusion did not affect how he disposed of his property. (i) First: Once a proponent of a will has offered prima facie proof that the will was duly executed, the contestant then assumes the burden of proving a lack of testamentary capacity, including a lack of sound mind, by a preponderance of the evidence. Two lines of inquiry determine what constitutes a sound mind 1. The Cunningham Test a. The mental capacity to create a will requires that : (1) the testator understands the nature of her act; (2) she knows the extent of her property; (3) she understands the proposed testamentary disposition; (4) she knows the natural objects of her bounty; and (5) the will represents her wishes. i. This is a catch-all test if one element is not satisfied, then the testator lacked the requisite mental capacity to create a will. 2. The Insane Delusion Test a. Insane delusion is a persistent belief in that which has no existence in fact, and which is adhered to against all evidence. A party asserting that a testator was suffering from an insane delusion must meet the burden of showing that the testator suffered from such a delusion. i. However, one may have insane delusions regarding some matters and be insane on some subjects, yet be capable of transacting business concerning matters wherein such objects are not concerned, and such insanity does not make on incompetent to contract unless the subject matter of the contract is so connected with an insane delusion as to render the afflicted party incapable of understanding the nature and effect of the agreement or of acting in the transaction. ii. Thus, the question is whether the insane delusion materially affects the contested disposition in the will. 3. These two tests are not mutually exclusive a. The Cunningham test is commonly applied to cases where the testator may have lacked general testamentary capacity due to a number of causes such as mental illness, physical infirmity, senile dementia, and general insanity.

37

b. The Insane Delusion test, on the other hand, ordinarily involves situations in which the testator, although in possession of his general faculties, suffers from delusions that often take the form of monomania or paranoia. i. Thus, the testator must have the capacities from Cunningham and the absence of insane delusions that materially affect the will. ii. A testator may test a testators soundness of mind using either or both of these tests. 4. The decedent could index the major categories of his property, knew his home address and rental addresses, and identified the devisee by name and provided her current address. The will was logical, legible and reasonably set out the testators intent. He had control of his motor skills and his handwriting was unremarkable. As to the insane delusion, it did not affect the contents of his will. (e) Notes: (i) In most jurisdictions the contestant must show that (1) the testator labored under an insane delusion, and (2) the will (or some part thereof) was a product of the insane delusion. (ii) In a minority of jurisdictions, the courts apply a lower standard for causation the dispositive provisions might have been caused or affected by the testators insane delusion. 1. In Honingman, 168 N.E.2d 676 (1960), the court found that even if the testators insane delusion about the infidelity of his wife and there were still other good alternatives for why he left her the minimum forced share. She had an independent fortune and his siblings had financial needs. a. Note: The court assumed causation by putting the burden on the proponent to show that an unnatural disposition arguably within the insane delusions was not in fact a product of that delusion, a burden the brothers and sisters could not meet. (iii) Do courts enforce a moral doctrine, that testators should leave their things in accordance with normative views, through covert manipulation of insane delusion and capacity doctrine? 1. It would seem that those wills not in line with normative views are automatically suspect and must be proved to be valid against these normative views. (iv) There is a Dead Mans Statute which does not allow an interested partys testimony of a statement the decedent made against the estate. Most states have abrogated this statute, including the Federal Rules of Evidence, while others have replaced it with a more permissive rule, giving the court discretion to allow it or not. (v) This man was drunk when he wrote the will and still passed the test. (vi) Why is this outcome so different from Strittmater? 38

iii) Undue influence. (1) Introduction (a) Lord Hannens Explanation of Undue Influence (i) There must be coercion. 1. It is only when the will of the person who becomes a testator is coerced into doing that which he or she does not desire to do, that it is undue influence. 2. The coercion may be of different kinds; ranging from actual confinement or violence, or the person may be so late in life that they have become so weak and feeble, that a very little pressure will be sufficient to bring about the desired result, and it may even be, that the mere talking to him at that stage of illness and pressing something upon him may so fatigue the brain, that the sick person may be induced, for quietness sake, to do anything. (b) Undue influence may occur where there is a confidential relationship between the parties or where there is no such relationship. (i) Proof may be wholly inferential and circumstantial 1. If part of the will is the product of undue influence, those portions of the will that are the product of such influence may be stricken and the remainder of the will allowed to stand, if the invalid portions of the will can be separated without defeating the testators intent or destroying the testamentary scheme. a. An inter vivos transfer that is a product of undue influence is likewise invalid. (2) Restatement (Third) of Property: Wills and Other Donative Transfers (2003) (a) Sec. 8.3. Undue Influence, Duress, or Fruad (i) (a) a donative transfer is invalid to the extent it was procured by undue influence, duress or fraud. (ii) (b) it is procured by undue influence if the wrongdoer exerted such influence over the donor that it overcame the donors free will and caused the donor to make a donative transfer that the donor would not otherwise have made (iii) Comment: 1. (e) Undue Influence: This doctrine protects against overreaching by a wrongdoer seeking to take unfair advantage of a donor who is susceptible to such wrongdoing on account of the donors age, inexperience, dependence, physical or mental weakness, or other factor. It is undue if the influence overcame the donors free will and caused the donor to make a donative transfer that the donor would not otherwise have madethe contestants case must usually be based on circumstantial evidence, and in certain cases, is aided by a presumption of undue influence. a. Circumstantial evidence is sufficient to raise an inference of undue influence if the contestant proves that i. (1) the donor was susceptible to undue influence

39

ii. (2) the alleged wrongdoer had an opportunity to exert undue influence iii. (3) the alleged wrongdoer had a disposition to exert undue influence, and iv. (4) there was a result appearing to be the effect of the undue influenced (3) Judges and juries are inevitably affected by social context since the assessment is of the alleged wrongdoers conduct (4) What Influence is Undue? (a) Estate of Lakatosh, 656 A.2d 1378 (Pa. Super. 1994) (i) Material Facts: Roger befriended and took care of a woman (Rose) in her 70s for about five years. During this period he had her sign over power of attorney to him and executed a will to leave all but a small portion of her estate to Roger. Rogers second cousin drafted the will. Rose was living in squalor and filth during this time and a few years later revoked Rogers power of attorney (after he had already converted almost $130,000.00 to his own and others benefit). (ii) Issue: Did Roger put undue influence of Rose, making the will invalid? (iii) Holding: Yes. (iv) Reasoning: 1. First, the burden of proof may be shifted as to require the proponent to disprove undue influence a. To shift the burden the contestant must prove with clear and convincing evidence that (1) there was a confidential relationship, (2) that the person enjoying such relationship received the bulk of the estate, and (3) that the decedents intellect was weakened. i. These were easily met (note, the power of attorney alone is proof of a confidential relationship). 2. Roses intellect was weakened there was evidence that she had trouble remembering things and there was even an audio recording by the attorney that showed here intellect was diminished. a. She was easily distracted and had trouble concentrating on the issues of the will. 3. Since the burden was shifted, it was upon the proponent to demonstrate the absence of undue influence by clear and convincing evidence. a. Roger did not meet this burden. (v) Notes: 1. Undue influence cases are complicated in questions of burden of proof (or more specifically, burden of persuasion) as this case illustrates. a. The proponent may easily prove a wills validity by showing due execution

40

b. The person contesting the will then has the burden of proving undue influence either directly or indirectly by proving facts that would give rise to a presumption of undue influence. i. To trigger this presumption, the contestant must establish the existence of a confidential relationship between the influencer and the testator plus, in most jurisdictions, one or more substantial suspicious circumstances. 2. Confidential Relationship a. The law requires one or both parties to be other-regarding because of the nature of the relationship and the potential for abuse i. This term embraces three sometimes distinct and sometimes overlapping relationships fiduciary, reliant, or dominantsubservient. ii. Fiduciary: i.e., donor and hired professional, agent to principal, power of attorney, etc. iii. Reliant: Question of Fact - the contestant must establish that there was a relationship based on special trust and confidence that the donor was accustomed to being guided by the judgment or advice of the alleged wrongdoer or was justified in placing confidence in the belief that the wrongdoer would act in the interest of the donor. iv. Dominant-subservient: Question of Fact the contestant must find that the donor was subservient to the wrongdoers dominant influence. b. Suspicious circumstances: Restatement (Third) or Property Sec. 8.3 Non-Exhaustive list: i. The extent the donor was in a weakened condition ii. The extent the alleged wrongdoer participated in the preparation or procurement of the will or substitute iii. Did the donor have independent advice iv. Was the will or substitute made in secrecy or haste v. Donors attitude toward others has changed due to the relationship with the alleged wrongdoer vi. Decided discrepancy between old and new wills or substitutes vii. Whether there was a settled intent of previous wills or substitutes and then changed viii. Whether the disposition of the property would be considered unnatural, unjust, unfair, etc. i.e., disinherited a faithful and deserving family for little to no reason c. Burden Shifting: i. When the undue influence is triggered, the proponent must rebut the presumption under the theory that a person who benefits from a confidential relationship can take precautions to ensure that 41

proof exists that the transaction was fair and that his principal was fully informed, and he is in the best position after the transaction to explain and justify it. Cleary v. Cleary, 692 N.E.2d 955, 960 (Mass. 1998). ii. In order to overcome the presumption the grantee of the transaction must prove by clear, satisfactory, and convincing evidence that the grantee acted in good faith throughout the transaction and the grantor acted freely, intelligently, and voluntarily. Jackson v. Schrader, 676 N.W.2d 599 (Iowa 2003). 3. By statute, California invalidates any donative transfer to a care custodian of the donor a. The term is defined to include any protective, public, sectarian, mental health, or private assistance or advocacy agency or person providing health services or social services to elders or dependent adults. i. While lower courts said this only applies to professionals, but the State Supreme court said it also applied to preexisting personal friends 4. Remember: a. We look for i. Opportunity ii. Motive iii. Likelihood of success b. If this is met, then we shift the burden and the proponent must show that he did not exercise this coercion. (5) Lipper v. Weslow, 369 S.W.2d 698 (1963). (a) Material Facts: Testatrix disinherited her daughter-in-law and children of her daughter in law because she felt they were unfriendly and distant. Her will set aside a large description articulating why that was the case. Her son, an attorney, helped draft and execute the will and had misgivings over his late brother. (b) Issue: Was there undue influence? (c) Holding: No. (d) Reasoning: While there is suspicion, there must also be proof. Here, the women was of sound mind and body when she executed the will and had told several others her intention to disinherit her grandchildren and daughter in law and why before the execution of the will. (e) Notes: (i) In most states, unlike this previous case, confidential and suspicion would be enough to shift the burden but it was not the case here (ii) Some testators use their will to show their wrath at their heirs however, this may raise a will contest on principle. 1. This may also raise a will libel case

42

(iii) This is not so clear cut as strict coercion and is based on circumstantial evidence (the person with the evidence is most likely the alleged wrongdoer, who is not likely to give up the evidence). It is not emotional, since most wills are made with many emotions in mind. (iv) Some courts have held that being a child is not enough for a confidential relationship however, here, the child was also the attorney (v) This court does not apply the three prong test found in Lakatosh 1. Even so, Conf. relationship and suspected bulk of estate portions are met. The weakened mind is up to debate, since evidence goes both ways the court states that this is suspicious, but not enough you need more proof. (vi) The attorneys should have gotten independent counsel this is a serious conflict of interest 1. You cannot assist a will of which you are a beneficiary unless you are family. (vii) No contest clause in the will here is stupid it serves absolutely no purpose since the contestants were not going to receive anything under the will anyway. 1. Even so, where the contestants would receive something, these clauses are generally not enforced if there is a good-faith basis for the challenge since there is strong policy that these contests should be heard. c) Execution of Wills i) The four functions (1) Jane B. Baron, Gifts, Bargains, and Form, 64 Ind. L.J. 155 (1989) (a) In the aspect of giving, people are fundamentally unreliable and deceitful. This is in contrast with business bargains. We are suspected when we give, relied on when we trade. (2) James Lindgren, The Fall of Formalism, 55 Alb. L. Rev. 1009 (1992) (a) The law of wills has treated people as though they were stupid in the fear that they may improvidently give away their property at death. This has led to an unmatched formalism in American Law. (i) The idea that people are so weak, feeble, and subject to pressure that they need protection from themselves. Their spoken words are completely worthless, and their written statements are without meaning unless theyre witnessed by two people. (ii) In the law of contracts, people are intelligent, great weight is given to their serious statements and they dont need protection from themselves. All of their written statements have meaning without witnesses. (3) Ashbel G. Gulliver and Catherine J. Tilson, Classification of Gratuitous Transfers, 51 Yale L.J. 1 (1941) (a) If the purpose is to give owners power to determine his or her successors, courts should favor giving effect to an intentional exercise of that power. (i) The form should only be justified as implements for its accomplishment, and should be interpreted as such by courts, not be revered as ends in themselves, enthroning formality over frustrated intent. 43

1. First, the court needs to be convinced that the statements of the transferor were deliberately intended to effectuate a transfer. a. The formalities of transfer therefore generally require the performance of some ceremonial for the purpose of impressing the transferor with the significance of his statements and thus justifying the court in reach the conclusion, if the ceremonial is performed, that they were deliberately intended to be operative. i. This is termed the ritual function. 2. Secondly, the requirements of transfer may increase the reliability of the proof presented to the court. a. There are several difficulties with allowing extrinsic evidence, especially since the person whom best to discuss his or her intent is already dead. b. The existing requirements of transfer emphasize the purpose of supplying satisfactory evidence to the court i. This is termed the evidentiary function 3. Thirdly, some of the requirements of the statutes of will have the stated prophylactic purpose of safeguarding the testator, at the time of the execution of the will, against undue influence or other forms of imposition a. This is the protective function 4. Finally, the Wills Act formalities serve a channeling function a. The idea is that a persons wishes at death are easier to determine if those wishes are recorded in a standardized form. (4) In determining the proper level of formalities, we shouldnt ask whether this formality or that would serve the accepted purposes of formalities. We should ask instead whether it promotes the intent of the testator at an acceptable administrative cost: We shouldnt have formalities that serve a ritual function only. ii) Attested wills (1) Requirements of due execution (a) The most basic formalities for an attested will are three: (1) writing, (2) signature by the testator, and (3) attestation by witnesses. (i) These formalities vary in great detail from state to state because the sources of the formalities came from both the Statute of Frauds and the Wills Act established in England. (ii) Prior to the enactment of the SOF, personal property was transferable at death by either a written or oral will. The enactment of the SOF frauds required writing and three witnesses. The Wills Act enacted more strict requirements: the witnesses needed to be there at the same time, the document had to be signed at the foot or end, which has come be to known as a subscription. 1. These requirements of the Wills Act have led to wide litigation. a. The UPC usually adopts whichever is the least strict requirement from the two acts.

44

i.

Also, the UPC in a break from prior law, allows for notarization as an alternative to attestation by witnesses.

Comparison of Statutory Formalities for Formal Wills Statute of Frauds (Land) (1677) Writing Signature Attestation & subscription by 3 witnesses

Wills Act (1837) Writing Subscription Attestation & subscription by 2 witnesses

Uniform Probate Code (1990) Writing Signature Attestation & signature by 2 witnesses

Uniform Probate Code (1990 rev. 2008) Writing Signature Attestation & signature by 2 witnesses OR notarization

UPC 2-502 (1990, as amended 2008) (a) [Witnessed or Notarized Wills.] Except as otherwise provided in subsection (b) and in sections 2-503, 2-506, and 2-513, a will must be: (1) In writing; (2) Signed by the testator or in the testators name by some other individual in the testators conscious presence and by the testators direction; and (3) Either: (A) signed by at least two individuals, each of whom signed within a reasonable time after the individual witnessed either the signing of the will described in paragraph (2) or the testators acknowledgement of that signature or acknowledgement of the will; or (B) acknowledged by the testator before a notary public or other individual authorized by law to take acknowledgments. (b) [Holographic Wills.] A will that does not comply with subsection (a) is valid as a holographic will, whether or not witnessed, if the signature and material portions of the document are in the testators handwriting (c) [Extrinsic Evidence.+ Intent that a document constitute the testators will can be established by extrinsic evidence, including, for holographic wills, portions of the document that are not in the testators handwriting.

(b) Writing, Signature, and Attestation: Strict Compliance (i) Under traditional law, for a will to be admitted to probate it must be in strict compliance with the formal requirements of the applicable Wills Act. 1. In re Groffman, 2 All E.R. 108 (1969) [High Court of Justice, England] a. Material Facts: A wife alleged that her husband would not have had her share in his estate with her daughter from her first marriage and with one of the decedents daughters from his first marriage. Rather, she 45

should take under intestate statute, which would be the entire estate. The dispute centers around whether the will is invalid given both witnesses who viewed the will were nor present together when the decedent acknowledged his signature. b. Issue: Is the will valid even though the witnesses did not view his acknowledgement of the signature at the same time. c. Holding: The will, while valid on its face, cannot be entered into probate. d. Reasoning: The English Wills Act of 1837 requires that both witnesses be present during the signing or both witnesses see that the decedent acknowledged his signature simultaneously. e. Notes: 2. Stevens v. Casdorph, 508 S.E.2d 610 (1998) West Virginia Supreme Court of Appeals a. Material Facts: Decedent, an elderly man, was taken by his Nephew and his wife to a bank where they had him sign a will and had it notarized. The notary then took the will to two other employees to sign as witnesses. These two employees did not actually see the decedent sign the will and the decedent did not see them sign the document as witnesses. The decedents raised this action to void the will on the basis that it was not properly executed. b. Issue: Is the will valid and should proceed to probate or is it invalid and go through intestate. c. Holding: The will is invalid. d. Reasoning: i. W. Va. Code Sec. 41-1-3 (1997) requires that the signature be made or the will acknowledged by him in the presence of at least two competent witnesses, present at the same time; and such witnesses shall subscribe the will in the presence of the testator, and of each other. ii. This decedent and witnesses did not comply with the statute. iii. Mere intent to create a will is insufficient; it must comply with statute. iv. The narrow exception held in Wade requires that if a witness acknowledges his/her signature on a will in the physical presence of the other subscribing witness and the testator, then the will is properly witnessed within the terms of the code - This did not occur either. e. Notes: i. Dissent: The majority take a very technocratic approach to the law, slavishly worshiping form over substance

46

In doing so, they not only create a harsh and inequitable result wholly contrary to the indisputable intent of the decedent, but also a rule of law that is against the spirit and intent of our whole body of law relating to the making of wills. o There is no claim to fraud, incapacity, undue influence, nor any other allegation; only a formality that now goes against the decedents wishes of how to convey his property after his death. o In Wade, we held that a narrow, rigid construction of the statute should not be allowed to stand in the way of right and justice, or be permitted to defeat a testators disposition of his property. o The court should further the underlying policy to prevent substitution or fraud, not impede it. ii. Attestation Clause: - Although no state requires the use of an attestation clause, such a clause gives a rise to a presumption of due execution, and it is almost certainly professional malpractice not to include one. o With an attestation clause (which recites that the will was duly executed), the will may be admitted to probate even though the witnesses predecease the testator or cannot recall the events of execution. It also gives an attorney for the proponent heavy ammunition to use in cross-examination, and the will can often be admitted to probate on the presumption of due execution despite such testimony. See UPC Sec. 3-406(3) (1990, rev. 2008). iii. The Meaning of Presence in Will Execution - In England and some American states, presence means that the witnesses and testator can see the act of singing. This line of sight test, requires the testator to be able to see the witnesses sign if he were to look. o An exception is for blind testators, where the test is if he could have been able to see the witnesses sign from where the testator was standing or sitting if the testator had to power of sight. - In other American States, the conscious presence test, requires that the witness is in the presence of the testator if the testator though sight, hearing, or general consciousness of the events, comprehends that the witness is in the act of signing.

47

UPC Sec. 2-502(a) dispenses altogether with the requirement that the witnesses sign in the testators presence. iv. The Meaning of Signature in Will Execution - The law in all states, as well as UPC Sec. 5-502(a), requires the testator to sign the will. The purpose of the signature is to provide evidence of finality, distinguishing it from drafts and notes, and to provide evidence of genuineness. A signature by the testator with her full name at the end of the document will almost always satisfy the signature requirement. Problems arise when they take another form. o Signature by mark, with assistance, or by another: A mark, cross, abbreviation, or nickname can be sufficient. Helping a person sign is also valid if that person intended the document to be their will. Also, a computer generated signature is also valid, so long as it also goes along with the other requirements under the statute. o Order of singing: In general, the testator must sign before the witnesses attest, but if they all sign as part of a single (or continuous) transaction, the exact order of signing is not critical. Restatement (Third) of Property Sec. 3.1, cmt. m (1999). In an unfortunate case, In re Coling, 1 W.L.R. 1440 (1972), a will was considered invalid since one witness had to leave the hospital room during the middle of the signature, where the signature was finished when she was not there, and then returned and signed. o Subscription and addition after signature: A few states have adopted the English Wills act that the signature must be at the bottom of the will (Subscription). Usually, if extra items are added below the signature line, they will be allowed if it is done before the signature if it is added afterward then it is not binding. o Delayed Attestation. States are divergent on this issue. Some have a hard line (New York is 30 days and California requires it be done before the death). The UPC allows for a reasonable time and the official comment takes the position that this could be after the testators death. 48

Note: The UPC has a harmless error rule that allows wills to enter probate if there is clear and convincing evidence that it was intended to be a will This would include video wills or computer files with unique electronic signatures (some states do not allow this)

(2) Competency of witnesses (a) Estate of Morea, 645 N.Y.S.2d 1022 (1996) (i) Material Facts: Decedent had his will witnessed by three individuals, two of which were beneficiaries. The will was before an uncontested proceeding in which its validity was at question since New York law requires that two out of the three witnesses not be beneficiaries. (ii) Issue: Is the will valid even though two beneficiaries were also witnesses? (iii) Holding: Yes. (iv) Reasoning: One of the beneficiaries would take less under the will and thus the purpose of the will to prevent those who something to gain from being witnesses and falsely testifying has been satisfied. (v) Notes: 1. Interested witnesses and purging statutes: a. The purging statute allowed a will attested by an interested witness to be admitted to probate, but voided (purged) any bequest to the interested witness. After the purging statute, a will attested by an interested witness would be valid, but at the cost of purging the interested witness of his bequest. i. A majority of states have Purging Statutes. - The witness forfeits only the extra benefit afforded to the witness by the will. - A few states, such as Massachusetts, follow the English model and purge the witness of the entire devise ii. The purging statutes apply only to a witness who is necessary for the wills validity. If the will is witnessed by a sufficient number of disinterested witnesses, the interested witness is said to be supernumerary and is entitled to take his full bequest. 2. Interested witnesses and the UPC: a. A substantial minority, following UPC 2-505(b) (1990), do not require that the witnesses be disinterested. i. The will is valid even if witnessed by an interested party, and that the interested witness does not forfeit his bequest even if it is greater than that which he would have received under a prior will or by intestacy. - The rational is that Interest no longer disqualifies a person as a witness, nor does it invalidate or forfeit a gift under the will. Of 49

course, the purpose of this change is not to foster use of interested witnesses, and attorneys will continue to use disinterested witnesses in execution of wills. But the rare and innocent of use of a member of the testators family on a homedrawn will is not penalized.The requirement of disinterested witnesses has not succeeded in preventing fraud and undue influence; and in most cases of undue influence, the influencer is careful not to sign as a witness, but to procure disinterested witnesses. [UPC Sec. 2-505, cmt.] o California has developed a middle ground whereby bequest to a witness triggers a rebuttable presumption that the bequest was procured by duress, menace, fraud, or undue influence. (b) 1990 UPC Sec. 2-505 (3) Attestation clause; Self-proving affidavit; Recommended execution ceremony (a) Recommended Method of Executing a Will (i) A lawyer should not rely on the formalities required by the Wills Act in the clients home state. The clients will may be offered for probate in another state or the client may be domiciled elsewhere at death or may own real property in another state, or the will may exercise a power of appointment governed by the law of another state. 1. Thus, a prudent lawyer should draw a will and have it executed in such a manner that satisfies all the formal requirements in all states. (ii) This Procedure will make sure the instrument is valid in all states, no matter in which state the testator is domiciled at the date of execution or at death or where the property is located 1. If the will consists of more than one page, the pages are fastened together securely. The will specifies the exact number of pages of which it consists. 2. The lawyer confirms that the testator has read the will and understands its contents. 3. The lawyer, the testator, two (or three) disinterested witnesses, and a notary public are brought together in a room from which everyone else is excluded. (If the lawyer is a notary, an additional notary is not necessary.) The door to the room is closed. No one enters or leaves the room until the ceremony is finished. 4. The lawyer asks the testator the following three questions: a. Is this your will? b. Have you read it and do you understand it? c. Does it dispose of your property in accordance with your wishes? i. After each question the testator should answer Yes in a voice that be heard by the three witnesses and the notary. It is neither necessary nor customary for the witnesses to know the terms of the 50

5.

6.

7.

8. 9.

10.

will. If, however, the lawyer foresees a possible contest, added precautions might be taken at this time. See page 205. The lawyer asks the testator the following question: Do you request _____, ______, and ______ (the witnesses) to witness the signing of your will? The testator should answer Yes in a voice loud enough for the witnesses to hear. The witnesses should be standing or sitting so that all can see the testator sign. The testator signs on the margin of each page of the will. This is done for purposes of identification and to prevent subsequent substitution of pages. The testator then signs her name at the end of the will. One of the witnesses reads aloud the attestation clause, which attests that the foregoing things were done. a. For example: On the ____ day of _____, 20___, Wendy Brown declared to us, the undersigned, that the foregoing instrument was her last Will, and she requested us to act as witnesses to it and to her signature thereon. She then signed the Will in our presence, and in the presence of each other, hereunto subscribe our names as witnesses, and each of us declares that in his or her opinion this testator is of sound mind. Each witness then signs and writes his or her address next to the signature. A self-proving affidavit, typed at the end of the will, swearing before the notary public, who in turn signs an attaches the required seal. a. This allows the will to enter probate should one of the witnesses be unavailable, a self proving affidavit reciting that all the requirements of due execution have been complied with permits the will to be probated. While this is not necessary (except in Louisiana) it is still a very popular invention that saves time and headache should there be a contest. b. UPC Sec.2-504 (1990) authorizes two kinds of self-proving affidavits. i. (a) authorizes a combined attestation clause and self-proving affidavit, so the testator and the witnesses (and the notary) sign their names only once; this is called a one-step process affidavit. ii. (b) authorizes a separate self-proving affidavit to be affixed to a will already signed and attested. The affidavit must be signed by the testator and witnesses in front of the notary after the testator and witnesses have signed the will. It is recommended that the two-step process be done since it is enforced in more states than the singlestep method. c. UPC Sec. 3-406(1) (1990) provides that, if a will is self-proved, questions of due execution may not be contested unless there is evidence of fraud or forgery affecting the acknowledgement or affidavit. i. This does not limit contests for other grounds. Although not required, the lawyer should undertake a few precautionary post-execution measures. In a quiet moment after everyone has left, the 51

lawyer should review the will to check that all signatures are in the correct places and that each page is initialed or signed in the margin. If an error was made, it is easier to correct by redoing the execution ceremony than by litigation after death. a. It is also a good practice for the lawyer to write a short memo to the file noting that the firms usual execution procedures were followed. If the firm is retaining possession of the original (as we recommend in states where this is not expressly discouraged by local courts), the lawyer should place the original in the firms vault or safe deposit box and put copies, noted as such, in the client files. It is a good idea to send a booklet to the client containing a photocopy of the will (marked copy), a cover letter stating where the original will is stored, and a copy of any earlier letters describing the estate scheme, so that after death the family might find both a copy of the will and the address of the firm having custody of the original. (b) 1990 UPC Sec. 2-504 (c) 1990 UPC Sec. 2-506 (d) 1990 UPC Sec. 3-406 (4) Safeguarding wills (a) Safeguarding a Will (i) Depending on where you are, you should recommend that the original be kept with the firm in safekeeping. 1. Some states are not so thrilled with the idea since there is the option to leave it with the probate clerk (in some states and UPC Sec. 2-515) and also that it looks like solicitation. a. A firm should safe guard it unless the testator wishes to take it home: But make him or her aware of the various issues that arise with holding a will (i.e. if it gets lost, accidentally destroyed, written on, etc.) (b) 1990 UPC Sec. 2-515. iii) Notarized wills (1) As amended in 2008, UPC Sec. 2-502(a)(3), provides that a will is valid if it is signed by two witnesses or by a notary. (a) Lawrence W. Waggoner, The UPC Authorizes Notarized Wills, 34 ACTEC J. 83 (2008) (i) A notarized will is upheld under UPC harmless-error rule allowing a will to be executed without the clear and convincing standard of proof (ii) A notarized will serves to fulfill all the functions of will-execution formalities (evidentiary, cautionary (ceremonial), channeling, and protective. 1. A notarized will, like a holographic will, yields greater assurance of the identity of the maker of the document. a. With notaries, this is because one of the principal duties of a notary is to verify the identity of the person signing the document.

52

b. Note: A validly notarized will does not prevent a contest based on testamentary capacity. (iii) It would minimize confusion and change for error if all of the documents could be executed with the same formality of a notary signature. (iv) As amended the UPC treats the notary the same as two attesting witnesses. (v) Notes: 1. 2-502(a)(3)(b) is broader: it validates a will that has been acknowledged by the testator before a notary public or other individual authorized by law to take acknowledgements. In many states this would include a lawyer. a. In what state would this provision validate a will signed by a lawyer but not notarized and not signed by a second witness? (2) 1990 UPC Sec. 2-502(a)(3) iv) Holographic wills (1) In slightly over half of the states, primarily in the South and West, holographic wills are permitted. (See page 269). (a) A Holographic Will is a will written by the testators hand and signed by the testator; attesting witnesses are not required. (i) Note however, that the relative incompleteness of the performance of the functions of the regular statute of wills, and particularly the absence of any ritual value, may account for the fact that holographic wills in states recognizing them, *require+ the most precise compliance with formalities. (2) Notes: (a) Opponents of holographic wills argue that they are inartful and breed litigation, a conclusion in part based on reviews of appellate opinions. (i) However, in the most careful study to date of actual experiences with holographic wills in probate courts, only 4% resulted in objection or hearing of any kind. 1. Thus, holographic wills are indispensable to testators who are either unwilling or unable to commission a traditional will. The instinct to convey property to loved ones arguable should be applauded, not the subject of derision. (b) Conditional Wills: (i) Most cases presume the language of condition does not mean that the will is to be probated only if the stated event happens but is, instead, merely a statement of the inducement for execution of the will, which can be probated upon death from any cause. Eaton v. Brown, 193 U.S. 411 (1904) 1. This is not always that case, as was in Estate of Perez, 155 S.W.3d 599 (2004), where the court found a will conditional on a heart transplant was not to go to probate where the testator survived the surgery but died six months later. (c) Holographic wills are often written in extremis, when the testatroy is close to death, and sometimes under heartrending circumstances. 53

(i) In Estate of Harris, a man pinned behind his tractor scratched a will into the tractors fender leaving everything to his wife. The will was allowed to go to probate. (d) Holographic wills have been written on a myriad of items (cigarette carton, a tattoo, a bedroom wall, etc.). Whether they are allowed into probate depends on the facts. (3) 1990 UPC Sec. 2-502(b) d) Components of a Will i) Despite these formal requirements of transfer, however, it is possible for documents and acts lacking testamentary formalities to have the effect of determining who takes what property belonging to the testator. These doctrines (incorporation by reference and independent significance) permit extrinsic evidence to resolve the identity of persons or property. The following two are often confused with these doctrines. ii) Integration of wills (1) Under this doctrine, all papers present at the time of execution, intended to be part of the will, are integrated into the will. Restatement (Third) of Property: Wills and Other Donative Transfers Sec. 3.5 (1999). (a) Litigation arises when these papers are not properly fastened, no internal coherence, evidence that staple has been removed, font changes, etc. (i) i.e., Estate of Rigsby, a holographic wills second page was not allowed into probate since it was possible it was just a preliminary listing of the testators property before he drafted his will this was held even though it was folded along with the first page. (However, there was no signature on the second page and some room on the first in which to continue writing). iii) Republication by codicil (1) Under this doctrine, a will is treated as re-executed (republished) as of the date of the codicil: A will is treated as if it were executed when its most recent codicil was executed, whether or not the codicil expressly republishes the prior will, unless the effect of so treating it would be inconsistent with the testators intent. Restatement (Third) of Property: Wills and Other Donative Transfers Sec. 3.4 (1999). (a) For example, if a man writes a first will, then writes a second will and finally creates a codicil amending the first will, the second will is squeezed out and the first will is republished as the testators last will. (i) Accordingly, this doctrine is only applied where updating the will carries the testators intent, though sometimes courts have ignored this qualification. (ii) Note: Unlike incorporation by reference, this doctrine only applies to a prior validly executed will. Incorporation can apply to a will language or instruments that have never been validly executed. 1. In Estate of Nielson, the court found that a testator who crossed out sections of his will and written in additions, had created a holographic codicil which republished the handwritten will, as modified. iv) Incorporation by reference (1) Clark v. Greenhalge, 582 N.E.2d 949 (1991) 54

(a) Material Facts: Testator wrote in a notebook personal items she wished to be given to certain individuals at the time of her death. One item in particular, a painting, was to be given to her friend. The executor denied this wish, kept the painting for himself, but in all other respects followed the directions of the notebook and will. (b) Issue: Is the notebook part of the will by the doctrine of incorporation by reference? (c) Holding: Yes (d) Reasoning: (i) A properly executed will may incorporate by reference into its provisions any document or paper not so executed and witnessed, whether the paper referred to be in the form ofa mere list or memorandum, if it was in existence at the time of the execution of the will, and is identified by clear and satisfactory proof as the paper referred to therein. (ii) The cardinal rule is the interpretation of wills, to which all other rules must bend, is that the intention of the testator shall prevail, provided it is consistent with the rules of law. 1. The intent of the testator is ascertained through consideration of the language which the testator has used to express his testamentary designs, as well as the circumstances existing at the time of the execution of the will. The circumstances existing at the time of the execution of a codicil to a will are equally relevant, because the codicil serves to ratify the language in the will which has not been altered or affected by the terms of the codicil. (iii) Thus, the testator here made it clear in her will that she wished to be able to alter or amend the bequests of tangible personal property in her will, without having to amend formally the will. The statements in the notebook unquestionably reflect Helen Nesmiths exercise of her retained right to restructure the distribution of her tangible personal property upon her death. That the notebook is not entitled memorandum is of no consequence, since its intended purpose is consistent with that of a memorandum under Article Fifth of her will. 1. To narrowly construe Article Fifth would undermine our long-standing policy of interpreting wills in a manner which best carries out the known wishes of the testatrix. (e) Notes: (i) Simon v. Grayson, 102 P.2d 1081 (Cal. 1940). 1. A will made reference to a letter which would direct the court to pay an amount to a person named in that same letter. A letter was found, which did just that, but it was not dated as the will had said it would be. The court found that the date was of no consequence and incorporated it into the will since it still was created prior to the date of a codicil to the will. (ii) The doctrine is not recognized in Connecticut, Louisiana, or New York. However, New York has stretched the doctrines of republication by codicil and integration to carry out the testators intent. 55

1. To the latter: If the testator refers in his will to a separate memorandum disposing of his tangible personal property, and if such memorandum is attached to the other pages of his will and was present at execution, such memorandum is entitled to probated under the doctrine of integration. (iii) If the testator refers to a separate property that instructs her executor to sell tangible property and give the proceeds to named individuals, this instruction is enforceable under UPC Sec. 2-513. Last Will and Testatment of Moor, 879 A.2d 648 (Del. Ch. 2005). 1. This same court found that there is no limit on the value of personal property that may be passed by separate writing. 2. In California, no single item can have a value in excess of $5,000.00 and the total cannot exceed $25,000.00. (2) Johnson v. Johnson, 279 P.2d 928 (1954) (a) Material Facts: Testator created a type written will that was not signed or properly witnessed. After the typed portion, he hand wrote an additional section and signed below it. (b) Issue: Is this instrument one complete, integrated writing, partly typed and partly handwritten; or is it an unexecuted nonholographic will to which is appended a valid holographic codicil? (c) Holding: It is a valid holographic codicil which incorporates the prior will by reference and republished and validated the prior will as of the date of the codicil, thus giving effect to the intention of the testator. (d) Reasoning: (i) The general principle of law is that a codicil validly executed operates as republication of the will no matter what defects may have existed in the execution of the earlier document, that the instruments are incorporated as one, and that a proper execution of the codicil extends also to the will. (e) Notes: (i) Concurring: It was not the intent of our lawmakers, in enacting these statutes, if substantially complied with, to ever allow a miscarriage of justice by a wrongful disposition of the testators property contrary to his intent. (ii) Dissent: This will was one complete will unattested and therefore not admissible to probate (iii) In Berry v. Trible, a comparable case, the testator added additions to a typed will and initialed each change. The court held this would not enter probate because the handwriting and typed portions were interwoven, both physically and in sequence of thought. (3) 1999 UPC Sec. 2-510: Incorporation by Reference (a) A writing in existence when a will is executed may be incorporated by reference if the language of the will manifests this intent and describes the writing sufficiently to permit its identification.

56

(4) 1999 UPC Sec. 2-513: Separate Writing Identifying Devise of Certain Types of Tangible Personal Property (a) Whether or not the provisions relating to holographic wills apply, a will may refer to a written statement or list to dispose of items of tangible personal property not otherwise specifically disposed of by the will, other than money. To be admissible under this section as evidence of the intended disposition, the writing must be signed by the testator and must describe the items and the devisees with reasonable certainty. The writing may be referred to as one to be in existence at the time of the testators death; it may be prepared before or after the execution of the will; it may be altered by the testator after its preparation; and it may be a writing that has no significance apart from its effect on the dispositions made by the will. v) Acts of independent significance (1) If the beneficiary or property designations are identified by acts or events that have a lifetime motive and significance apart from their effect on the will, the gift will be upheld under this doctrine. (Also called the doctrine of nontestamentary acts). (a) This is true even if the phrasing of the will leaves it in the testators power to alter the beneficiaries or the property by a nontestamentary act. (2) 1990 UPC Sec. 2-512. Events of Independent Significance (a) A will may dispose of property by reference to acts and events that have significance apart from their effect upon the dispositions made by the will, whether they occur before or after the execution of the will or before or after the testators death. The execution or revocation of another individuals will is such an event. (3) See problems on page 324. e) Revocation of Wills i) Revocation by operation of law: change in family circumstances (1) In all but a few states, statutes provide that a divorce revokes any provision in the decedents will for the divorced spouse. In the remaining states, revocation occurs only if divorce is accompanied by a property settlement. (a) These statutes ordinarily do not apply to nonprobate transfers. (2) UPC Sec. 2-804 (1990) applies to nonprobate transfers as well as to wills. The term governing instrument in Sec. 2-804 is defined in UPC Sec. 1-201(18) to mean deed, will, trust, insurance or annuity policy, account with a payable-on-death designation, pension plan, or similar nonprobate donative transfer. (3) 1990 UPC Sec. 2-804. Revocation of Probate and Nonprobate Transfers by Divorce; No Revocation by Other Changes of Circumstances (a) (b) [Revocation Upon Divorce]. Except as provided by the express terms of a governing instrument, a court order, or a contract relating to the division of the marital estate made between the divorced individuals before or after the marriage, divorce, or annulment, the divorce or annulment of a marriage: (i) (1) revokes any revocable (i) disposition or appointment of property made by a divorced individual to his [or her] former spouse in a governing instrument and any disposition or appointment created by law or in a governing instrument to a 57

relative of the divorced individuals former spouse, (ii) provision in a governing instrument conferring a general or nongeneral power of appointment on the divorced individuals former spouse, and (iii) nomination in a governing instrument, nominating a divorced individuals former spouse or a relative of the divorced individuals former spouse to serve in any fiduciary or representative capacity, including a personal representative, executor, trustee, conservator, agent, or guardian; and (ii) (2) severs the interests of the former spouses in property held by them at the time of the divorce or annulment as joint tenants with the right of survivorship [or as community property with the right of survivorship], transforming the interests of the former spouses into equal tenancies in common (b) (d)[Effect of Revocation]. Provisions of a governing instrument are given effect as if the former spouse and relatives of the former spouse disclaimed all provisions revoked by this section or, in the case of a revoked nomination in a fiduciary or representative capacity, as if the former spouse and relatives of the former spouse died immediately before the divorce or annulment. (c) (f)[No Revocation for Other Change of Circumstances.] No change in circumstances other than as described in this section or in section 2-803 effects a revocation. (4) See problem on page 306. ii) Revocation by writing or physical act (1) A will is an ambulatory document, which means that it is subject to modification or revocation by the testator during her lifetime. (2) All states permit revocation of a will in one of two ways: (a) By a subsequent writing executed with testamentary formalities (UPC Sec. 1-201(57) see page 286 n. 31) (b) By a physical act such as destroying, obliterating, or burning the will. (3) 1990 UPC Sec. 2-507. Revocation by Writing or by Act (a) A will or any part thereof is revoked: (i) (1) by executing a subsequent will that revokes the previous will or part expressly or by inconsistency; or (ii) (2) by performing a revocatory act on the will, if the testator performed the act with intent and for the purpose of revoking the will or part if another individual performed the act in the testators conscious presence and by the testators direction. For purposes of this paragraph, revocatory act on the will includes burning, tearing, canceling, obliterating, or destroying the will or any part of it. A burning, tearing, or cancelation is a revocatory act on the will, whether or not the burn, tear, or cancellation touched any of the words on the will. (4) Revocation by Inconsistency (a) If a subsequent will does not make a complete disposition of the testators estate, it is not presumed to revoke the prior will but is viewed as a codicil, and the property not disposed of under the codicil is disposed of in accordance with the prior will.

58

(i) A codicil is a testamentary instrument that supplements, rather than replaces, an earlier will; the codicil supersedes the will to the extent of inconsistency between them. See UPC Sec. 2-507(b)-(d) (1990). (ii) The older view is that, in the absence of a revocation clause, a general residuary clause in a later will was not enough to revoke specific bequests in an earlier will because the earlier individual bequests and the later residuary clause were not literally inconsistent. (5) Harrison v. Bird, 621 So. 2d 972 (1993). (a) Material Facts: Testatrix asked her attorney to revoke her will. He had the will destroyed and sent to the client, notifying her that she was now without a will. She died a short time after and never created a new will. At the time of her death, they found the envelope that contained the ripped will, but could not find the will itself. The original will beneficiary challenged on the basis that the original will was not properly revoked. (b) Issue: Is this will properly revoked? (c) Holding: Yes. (d) Reasoning: (i) If the evidence shows that the testator was in possession of the will before her death, but the will is not found among her personal effects after her death, a presumption arises that she destroyed the will. (ii) If she destroys the copy of the will in her possession, a presumption arises that she has revoked her will and all duplicates, even though a duplicate exists that is not in her possession. 1. These presumptions are rebuttable but the burden of rebutting the presumptions is on the proponent of the will. (e) Notes: (i) Some courts require clear and convincing evidence for rebuttal of these presumptions (Estate of Pallister) while others only require a preponderance of the evidence (Estate of Turner). See page 289, note 2. (ii) Probate of Lost Wills 1. In the absence of a statute to the contrary, a will that is lost, destroyed without the consent of the testator, or destroyed with the consent of the testator but not in compliance with the revocation statute can be admitted into probate if its contests are proved. a. A lost will can be proved by a copy in the lawyer-drafters office or by other clear and convincing evidence. 2. In a few states, statutes prohibit the probate of lost or destroyed will unless the will was in existence at the testators death (and destroyed thereafter) or was fraudulently destroyed during the testators life. a. However, courts have chosen to give effect to the will revocation statutes and have gutted the proof statutes by holding either that will not lawfully revoked continues in legal existence until the testators 59

death (existence meaning legal existence) or that a will destroyed by a method not permitted by the will revocation statute has been fraudulently destroyed. (6) Thompson v. Royall, 175 S.E. 748 (1934). (a) Material Facts: A will was intended to be revoked, but the testatrix decided to hold onto it in case she wanted to make a new will later on. A judge wrote upon the back cover of the will that it was null and void. (b) Issue: Is this will void or valid? (c) Holding: It is valid. (d) Reasoning: (i) Revocation requires (1) the intent to revoke and (2) the doing of one of the methods prescribed by statute 1. While the intent to revoke was done, none of the actual acts prescribed by statute were fulfilled. 2. There was nothing on the face of the will to show sufficient defacement to bring it within the meaning of the prescribed methods of revocation. 3. If written words are used to revoke, they must be so placed as to physically affect the written portion of the will, not merely on blank parts of the paper on which the will is written. a. It must physically obliterate, mutilate, deface, or cancel any written parts of the will. (e) Notes: (i) UPC Sec. 2-507(a)(2), allows for cancellation regardless of whether the cancellation touches any of the words of the will. 1. The will itself is a different problem, since in this case the words were written on the manuscript cover and not the will itself. a. Even so, under the harmless error rule of Sec. 2-503, if there was clear and convincing evidence that the writing on the manuscript cover was intended to revoke the instruments and surely there was then the attempted revocation would be effective. (ii) Partial Revocation by Physical Acts: 1. Although UPC Sec. 2-507 and the statutes of many states allow for partial revocation by physical act, in several states this is not the case. It can only be partially revoked by another instrument, such as a codicil. The reasons are: a. Canceling a gift to one person necessarily results in someone else taking the gift, and this new gift can be made only by an attested writing b. Permitting partial revocation by physical acts offers opportunity for fraud. 2. If partial revocation by physical act is not recognized, the will must be admitted to probate in the form in which it was originally executed if the original language can be ascertained. 60

3. Some courts have said that this is not allowed where it would result in a substantial enhancement of another bequest. 4. A few cases have held that the testator may completely remove a devise but may not rearrange the shares in a single dives to increase the other devisees gift. 5. See problems on page 294. 6. 1990 UPC Sec. 2-507 iii) Dependent relative revocation; revival. (1) (DDR): If the testator purports to revoke his will upon a mistaken assumption of law or fact, the revocation is ineffective if the testator would not have revoked his will had he known the truth. (a) The testator does not have true revocatory intent if the revocation was based on a mistaken belief. (b) The doctrine is one of presumptive intent, not actual intent. (2) LaCroix v. Senecal, 99 A.2d 115 (1953) (a) Material Facts: Testatrix created a will which gave a large portion to her cousin and her friend. However, thinking there may be an issue with how she drafted the will (using her cousins nick-name) she created a codicil to change the name to the proper one. The codicil changed nothing else. Upon probate of the codicil it came to the attention of the court that one of the witnesses of the codicil was the husband of one of the beneficiaries. Under statute, he could not be a witness and a beneficiary so the codicil is invalid. (b) Issue: Should the original will be put into probate, or should the entire section of the original will be subject to ?intestacy? (c) Holding: The original will is valid. (d) Reasoning: (i) DDR simply stated: Where the intention to revoke is conditional and where the condition is not fulfilled, the revocation is not effective. (ii) It is a rule of presumed intention, applicable to partial as well as total revocation. (iii) Here, the testatrixs clear intent was to clear up any possible uncertainty to who her cousin was. It was in no way to make any major changes to her estate plans. 1. When it became clear that the gift to her friend became void, the conditional intention of the testatrix to revoke the will was rendered inoperative, and the gift to her friend under the will continued in effect. 2. When a testator repeats the same dispositive plan in a new will, revocation of the old one by the new is deemed inseparably related to and dependent upon the legal effectiveness of the new. (e) Notes: (i) While DDR worked well in this state, a few have so limited that the same facts or similar facts in one of these states would reach an opposite result. See Rosoff v. Harding, page 297 in n. 1. 61

1. However, under traditional DDR the will in Rosoff would have been presumptively ineffective to the extent that its execution was based on the mistaken assumption that it validly exercised the power of appointment. a. Restatement (Third) of Property: Wills and other Donative Transfers Sec. 4.3, cmt. e (1999), the doctrine of DDR may be applied when a later will that expressly revoked an earlier will contains one or more dispositive provisions that fail under applicable law. The revocation of the earlier will is presumptively ineffective to the extent necessary to give effect to the dispositive provision in the earlier will that the failed dispositive provision in the later will replaced. (ii) See problems 2 - 3 on page 298 (very useful example used in class look at corresponding class notes for the answer). (iii) Note: With rare exception, court have limited DDR by stating it only applies (1) where there is an alternative plan of disposition that fails, or (2) where the mistake is recited in the terms of the revoking instrument or, possibly, is established by clear and convincing evidence. 1. The alternative plan of disposition is usually in the form of another will, either duly or defectively executed. By so limiting the doctrine, the kind of extrinsic evidence that can be considered is narrowed. (iv) See notes on page 399 for parameters of how the doctrine has been used in special circumstances. (3) Estate of Alburn, 118 N.W.2d 919 (1963). (a) Material Facts: Testatrix executed one will in one state. Then, a few years later, executed another will in another state with different beneficiaries. Some time later, she move yet again, and disposed of her second will. The beneficiaries of both wills argue that each is valid. (b) Issue: Which will stands? (c) Holding: The first will. (d) Reasoning: (i) DDR is applied to the unusual situation in which a testator revokes a later will under the mistaken belief that by doing so he is reinstating a prior will. (ii) We are under the presumption that the testatrix did not want to die intestate, since neither will acknowledged any payments to next of kin (except to her brother [which was a small amount]). Testatrix must have known that the original of the first will was still in possession of her attorney and the only impediment was her new will. She also knew that her new will was destroyed and made no attempt to create a new one. 1. These presumptions and evidence taken together are evidence that testatrix destroyed her second will on the mistaken belief that her first would stand. Thus DDR applies. (e) Notes: (i) Revival 62

1. This doctrine falls on the following facts: a. Testatrix executes a will, and then subsequently executes a second will, revoking the first will. Later, testatrix revokes will two, is will one valid? i. A few states find that the first will is not revoked until the second will speaks at death. Thus, writing a second will doesnt really revoke the first will until the testatrixs death. ii. A large majority holds that will one is indeed revoked at the time of execution of the second will. However, they split into two groups: - A majority hold that upon revocation of will two, will number 1 is revived if the testator so intends. o The testators intent is taken from circumstances surrounding revocation of the second will or from contemporaneous or subsequent oral declarations from the testator that the first will is to take effect. - A minority of states take the view that the original will cannot be revived unless re-executed with testamentary formalities or republished by being referred to in a later duly executed testamentary writing. iii. Nearly half of the states have adopted a statute based on UPC Sec. 2-509 (1990 or 1969 versions). b. See problems on page 305 for practice. (4) 1990 UPC Sec. 2-509. Revival of Revoked Will (a) if a subsequent will that wholly revoked a previous will is thereafter revoked by a revocatory act under section 2-507(a)(2), the previous will remains revoked unless it is revived. The previous will is revived if it is evident from the circumstances of the revocation of the subsequent will or from the testators contemporary or subsequent declarations that the testator intended the previous will to take effect as executed. (b) If a subsequent will that partly revoked a previous will is thereafter revoked by a revocatory act under Section 2-507(a)(2), a revoked part of the previous will is revived unless it is evident from the circumstances of the revocation of the subsequent will or from the testators contemporary or subsequent declaration that the testator did not intend the revoked part to take effect as executed. (c) If a subsequent will that revoked a previous will in whole or in part is thereafter revoked by another, later, will, the previous will remains revoked in whole or in part, unless it or its revoked part is revived. The previous will or its revoked part is revived to the extent it appears from the terms of the later will that the testator intended the previous will to take effect. 5) PLANNING FOR INCAPACITY a) Durable Powers of Attorney.

63

i)

An ordinary power of attorney creates an agency relationship whereby the agent, called an attorney-in-fact (though need not be, and is often not an attorney), is given a written authorization to act on behalf of the principal. (1) This is limited to the traditional rule of agency law that the agents authority ends when the principal becomes incapacitated. ii) A Durable power of attorney, on the other hand, continues throughout the incapacity of the principal until the principal dies. (1) UPC Sec. 5-501 to 5-505 (1990) and by statutes in all states. (a) The principal, if competent, can end this at any time and the agent is subject to agency fiduciary duties of loyalty, care, and obedience. (i) These must be created by written instruments and in some states witnessed and notarized. (2) A durable power ends with the principals death (a) This does not a means to avoid probate. In contrast, a trust can continue past the settlors death, transferring property without probate. (3) If the agent dies, the power terminates unless the principal names a successor agent. (a) If a trustee dies, a successor trustee is appointed by the court (4) Unlike trustees, the power of attorney does not own the property, and traditional agency law strictly construes express powers and sparingly implies other powers. (5) Third parties will general deal with trustees, since banks and other institutions are uncertain of an attorney-in-facts authority (though they can be compelled to do so). (6) Nonetheless, durable powers of attorney are useful for persons seeking a way of dealing with incompetency without creating a trust (but trusts are more flexible and satisfactory to most clients) (a) Durable powers are extremely popular to those with modest means (i) However, it is subject to great abuse and leads to a lot of litigation. iii) In re Estate of Kerrelmeyer, 895 A.2d 207 (2006) (1) Material Facts: Testator appointed his wife as durable attorney in fact. When he became incapacitated, she transferred the larger portion of his property (a home) to herself through a revocable living trust. When the testator died, his will had specified that other things would have been done with the property, specifically for the benefit of his children. (2) Issue: Did the wife overstep her power in transferring the property to herself through a trust? (3) Holding: She can create a trust. Whether she breached her duty is an issue for remand. (4) Reasoning: (a) We look at the written instrument as a whole to determine the principals intent, not a strict analysis. The written document allowed for transfers of trust instruments and property. It also allows to do so for estate planning objectives. The issue of whether she overstepped her bounds is remanded. (b) The fact that the trust was created by a power of attorney does not void its legitimacy. 64

(i) Under some circumstances a durable power of attorney gives the agent power to create a trust on behalf of the property owner. Restatement Sec. 11(5). (c) We are also not persuaded that this trust is an invalid usurpation of the will. The decedent when making the power of attorney knew that it could change his will given certain circumstances. (5) Notes: (a) At remand, the lower court upheld the trust since it was in keeping with the intention of the will that she have power over the home anyway. The overarching goal was to provide for his wife. (b) The majority view is that an agent acting under a power of attorney cannot make, amend, or revoke the principals will, but the agent may create, modify, or revoke a trust if the power to do so is expressly granted in the power of attorney instrument. (i) Some states allow a guardian to make changes to a will only if they pass a substituted judgement standard that show that the testator would have done the same if he could. UPC Sec. 5-411. (1990) (c) The very lack of oversight and ease of use that make powers of attorney so attractive in planning for incapacity make them easy to abuse. (d) Whether an agent under power of attorney can make gifts raises particular difficulites. Many courts require the power to make gifts to be explicit, and they subject the agents exercise of such a power in favor of the agent to close fiduciary scrutiny, requiring the gift clearly to be in the principals best interests. (e) The uniform Law Commission promulagated a new Uniform Power of Attorney Act, replacing the term with agent. Also, it is durable unless specifically stated otherwise. Third parties may report that the principal is being subject to physical or financial abuse. If the instrument clearly allows the agent to do something, the agent can do it. b) Advance Health-Care Directives i) Living Wills, Health Care Proxies, and Hybrids (1) Instrumental Directives (a) Such as a living will, or medial directive, which specify whether generally or by way of hypothetical examples how one wants to be treated in end-of-life situations or in the event of incompetence (2) Proxy directives (a) Such as health care proxy or durable power of attorney for health care (i) This designates an agent to make health care decisions for the patient (the power does not expire with the incompetency of the principal) (3) Hybrid or combined directives ii) Rebecca Dresser, Precommitment: A Misguided Strategy for Securing Death with Dignity, 81 Tex. L. Rev. 1823 (2003) (1) While Advance directives have been around an enforced for quite some time, only 25% of people have them (a) Most supply only a general indication of how they would like to be treated. 65

(b) To remedy, health professionals now elicit more detailed documents with several preferences and specific information about treatment preferences. (i) The well-known is the medical directive 1. It sets out six hypothetical clinical situations involving different physical and mental impairments and six categories of life-sustaining interventions. a. Even so, these are inadequate. Most people cannot predict all of the medical issues that the future might bring. Also, a persons preferences may change over time. Moreover, people pick poorly in situations they have never experienced. (2) Notes: (a) Every state has statutes providing for advance direction of health care, but these vary in the particulars and often require specific forms (b) There are also very difficult decisions for children making end-of-life decisions for parents. c) Disposition of the Body & Organ Donation. i) Postmortem Remains (1) More recently, in some states codified by mortal remains statutes, gives a person the power to have his wished carried out as to how he wants his remains dealt with. This is a hope, though, rather than an assurance. Note: If a person dies by violence or in suspicious circumstances, statutes in all states require an autopsy regardless of the wishes of the deceased person. ii) Organ Donation (1) To increase the quantity of cadaver organs for transplantation, all states have enacted some form of the Uniform Anatomical Gift Act. (a) This act permits a person to give her body to any hospital, physician, medical school, or body bank for research or transplantation. It also allows for a gift of a body, or parts thereof, to any specified individual for therapy or transplantation. (i) This act has done little to effect on the shortage of needed organs. 1. This is because of (a) the difficulty of imagining dying and others using their organs; (b) the fear that a physician would hasten death to get the organs; (c) unwillingness to be cut open after death and; (d) simply not thinking about the matter. a. Several arguments have been raised to deal with these issues including an organ market, tax benefits, hospital Medicare eligibility to consult dying patients of this option, mandated choice or even a presumption that the deceased wants to have the organs donated. (2) Note: Elder Law (a) Elder law deals with a wide range of issues facing the elderly, including health care, asset preservation, Medicaid eligibility, retirement, competency and guardianship, discrimination, elder abuse, and housing and institutionalization. 6) WILLS: CORRECTING MISTAKES AND OTHER CONSTRUCTION PROBLEMS a) Correcting Mistakes in the Execution of Wills. 66

i) See In re Groffman and Stevens v. Casdorph. ii) Curing Defects in the Execution of Attested Wills (1) Excusing Execution Defects by Ad Hoc Exception (a) To avoid the harsh result of voided a will on any mistake on the execution of a will, some courts have occasionally excused or corrected an obvious execution defect, while others have taken the position that there can be no relief from the rule of strict compliance. (b) In re Pavlinkos Estate, 148 A.2d 528 (1959) Pennsylvania. (i) Material Facts: A foreign speaking husband and wife created wills leaving the property to each other, and others if they predeceased. They accidentally signed eachothers wills. They died sometime after. One of the beneficiaries of the wills brought it to probate but was denied. (ii) Issue: Are the wills valid? (iii) Holding: No. (iv) Reasoning: The wills act is very clear that the will must be signed by the testator. Since they were not signed by the correct person, the wills are not valid. (v) Notes: 1. Strong dissent arguing for the clear intent of the parties, and their sympathetic situation. (c) In re Snide, 418 N.E.2d 656 (1981) New York (i) Material Facts: Husband and wife executed the wills meant for the other at the same execution ceremony. There were no other issues as to the execution of the wills and they are identical except for the names of beneficiaries and donors. The husband died sometime later and the wife offered his will to probate. (ii) Issue: Is the husbands will valid? (iii) Holding: The will is valid. (iv) Reasoning: 1. This is an issue of genuine mistake. The opponent of the will states that the husband did not properly intent to execute the document he actually signed, which in some states has been the basis for denial of probate. 2. It would indeed by ironic if not perverse to state that because what has occurred is so obvious, and what was intended so clear, we must act to nullify rather than sustain this testamentary scheme. a. This is a very unusual case; not only did the two instruments constitute reciprocal elements of unified testamentary plan, they both were executed with statutory formality, including the same attesting witnesses, at a contemporaneous execution ceremony. Thus, there is absolutely no danger of fraud, and to not read the wills together would be an unnecessary expansion of formality without any benefit. (v) Notes: 1. These two courts reached different results with the same facts. Why? 67

2. Two different modes of correcting switched wills errors a. Probate the will that the decedent intended to sign but did not b. Probate the will the decedent actually signed and then to reform its terms to make sense. 3. Guardians ad litem a. When the interests of minor children or unborn heirs are at stake, courts usually appoint a guardian ad litem, a person assigned to represent the interests of the minor or unborn heir in the litigation i. This is the person who challenged the will in the Snide case of behalf of the decedents minor child. - Shouldnt this guardian take into consideration the entire familys interests, and not just the minor child? This is an issue of ethics iii) Curative Doctrines: Substantial Compliance and Harmless Error (1) Substantial Compliance (a) The court may deem a defectively executed will as being in accord with statutory formalities if the defective execution nonetheless fulfills the purposes of those formalities. (2) Harmless Error Rule (a.k.a. Dispensing Power) (a) The court may excuse noncompliance with statutory formalities if there is clear and convincing evidence that the decedent intended the document to be his will. See 1990 UPC Sec. 2-503. (3) In re Will of Ranney, 589 A.2d 1339 (1991) (a) Material Facts: The attorneys prepared a two-step self-proving will. The testator, two witnesses and a notary each signed an attached affidavit, stating they signed the will, but did not in fact sign the actual will. (b) Issue: Is the will valid? (c) Holding: Yes, it substantially complies with law and should be allowed to probate. (d) Reasoning: (i) While New Jersey statute makes it clear that a self-proving affidavit is to be used solely in conjunction with a duly-executed will, substantial compliance is a functional rule designed to cure the inequity caused by the harsh and relentless formalism of the law of wills. (ii) The underlying rationale is the finding of a formal defect should lead not to automatic invalidity, but to a further inquiry: does the noncomplying document express the decedents testamentary intent, and does its form sufficiently approximate Wills Act formality to enable the court to conclude that it serves the purposes of the Wills Act. (iii) The purpose of formalities is to ensure that the document reflects the uncoerced intent of the testator. Rigid insistence on literal compliance often frustrates these purposes. Thus, compliance with statutory formalities is

68

important not because of the inherent value that those formalities possess, but because of the purposes they serve. (iv) While we allow the will to probate based on substantial compliance and clear and convincing evidence that it does indeed substantially comply with statute, we do not allow the affidavit to be both a validation of the will and self-proving (e) Notes: (i) In Estate of Fordonski, 678 N.W.2d 413 (2004), witnesses did not sign the will but did sign a two-step self-proving affidavit that was affixed to the will. However, the applicable Wills Act did not specify where to sign and the court allowed it into probate. Did they use the substantial compliance doctrine, even though they did not pronounce they had done so? (ii) We need to focus on the mistake 1. What formality was not done, and determine whether what was done substantially complies with that formalitiy. (4) In re Estate of Hall, 51 P.3d 1134 (2002) Montana [legislature has adopted new UPC] (a) Material Facts: Testator created an original will. Later on he met with his attorney to create a joint will. They wanted to make sure that it would be valid so they signed it and the attorney notarized it. Once home, the testator ripped up his original will. The wife brought the will to probate after testators death, but his daughter from another marriage challenged the will on the grounds it wasnt properly executed (they needed to witnesses). (b) Issue: Is the new will valid? (c) Holding: Yes. (d) Reasoning: (i) The harmless error rule only requires that there be clear and convincing evidence that the testator intended the document to be his will. Here, the testimony and facts surrounding the case clearly show this intention (specifically destroying the original will and signing the new will with the lawyer notarizing it). (e) Notes: (i) The creator, Prof. ?Langbaum?, of both doctrines favors harmless error over substantial compliance. Courts read into their substantial compliance doctrine a near-miss standard, ignoring the central issue of whether the testators conduct evidenced testamentary intent. (ii) A version of UPC 2-503 has been adopted in California, Colorado, Hawaii, Michigan, Montana, New Jersey, South Dakota, Utah, and Virginia. 1. Note, however the various limitations by each state (see page 263 note 19.) (iii) If in almost every case attestation defects are going to be excused, why not use a rule rather than a litigation-breeding standard? 1. Pennsylvania does not have an attestation requirement for formal wills, and there has been no evidence that fraud has run wild.

69

(iv) In the states that have adopted the harmless error rule, the effective minimum requirement for admitting a document to probate as a will has been reduced to little more than the intent that the document be a will. 1. Thus, litigation has shifted to the intent of the testator rather than focus on formalities in wills contests. a. More casual documents may have to be examined to determine whether they were intended to be a will, a concern raised by academics. i. Unfortunately, testatmentary intent has not been well defined or analyzed. There are many possible components to testamentary intent - Intent that: a document convey no present interest, that it be a will, that it not be a will substitute, to execute a document, that it be final unless later revoked, that after death certain beneficiaries receive certain property, etc. (5) In re Estate of Kuralt, 15 P.3d 931 (2000) [Montana] (a) Material Facts: Testator had basically two families. He wrote a holographic will, giving certain property to his girlfriend and a formal will written out to his wife and family. Then, on his sick bed (which would become his death bed) he wrote a letter containing 17 thousand dollars to his girlfriend saying that the wanted to make sure she inherited the property they both had shared. He died two weeks later and the girlfriend brought both the holographic will and the letter to probate. (b) Issue: What happens here? (c) Holding: The girlfriend gets the property they shared and said he wanted to give to her in the letter. The letter is a holographic codicil. (d) Reasoning: (i) The letter expressed the testators intent to effect a posthumous transfer to his girlfriend. Also concentrating on the last 20-months of his life, they found that he intended to transfer certain property that they shared to her. 1. The evidence included, in extremis [which is not really a good argument], he wanted to keep the relationship a secret so he didnt consult an attorney, and the fact that he used the word inherit in the letter. (e) Notes: (i) While there was great evidence of what he wanted to do and intended to do, there is no evidence that he intended the letter itself to be a codicil. Interesting choice by the court to allow this to occur guess they realized the girlfriend had nothing if they didnt uphold this letter. (ii) This is a very expansive reading of the most expansive curing document we have. a. There is a danger of judges deciding what the testators intent is they are deciding what he actually wanted. iv) 1990 UPC Sec. 2-503. Harmless Error Rule

70

Although a document or writing added upon a document was executed in compliance with Sec. 2-502, the document or writing is treated as if it had been executed in compliance with that section if the proponent of the document or writing establishes by clear and convincing evidence that the decedent intended the document or writing to constitute (i) the decedents will, (ii) a partial or complete revocation of the will, (iii) an addition to or an alteration of the will, or (iv) a partial or complete revival of his [or her] formerly revoked will or of a formerly revoked portion of the will. b) Correcting Mistakes in the Content of Wills - The Use of Extrinsic Evidence. i) Mistaken or Ambiguous Language in Wills (1) The traditional Approach: No Extrinsic Evidence [plain meaning rule], no Reformation (a) Plain Meaning Rule (i) Under this rule, extrinsic evidence may be admitted to resolve some ambiguities, but the plain meaning of the words of the will cannot be disturbed by evidence that another meaning was intended. 1. We dont care about what the testator meant, we care what the words would mean in the normal persons mind. Holmes, kinda. (b) No Reformation rule (i) Reformation is an equitable remedy that, if applied to a will, would correct a mistaken term in the will to reflect what the testator intended the will to say. 1. The issue here is that courts are compelled to interpret the words that the testator used, not interpret the words that the testator is purported to have intended to use. (c) Mahoney v. Grainger, 186 N.E. 86 (1933) Mass. (i) Material Facts: Testatrix asked her lawyer to draft a will that would allow her heirs to split it amongst themselves evenly. The lawyer drafted the will with heirs at law. At her death, she only had one heir at law, but many first cousins, which she orally told her lawyer she wanted to include in her will by the meaning of heirs. (ii) Issue: Does the testatrixs estate split evenly between all her heirs, or just her heir at law (her aunt). (iii) Holding: Just the aunt (iv) Reasoning: 1. Extrinsic evidence must be excluded. We can only read from the will itself. There is no ambiguity to the words heirs at law living at the time of my decease as used in the will. 2. The fact that it was not in conformity to the instructions given to the draftsman who prepared it or that he made a mistake does not authorize a court to reform or alter it or remold it by amendments. (v) Notes: 1. Heirs at law is meant by degree of separation, since this case works in Massachusetts

71

a. Thus, the aunt gets everything we dont know who are heirs at law are until the time of your death. 2. The judge here was a sexist. 3. The lawyer didnt ask enough questions: This is not easy to do, it takes some practice dont put things into complicated language unless you are really sure you know what that means. 4. In Gustagson v. Svenson, 366 N.E.2d 761 (Mass. 1977), the court did not allow reformation of a will that stated he wanted to give his estate to his heirs per stirpes. Under Mass. law at the time, his widow was his heir and took from the estate. 5. Estate of Smith, 738 N.E.2d 739 (Ill. App. 1990), the court held that a testators language directed his estate go to Nevada corporation, instead of the nursing home he wanted it to go to, based on extrinsic evidence, since the Nevada Corporations name was the exact matched name to the will. 6. The plain meaning rule has been criticized as fundamentally misdirected. The fallacy consists in assuming that there is or ever can be someone real or absolute meaning. In truth there can by only some persons meaning: and that person, whose meaning the law is seeking, is the writer of the document. The plain meaning is just the meaning of people who did not write the document. ii) The use of extrinsic evidence to address the validity of wills. (1) Fleming v. Morrison, 72 N.E. 499 (Mass. 1904) (i) Massachusetts court held a will invalid where the testator told one of the witnesses the will was a shame for the purpose of sleeping with a woman. The evidence shows that there was no real intent to create the will. If there is no intent to create a true will, then the statute is not satisfied. One of the witnesses is tainted, and therefore there are only two witnesses and the will does not stand. (ii) Notes: 1. This is about whether the will itself is valid, not the content of the will iii) Patent and latent ambiguity. (1) In some cases extrinsic evidence may be admitted if the words of the will are ambiguous or lack a plain meaning. (a) Patent (on its face) Ambiguities: (i) A patent ambiguity appears on the face of the will. 1. Under traditional law, there is no extrinsic evidence allowed the idea is that we dont want to add anything. a. Increasingly, however, courts have allowed extrinsic evidence to clarify patent ambiguities. (b) Latent Ambiguity:

72

(i) A latent ambiguity manifests itself only when the terms of the will are applied to the testators property or designated beneficiaries. Generally there are two types of latent ambiguity: 1. The first type occurs when a will clearly describes a person or thing, and two or more persons or things exactly fit that description. a. This is called equivocation admission of extrinsic evidence first occurred here since courts reasoned that the extrinsic evidence merely made the terms of the will more specific without actually adding to the wills terms. 2. Personal Usage Exception i. If the extrinsic evidence shows that the testator always referred to a person in an idiosyncratic manner, the evidence shows that the testator always referred to a person in an idiosyncratic manner, the evidence is admissible to show that the testator meant someone other than the person with the legal name of the legatee. 3. The third exists when no person or thing exactly fits the description, but two or more persons or things partially fit. - Misdiscription (ii) Oral declarations by the scrivener are admitted in most jurisdictions in cases of latent ambiguity. (c) Collapsing the Patent/Latent Distinction (i) Determining which ambiguity is at hand is often a subjective determination. 1. However, while courts have been more amenable to the admission of extrinsic evidence, the distinction has begun to fade. See Restatement (Third) Sec. 112., cmt. d (2003) once either has been found, extrinsic evidence can be used to find the donors intention. (d) Note: (i) We often allow extrinsic evidence in circumstances of the execution this is always allowed. (ii) Rules of construction: 1. Like statutes, courts have used general rules of construction to help interpret ambiguities or inconsistencies a. Stricter rule trumps, first named gets to choose, tenants in common, etc. b. We dont look at what the testator wanted, just what the rules say we should construe these inconsistencies as. iv) An alternative to the plain meaning rule - Focus on what the words meant to the testator rather than on what the words mean to the reader. (1) Estate of Russell, 444 P.2d 353 (1968) (a) Material Facts: Testatrix gave estate to a person and a dog. Plaintiff sought to bring extrinsic evidence to prove that the provision was only for the purpose of providing the person with funds to take care of the dog, not give the person funds themselves. (b) Issue: 73

(c) Holding: The testatrix left her property to the dog and her friend, in equal shares. Thus, the friend gets half and the other half, since no gift to a dog is valid, lapses under statute to testatrixs heirs. (d) Reasoning: (i) If an uncertainty arises upon the face of a will and extrinsic evidence is relevant to prove any of such meanings is admissible however, if the extrinsic evidence is used to show an intention different from that expressed by the words of the will, and there is no uncertainty arising upon the face of the will, such extrinsic evidence is inadmissible. (ii) No words give the entire estate to her friend or indicate the provision for the dog is precatory in nature. Such a disposition is inconsistent with a disposition which by its language leaves the residuum in equal shares to Quinn and the Dog. (iii) Interpreting the provisions relating to testatrixs residuary estate in accordance with the only meaning to which they are reasonably susceptible, we conclude that testatrix intended to make a disposition of all of the residue of the estate to Quinn and the dog in equal shares; therefore as tenants in common. 1. Since a dog cannot get a gift, the gift is void any lapsed gift, undisposed of by the will, passes to the heirs-at-law. (e) Notes: (i) The no-residue-of-a residue rule, used here to give the lapsed gift to the heirs at law, is a minority position. The majority of states use the assumption that the testatrix would rather the whole estate go to the beneficiaries named if one or more of the beneficiaries cannot take their share. Restatement (Third) of Property; Wills and Other Donative Transfers Sec. 5.5, cmt. o (1999). (ii) Before we look for ambiguity, we look at personal usage first 1. This is a dramatic departure from the traditional approach of only caring what those words mean to most folks (or to most lawyers and judges). 2. We look at her and her situation lets find out who the testatrix was. a. If we know something about her, using empathy, we can get an understanding about what she wanted however, at this point there is no extrinsic evidence of her testamentary intent we still dont want to overturn wills with extrinsic evidence. b. While it isnt easy to empathize with another person, we can at least try. (iii) If we cant figure out what she wanted (and we are not playing the patent and latent game) by using personal usage, and circumstances to determine what the language means, then we go out and get extrinsic evidence to figure it out. 1. Here, the court tries to preserves the sanctatity of the wills while taking an original approach to these circumstances. v) An alternative to the plain meaning rule - Correcting a scriveners mistake. (1) Slouching Toward Reformation: Correcting Mistakes Without the Power to Reform Wills (a) The traditional refusal to reform wills rests on the premise that mischief will ensue if the courts are allowed to reject the seemingly clear words of the will 74

(i) However, this is at odds with the courts actions towards the doctrines of undue influence, testamentary capacity (including insane delusions, duress and fraud. 1. Under each of these doctrines the courts look at evidence of surrounding circumstances to determine a proper remedy, which includes changing the terms of the will or voiding the will outright. Effect: Lack of Volition Undue Influence, Duress (Relief Granted) Lack of Capacity, Insane Delusion (Relief Granted) Effect: Mistaken Terms Fraud (Relief Granted) Mistake (No Relief)

Cause: Intentional Wrongdoing Cause: Innocent Acts

(b) A careful review of the following cases, however, shows an unmistakable trend toward admitting extrinsic evidence to correct mistaken terms to conform the will to the actual intent of the testator. (c) Arnheiter v. Arnheiter, 125 A.2d 914 (1956) N.J. (i) Material Facts: Testatrixs will had a written mistake in the address of her property that she wanted to sell. (ii) Issue: Does the executrix have power to sell the proper home, given that it was not properly written down? (iii) Holding: Yes. (iv) Reasoning: 1. While the courts cannot correct obvious mistakes, we can employ the doctrine of falsa demonstration non nocet (mere erroneous description does not vitiate). a. Where a description of a thing or person consists of several particulars and all of them do not fit any one person or thing, less essential particulars may be rejected provided the remainder of the description clearly fits. 2. Thus, we strike the address number, which is clearly erroneous, leaving the remaining description in the will which obviously points to the only property testatrix was in fact in ownership of. (v) Notes: 1. Is there really a difference between this doctrine and reformation? Dont both give effect to what the testator meant to say? (d) Estate of Gibbs, 111 N.W.2d 413 (Wis. 1961) (i) Material Facts: George and Lena Adele left 1% to an individual. However, that individual was not their long-time friend, but just some guy at the address given with a similar name. (ii) Issue: Where there is a clear mistake of identity, should the will be reformed to reflect this clear mistake, or should the contents of the will go as the will speaks? 75

(iii) Holding and Reasoning: 1. Where such details of identification are involved, courts should receive evidence tending to show that a mistake has been made and should disregard the details when the proof establishes to the highest degree of certainty that a mistake was, in fact, made. 2. Thus, we disregard the middle initial and address, and give the property to the person the testators had in mind. (iv) Notes: (2) Openly Reforming Wills for Mistake (a) Erickson v. Erickson, 716 A.2d 92 (1998) (i) Material Facts: Testator executed a will giving all of his assets to his then girlfriend two days before his marriage to the same woman. He later died and his daughters challenged the will on the grounds that a marriage automatically revokes a will that does not provide for the contingency of a future marriage. The result being that the estate would go to intestacy and she would get a share. The defendant offered extrinsic evidence that the testator intended the will to stand regardless of his marriage this was based on his conversations with his scrivener (his attorney) and the fact that he married the woman he had given his entire estate to, two days later (ii) Issue: Should the court allow extrinsic evidence to correct a scriveners error? (iii) Holding: Yes. (iv) Reasoning: 1. There is no discernible policy difference between allowing evidence for fraud, duress, and undue influence and a case in which a will is executed on erroneous beliefs induces by the innocent error, or innocent misrepresentation, of the scrivener of the will. a. In both cases, the testamentary process was distorted by the interference of a third person who misled the testator into executing a will that would not otherwise have been executed. 2. We do not want to blindly enforce a testamentary disposition that substantially misstates the testators intent. a. The statute of wills does not compel enforcement of testamentary dispositions that a testator never intended to make. 3. The two main objections are not sound: a. Signing a will is a strong presumption that the testator intended the contents, but this is a rebuttable presumption i. Here the attorney assured the testator that the will would not be affected by his subsequent marriage. b. There is no worry of a slippery slope of law suits where the law is narrowly tailored to allow the opponent of a will to introduce extrinsic evidence of the error of a scrivener and require proof of such error to be established by clear and convincing evidence. 76

4. Applying these principles, the extrinsic evidence, if offered, could prove clearly and convincingly that the scrivener mistakenly induced the testator to execute a will that he intended to be valid despite his subsequent marriage. (v) Notes: 1. This case directly attacks the issue of reformation for mistakes supported by clear and convincing evidence. 2. This is a very narrow holding limited only to scrivener situations (does not apply to holographic will). Must be third person scrivener under this case law. (b) 2008 UPC Sec. 2-805. Reformation to Correct Mistakes (i) The court may reform the terms of a governing instrument, even if unambiguous, to conform the terms to the transferors intention if it is proved by clear and convincing evidence that the transferors intent and the terms of the governing instrument were affected by a mistake of fact or law, whether in expression or inducement. (c) John H. Langbein, Curing Execution Errors and Mistaken Terms in Wills, 18 Prob. & Prop. 28 (Jan./Feb 2004) (i) The Trend Away from Formalism 1. Both by judicial decisions and by legislation, the courts have been empowered to excuse harmless execution errors and to reform mistaken terms, counter to the intent-defeating traditional approach of voiding a will outright. (ii) Mistaken Terms 1. Restatement (Third) Sec. 12.1 provides that a court may reform any donative document, including a will, to conform the text to the donors intention if it is established by clear and convincing evidence (1) that a mistake of fact or law, whether in expression or inducement, affected specific terms of the document; and (2) what the donors intention was. a. The restatement also allows reformation in other ways, not covered here. See 12.2 where the restatement allows reformation for a tax benefit the testator would have wanted had they known about it. (iii) Why the change? 1. The rise of the nonprobate system; 2. Far more wealth now passes through nonprobate transfers than probate a. The harmless error and reformation rules, long used for nonprobate transfers, are now being applied to probate transfers as a way to unify the two systems. 3. Experience in other jurisdictions 4. Growing embarrassment that failure to cure well-proved mistakes inflicts unjust enrichment; and

77

a. Not allowing a will under these issues not only frustrates intent, but also raises unjust enrichment. Those who take from the will are unjustly enriched at the expense of the intended beneficiary. i. Preventing unjust enrichment is the central policy of restitution law. This policy has been an important stimulus in the changing laws for reformation in wills. 5. Concern to spare lawyers from needless malpractice liability. a. While there are many lines of argument against this reasoning (i.e. many of these cases arise from laypersons drafting wills and many cases where attorneys are judgment proof for lack of insurance or not enough insurance). (iv) New Vistas for the Probate Lawyer 1. Lawyers processing probate matters need to be alert to the opportunity they now have to raise issues that used to be foreclosed. Sad cases of defeated intent that used to be beyond hope are now remediable, an innocuous formal defect can be excused, mistaken terms can be reformed, but only if counsel sees the issue and brings it forward. (d) Notes: (i) Reformation in the cases; correcting mistakes by the scrivener: 1. In Estate of Lord, 795 A.2d 700 (Me. 2002), the residuary clause was mistakenly drafted to refer to a trust and a trustee that never came into existence, but the court interpreted these words to mean estate and personal representative instead. 2. In Estate of Getman, 15 Quinnipiac Prob. L.J. 257 (Conn. Prob 2000), the court corrected language in a will that identified intervivos trust as created date to created May 9, 1999. 3. Where there has been an accidental omission by the scrivener or typist, courts have sometimes inserted the missing words when convinced from the face of the will and extrinsic evidence what was intended. See Estate of Herceg, 747 N.Y.S.2d 901 (Sur. 2002) page 354. 4. There is also a growing trend in the cases, endorsed by UPC Sec. 2-806 (2008), toward allowing reformation of mistake (as well as outright modification) of wills and trusts as necessary to obtain a tax advantage. (ii) Malpractice: 1. Should a lawyer be held accountable for drafting an ambiguous will? a. In Ventura County Humane Socy. V. Holloway, 115 Cal. Rptr. 464 (App. 1976), the court held that although an attorney is liable to testamentary beneficiaries if the beneficiaries clearly designated by the testator lose their legacy as a direct result of the attorneys negligence, the attorney is not liable for drafting an ambiguous document b. However, see Angela M. Vallarios, Shape up or Ship out: Accountability to Third Parties for Patent Ambiguities in Testamentary Documents, 26 78

Whittier L. Rev. 59 (2004) (arguing that drafting a will with patent ambiguity should be prima facie evidence of negligence). (iii) Nonprobate Transfers: 1. Drafting mistakes in nonprobate transfers, such as inter vivos trusts, can be corrected after the settlors death. See Bilafer v. Bilafer, 73 Cal. Rptr. 3d 880 (Cal. App. 2008) on page 355. (iv) Clear and Convincing Evidence: 1. Some argue that the testator should affirmatively authorize reformation in order for it to be used by courts. This is on the basis that courts now have too much power to change a will, and perhaps, the testators intent a. I dont buy it. The level of evidence needed is pretty substantial to make the change. (v) Gifts by Implication: 1. To fill the gaps of dispositive provisions particular contingencies (which occur) are not provided for New Jersey courts have developed the doctrine of probable intent. a. If a contingency occurs for which no provision is made in the will, the court studies the family circumstances and the plan of testamentary disposition set forth in the will b. The court then places itself in the position of the testator and decides how the testator probably would have responded to the contingency had she envisioned its occurrence. (vi) Not everyone is enamored with the trend toward reforming wills and the admission of extrinsic evidence. Massachusetts, for example, rejected reformation for other purposes than obtaining tax advantages. The basis is the avoidance of what they believe would be a soaring amount of groundless will contests. 1. However, Massachusetts does not resist extrinsic evidence to vary the plain meaning of the will in all cases. See Fleming v. Morrison, supra. c) Lapse: Death of the Beneficiary Before Death of the Testator. i) Introduction: (1) If a devisee does not survive the testator, the devise lapses, (that is, it fails). All gifts made by the testator require that the devisee survive the testator, unless the testator states otherwise. (a) Nearly all states have created an antilapse statute where, under specified circumstances, courts will substitute another beneficiary for the predeceased devisee. (2) Default Rules of Common Law: (a) Specific or general devise (i) If a specific or general devise lapses, the devise falls into the residue (b) Residuary devise

79

(i) If the residuary devise lapses, the heirs of the testator take by intestacy. If only a share of the residue lapses, such as when one of two residuary devisees predeceases the testator, at comm law the lapsed residuary share passes by intestacy to the testators heirs rather than to the remaining residuary devisees. This rule is called the no-residue-of-the-residue rule. 1. In most states, this rule has been overturned due to overwhelming criticism. The majority relies on the assumption that the testator would rather have the other devisee to have the entire estate then to pass to unnamed heirs. (c) Class gift (i) If the devise is to a class of persons, and one member of the class predeceases the testator, the surviving members of the class divide the gift. 1. Much turns on whether a gift is to a group of persons is a class gift. (d) Void devise (i) Where a devisee is already dead at the time the will is executed, or the devisee is a dog or cat or some other ineligible taker, the devise is void. The same default rules govern disposition of a void devise as govern a lapsed devise. ii) Antilapse Statutes (1) A typical antilapse statute provides that if a devisee is of a specified relationship to the testator and is survived by descendants who survive the testator, the descendants are substituted for the predeceased devisee. This is done unless the testator provides otherwise. (2) Presumed Intent (a) The theory behind these statutes is presumed intent: for certain predeceasing devisees, the testator would prefer a substitute gift to the devisees descendants rather than for the gift to lapse. (3) Scope (a) These statutes apply only if the devisee bears the particular relationship the testator specified by statute. These vary widely (i) The 1969 UPC applies only to a devise to a grandparent or a lineal descendant of a grandparent while the 1990 UPC adds a devise to a stepchild. Other states go even further, allowing the statute to apply to all devisees, whether they are related or not. 1. Experience suggests that most antilapse statues are too narrowly drawn. There seems to be no evidence to support limiting the statute to close relatives even though most legislatures do any way. (4) Default Rules (a) When the will is not clear for contrary intent, courts struggle with whether the testator has a contrary intention (i) To my living brothers and sisters, to share and share alike was held to not apply the antilapse statute, while the words share and share alike were insufficient to not apply the antilapse statute. (5) Words of survivorship 80

(a) UPC Sec. 2-603 (1990, rev. 2008) introduces an additional complexity. Paragraph (b)(3) of that section provides that words of survivorship, such as in a devise to an individual if he survives me, or in a devise to my surviving children, are not, in the absence of additional evidence, a sufficient indication of an intent contrary to the application of this section. (i) The official comment argues that it is equally plausiblethat the words of survivorship are in the testators will merely because the testators lawyer used a will form with words of survivorship. The Restatement Sec. 5.5, cmt. h, amplifies on this reasoning. See page 366. (ii) Sec. (b)(3) has come under sharp criticism from commentators, and has only been fully accepted by two states, with five others adopting a modified version that expressly preserve the majority rule that survivorship words are a sufficient indication of an intent contrary to the application of antilapse statutes. (6) The importance of sound drafting. (a) A good lawyer does not rely upon presumptions. Instead she makes the clients intent clear by providing what happens if the intended devisee does not survive the testator. (7) Boilerplate revisited (a) Lawyers may add new terms such as no lapse or antilapse statute may apply or lawyers may continue to use old forms because they are not aware of the new statutes. The intention of these statutes is for attorneys to have conversations with their clients about what they should be aware of, instead of just filling out a form. iii) 1969 UPC Sec. 2-605. Antilapse; Deceased Devisee; Class Gifts (1) If a devisee who is a grandparent or a lineal descendant of a grandparent of the testator is dead at the time of execution of the will, fails to survive the testator, or is treated as if he predeceased the testator, the issue of the deceased devisee who survie the testator by 120 hours take in place of the deceased devisee and if they are all of the same degree of kinship to the devisee they take equally, but if of unequal degree then those of more remote degree take by representation. One who would have been a devisee under a class gift if he had survived the testator is treated as a devisee for purposes of this section whether his death occurred before or after the execution of the will. iv) 1990 UPC Sec. 2-603 d) Class Gifts. i) Class gifts are treated differently, under common law lapse rules, than gifts to individuals. (1) If a class member predeceases the testator, the surviving members of the class divide the total gift, including the deceased members share. (2) What is a class? (a) The test is whether the testator was group-minded this can be determined by language (to As children or to my nephews and nieces) (b) A gift to beneficiaries who form a natural class, but are described by individual name, may be determined a class, after admitting extrinsic evidence, that the

81

testator would want the survivors to divide the property rather than for a predeceasing beneficiarys share to lapse. ii) Rstmt. (Third) of Property: Wills and Other Do native Transfers (T.D. No. 4, 2004) (1) 13.1 Class Gift Defined How Created (a) A class gift is a disposition to beneficiaries who are described by a group label and are intended to take as a group. Taking as a group means that: (i) (1) the membership of the class is typically not static, but is subject to fluctuation by increase or decrease until the time when a class member is entitled to distribution; and (ii) (2) upon distribution, the property is divided among the then-entitled class members on a fractional basis (b) If the terms of the disposition identify the beneficiaries only by a group label, the disposition creates a class gift, unless the language or circumstances indicate that the transferor intended the beneficiaries to take as individuals. (2) 13.2 Class Gift Distinguished from Disposition to Beneficiaries Taking as Individuals How Created (a) (b) If the terms of the disposition identify the beneficiaries only by n ame, without any reference to a group label, the disposition does not create a class gift, but is to the beneficiaries taking as individuals. (b) (c)if the terms of the disposition identify the beneficiaries (i) By a group label and (ii) Either by name or by the number of beneficiaries who then fit the group label, the disposition is presumed not to create a class gift, but is to the beneficiaries taking as individuals. The presumption is rebutted if the language or circumstances indicate that the transferor intended the beneficiaries to take as a group. iii) Dawson v. Yucas, 239 N.E.2d 305 (1968) (1) Material Facts: Testatrix left a will giving her 1/5th ownership of farm land to two nephews, individually named, with the intention to return these lands to her late husbands side of the house. One of the nephews predeceased and the court needed to determine whether this was a class gift or not if it is, the proceeds would go to the surviving nephew, if not, it would go to the residuary estate. (2) Issue: Is the clause in this will a class gift? (3) Holding: No. (4) Reasoning: (a) The language of the will makes it clear that she intended it to go to two specific persons, these nephews. Taken with extrinsic evidence that other similarly situated family members, on her husbands side, were not named and that they did not have any contact with the testatrix, it is clear that this was an individual gift. (i) Thus, the remainder gift lapses to her residuary estate. (5) Notes:

82

(a) This case is in agreement with the weight of the modern authority The Jarman approach, cited by the court, takes a rules based approach to determining whether there was a class gift or not. (i) Other courts that disagree and take the approach that the class gift doctrine should be applied as necessary in situations to best meet the intentions of the testator. See Thomas M. Cooley, II, What Constitutes a Gift to a Class, 49 Harv. L. Rev. 903 (1936) iv) Application of Antilapse Statutes to Class Gifts (1) Almost all states apply their antilapse statutes to class gifts to a single-generation class such as to children or siblings, and most statutes so provide. See UPC Sec. 2-605 (1969) on page 365. (a) In states where the statute is unclear, the courts reason that the average testators intent and preference is that the deceased beneficiarys share should be passed to the beneficiarys descendants rather than to the surviving members of the class. (b) In some states antilapse statutes do not apply to dispositions to class members who die before execution of the will. See Tex. Prob. Code Ann. Sec.68(a) (2008). (i) In these states, the presumption is that the testator did not want the dead class member to take. e) Changes in Property After the Execution of a Will i) The distinction between specific and general devises. (1) Speaking generally, a specific devise is a disposition of a specific item of the testators property i.e., my home, my diamond ring, etc. (2) A devise is general when the testator intends to confer a general benefit and not give a particular asset for example, a legacy of 100 k to A. if there is not 100 cash in the testators estate at death, the legacy is not adeemed; other property must be sold from the estate to satisfy As general legacy. (3) A demonstrative devise is a hybrid; a general devise, yet payable from a specific source if that source does not exist or does not fully fulfill the amount, other property must be sold to meet the requirement; thus, it is not adeemed either. (4) A residuary devise conveys that portion of the testators estate no otherwise effectively devised by other parts of the will, such as a devise to A of all the rest, residue, and remainder of my property and estate. ii) Ademption. (1) Under the traditional identity theory of ademption, if a specifically devised item is not in the testators estate, the gist is extinguished (subject to limited expectations). (2) Under the intent theory, if the specifically devised item is not in the testators estate, the beneficiary may nonetheless be entitled to the replacement for, or cash value of, the original item, depending on whether the beneficiary can show that this is what the testator would have wanted. (3) In re Estate of Anton, 731 N.W.2d 19 (2007) (a) Material Facts: Attorney-in-fact, under a durable power of attorney, sold the remaining asset under the testators will, which was bequeathed to the attorney-in83

(b) (c) (d)

(e)

fact and another person, for the purpose of paying for health care for the testator who was no longer competent to handle her own finances. At death, the other beneficiary sought to claim a portion of the proceeds remaining of the sold property against the identity theory of ademption. Issue: Is the sale of certain property by an attorney-in-fact prior to the death of the testator resulted in ademption of a specific property bequest? Holding: The sale of the property by an attorney-in-fact under the circumstances presented did not result in ademption of the bequest. Reasoning: (i) The court uses a modified intention theory, which holds that the strict identity theory will not be applied to cases where specifically devised property is removed from the estate through an act that is involuntary as to the testator. 1. This includes cases where the property is sold by a guardian, or conservator, or is destroyed contemporaneously with the death of the testator. (ii) The rationale of our cases is that ademption occurs where a testator had knowledge of a transaction involving a specific devise, realizes the effect of the transaction on his or her estate plan, and has an opportunity to revise the will. 1. Where these elements are not present, no ademption occurs the focus of the analysis is on the testator and whether the testator has made a deliberate decision not to revise the will, and not on the nature of the agency causing the involuntary act. a. Note: The other case against this holding had a legislature who did not specifically exclude acts of agents pursuant to durable powers of attorneys, while this states legislature has done no such thing. (iii) Here, the testator had no knowledge of the actual sale of the property since discussing it would aggravate her condition. 1. If the testator was aware of the transaction, was aware of the impact the transaction had on her estate plan, and did not change her will, ademption would occur. Again, here the testator had no such knowledge and was vaguely aware that assets needed to be sold to continue her health care arrangements most ordinary persons would not run down to the lawyers office to change their will in light of a remote future contingency that has not been specifically discussed and which may or may not occur in the future. a. The question is whether the testator had the opportunity to change the will once she knew the property would be sold she simply did not have that opportunity. Notes: (i) A rule tempered by exceptions the identity theory 1. Courts and legislatures who follow this theory have developed exceptions to ademption where the property is not in the estate because of an accident or

84

the action of someone other than the testator, or where the facts indicate a high likelihood that the testator did not intend for ademption. a. Many jurisdictions give the devisee any unpaid amount of a condemnation award for the property or any unpaid fire or casualty insurance proceeds after the property is destroyed. See UPC Sec. 2608(a) (1969). b. Other courts will classify the devisee as general or demonstrative rather than specific to avoid ademption. c. Another way is to classify the inter vivos disposition as a change in form, not substance. (ii) A standard tempered by presumptions and burdens the intent theory 1. The 1990 UPC abandons the identity theory and adopts the intent theory. See UPC Sec. 2-606(a)(1)-(4) and (b). It codifies the exceptions and also provides additional exceptions for replacement property, (a)(5), and for the pecuniary value of property disposed of during the testators lifetime if the devisee can show that the testator did not intend ademption, (a)(6). (iii) In 1997, Sec. 2-606(a)(6) was amended to put the burden on the party opposing ademption in order to prevent frivolous litigation and to change the meaning of any specific section of a will to and its equivalent value. (4) Stock Splits and the Problem of Increase (a) Modern courts have held that, contrary showing of intent, a devisee of stock is entitled to additional shares received by the testator as a result of a stock split under the change in form, not substance, rationale (b) Some courts treat dividends differently UPC Sec. 2-605 (1990) treats dividends the same as stock splits, noting that the percentage in ownership remains the same even after a stock dividend (just like a stock split). (5) 1990 UPC Sec. 2-606: Nonademption of Specific Devises; Unpaid Proceeds of Sale, Condemnation, or Insurance; Sale by Conservator or Agent (a) A specific devisee has a right to the specifically devised property in the testator's estate at death and: (1) any balance of the purchase price, together with any security agreement, owing from a purchaser to the testator at death by reason of sale of the property; (2) any amount of a condemnation award for the taking of the property unpaid at death; (3) any proceeds unpaid at death on fire or casualty insurance or on other recovery for injury to the property; (4) property owned by the testator at death and acquired as a result of foreclosure, or obtained in lieu of foreclosure, of the security interest for a specifically devised obligation; (5) real or tangible personal property owned by the testator at death which the testator acquired as a replacement for specifically devised real or tangible personal property; and

85

(6) if not covered by paragraphs (1) through (5), a pecuniary devise equal to the value as of its date of disposition of other specifically devised property disposed of during the testator's lifetime but only to the extent it is established that ademption would be inconsistent with the testator's manifested plan of distribution or that at the time the will was made, the date of disposition or otherwise, the testator did not intend that the devise adeem.

(b) If specifically devised property is sold or mortgaged by a conservator or by an agent acting within the authority' of a durable power of attorney for an incapacitated principal, 28 or if a condemnation award, insurance proceeds, or recovery for injury to the property' are paid to a conservator or to an agent acting within the authority of a durable power of attorney for an incapacitated principal, the specific devisee has the right to a general pecuniary devise equal to the net sale price, the amount of the unpaid loan, the condemnation award, the insurance proceeds, or the recovery. (c) The right of a specific devisee under subsection (b) is reduced by any right the devisee has under subsection (a). iii) Satisfaction. (1) Satisfaction of General pecuniary bequests (a) The doctrine of Satisfaction (ademption by satisfaction) applies when the testator makes a transfer to a devisee after executing the will. (i) If the testator is a parent of the beneficiary (or stands in loco parentis) and sometime after executing the will transfers to the beneficiary property of a similar nature to that devised in the will, there is a rebuttable presumption that the gift is in satisfaction of the devise made by the will. 1. this doctrine (akin to the doctrine of advancements under intestacy law) usually applies to general pecuniary bequests but not to specific bequests. a. When it is specific, the gift is treated as adeemed by extinction, not by satisfaction. (2) This intent is difficult to ascertain, and thus some statutes have required the satisfaction to be show in writing. See UPC Sec. 2-609 (1990) which has this requirement, paralleling its rule on advancements (Sec. 2-109). iv) Exoneration of liens. (1) When a will makes a specific devise of land, on which there is a mortgage, the question may arise whether the devised land passes freed of the mortgage. (a) Under the exoneration of liens doctorine (applied in some states) when a will makes a specific disposition of real or personal property that is subject to a mortgage to secure a note on which the testator is personally liable, it is presumed, absent contrary language in the will, that the testator wanted the debt, like other debts, to be paid out of the residuary estate. (i) Disatisfaction with this result has led many states to reversing this common law rule, as does the UPC Sec. 2-607 (1990). v) Abatement. (1) The problem of abatement arises when the estate has insufficient assets to pay debts as well as al the devises; in such a case, some devises must be abated or reduced. 86

(a) Abatement operates like bankruptcy in that it divvies up a limited pie among claimants of different priorities. (i) Absent of any indication how this should be done, it usually abates in the following order 1. Residuary devises are reduced first 2. General devises are reduced second 3. Specific and demonstrative devises are the last to abate and are reduced pro rata. a. While this follows the intent that specific devises be given effect before more general ones, it is common that the residuary devisee is the most important. i. UPC Sec. 3-902 (1990) provides that if the testamentary plan distributes abate as many be necessary to give effect to the intention of the testator. ii. Of course, it is better to avoid these problems through better drafting. - It is often wise to make substantial devises in the form of shares of the residue. 7) CREATION OF TRUSTS a) Introduction. i) A trust is, functionally speaking, an arrangement whereby a trustee manages property as a fiduciary for one or more beneficiaries. (1) The trustee holds legal title to the trust property and, in the usual trust, can sell the property and replace it with property the trustee thinks is more desirable. (2) The beneficiaries hold equitable title and, in the usual trust, are entitled to payments from the trust income and sometimes from the trust corpus as well. ii) Trust Purposes (1) The diverse purposes for which the trust may be used range from a simple estate plan to provide for a surviving spouse and children in accordance with their respective needs, to commercial enterprises such as mutual funds, pensions, and various structured finance transactions. (a) The crucial point is that a trust provides managerial intermediation. (i) Because the trustee manages the property on behalf of the beneficiary, the trust separate*s+ the benefits of ownership from the burdens of ownership. (2) Private trusts uses of gratuitous wealth transfer in estate planning (a) Revocable trusts (b) Testamentary marital trusts (c) Trusts for incompetent persons (d) Trusts for minors (e) Discretionary trusts iii) Sources of law (1) Restatement 87

(2) Uniform Trust Code (UTC) iv) The parties to a Trust (Note: One party can wear two hats, and sometimes, all three) (1) The Settlor (a) The person who creates a trust (also called the trustor or grantor) (i) The trust may be created in life (inter vivos trust) or be created by will (testamentary trust). 1. An inter vivos trust may be created either by a declaration of trust in which the settlor declares that he holds certain property in trust, or by a deed of trust in which the settlor transfers property to another person as trustee. a. Under a declaration of trust, the settlor is the trustee the donor need only manifest an intention to hold the property in trust (unless it is real property, where the statute of frauds requires a writing). (b) In order to have a valid trust, the trustee must owe equitable duties to someone other than herself. (i) If the X were the sole trustee and also the sole beneficiaries, the equitable and legal title would merge, leaving X with absolute legal title (eliminating the trust). UTC Sec. 402(a)(5) (2005); RST. (Third) of Trusts Sec. 69 (2003). 1. Merger rarely occurs since most trusts have different beneficiaries at some point in the life of the trust (c) If the settlor is not the trustee, a deed of trsut or actual delivery of the trust property to the trustee is necessary. (d) If the trust is created by will, the trustee will necessarily be someone other than the settlor. (2) The Trustee (a) A trust may have one trustee or several trustees. A trustee may be an individual or a corporation. A trustee may be a third party, the settlor, or a beneficiary. (i) If the settlor intends to create a trust but fails to name a trustee, the court will appoint a trustee. RST. (Third) of Trusts Sec. 31, Sec. 34 (2003). 1. No matter what, a trust will not fail for want of a trustee the court will appoint one. a. There is a rarely invoked exception where the settlor intends the trust to end with the termination of the trustee (b) The trustee must have some active duties to perform if they do not, then the trust is dry or passive, and the trust fails, and the beneficiaries acquire legal title to the trust property (this is also a rare occurrence. (c) The central feature of the trust is bifurcation: The trustee holds legal title to the trust property, but the beneficiaries have the equitable, or beneficial, interests (i) Two categories of issues arise from this splitting of legal and equitable ownership of property 1. Third-Party Rights a. The trust is not a freestanding legal entity the trustee sues, is sued, and transacts in his capacity as such. 88

Note: A personal creditor, however, of the trustee has no recourse against the trust property. RST (Third) of Trusts Sec. 308 (1959). ii. A creditor of the trustee, as his capacity as trustee a person who transacts with the trustee in regard to the trust property has recourse against the trust property, but not against the trustees personal property. UTC Sec. 1010 (2010). 2. Rights of the beneficiary: The Fiduciary Obligation a. It is the beneficiaries who bear the burden of the trustees good or bad decisions to safeguard against mismanagement or misappropriation by the trustee, the trustee is held to a fiduciary standard of conduct i. This duty includes duties of loyalty, prudence, and a host of subsidiary rules that reinforce the duties of loyalty and prudence - Unless the trust provides otherwise, the duty of loyalty requires the trustee to administer solely in the interest of the beneficiary no self dealing. - Duty or prudence requires that the trustee be held to an objective standard of care in managing the trust property - Subsidiary rules: Duty of impartiality between classes of beneficiaries; duty not to commingle requires the trust property not be mingle with trustees own property; duty to inform and account to the beneficiaries. b. A trustee who breaches her fiduciary duties may be denied compensation, subjected to personal liability or removed as trustee i. Because of the high duty, a person is not intrusted with the office unless that person accepts. ii. UTC (2010) allows a trustee to reseign if they give 30 days notice to all interested parties common law required consent from the beneficiaries or by court order 3. Note: Individual versus Corporate Trustees a. An individual trustee is usually a close friend or relative, who does the job out of moral obligation or friendship they are relatively inexpensive but issues of succession may become an issue if/when they can no longer hold the position b. A corporate trustee on the other hand is usually a bank or trust company that is experienced in portfolio management and trust administration. i. Banks have deep pockets and they also provide additional safeguards (federal and state banking regulations) that allow beneficiaries to recover damages in the case of breach. ii. The cost is higher fees and they often are perceived as being unresponsive to the beneficiaries and highly inflexible.

i.

89

iii. UTC Sec. 708 and RST. (Third) of Trusts Sec. 38 (2003), entitles the trustee to reasonable compensation. The older rule, followed in some states, is to award the trustee an annual commission set by a statutory formula (usually a percentage of the trust corpus, or percentage of trust income, or some combination of both). Note: These rules are defaults that could be displaced by contrary agreement, and corporate trustees typically insist upon agreement to their published or other negotiated fee schedule before agreeing to serve. c. Professor Langbein: The Roles of the trusteeship; i. Investment - Selection of securities or other assests, monitoring the investments for continuing suitability, investing new funds, and voting the shares. ii. Administration - accounting, reporting, tax filing, taking custody of securities, maintaining and leasing real estate, etc. iii. Distribution - Interpreting and applying the sometimes complex language of the trust instrument; and it commonly involves contact with the current beneficiaries, in order to keep abreast of their needs and circumstances. d. Given the different roles a trustee takes part in, depending on the particular trust, settlors should consider naming co-trustees (an individual and institution) with the idea that they will have complimentary strengths and weaknesses. The modern law also encourages delegation by the trustee(s) to experts on matters outside of the trustees competence. i. Another fragmenting tool is to name a trust protector who is given specific powers, such as to order distributions, replace the trustee, or modify the trust in light of changed circumstances. ii. In a directed trust, the trustee, perhaps serving at a reduced commission, is responsible for administration but otherwise must follow the directions of a third party, such as a named advisor or distribution committee. e. Private trust company i. A company formed specifically to serve as trustee of on or more trusts created by a single family they consolidate the various functions of trusteeship. This is desirable if the family wants to retain control over a family business or has other nonfinancial objectives. (3) The Beneficiaries 90

(a) The beneficiaries hold equitable interests. These are generally different than legal interests (i) The beneficiaries have claims against the trustee for breach of trust, but this is not given any greater weight than the claims of other creditors of the trustee. (ii) Equity gives the beneficiaries additional remedies relating to the trust property itself 1. i.e., if the trustee disposes of the property, the beneficiaries have a right to the property (unless it lands in the hands of a bona fide purchaser for value) or if the property is replaced with proceeds or other property, they can enforce the trust on the newly acquired property. (iii) Since private trusts almost always create successive beneficial interests, the creation of a trust usually involves the creation of one or more equitable future interests as well as a present interest in the income. 1. Today, most life estates and future interests are equitable rather than legal interests; they are created in trusts. a. Legal life estates and future interests in tangible or intangible property are rare and unwise. The same is true for land. i. A trust with equitable interests is a much flexible and useful means of giving successive interests in property than are legal interests. (4) A Trust Compared with a Legal Life Estate (a) A legal life tenant has possession and control of the property, whereas a trustee, not the life beneficiary, has legal title to the trust property. (i) Legal life estates 1. The legal life tenant has no power to sell a fee simple unless such power is granted in the instrument creating the life estate. Otherwise, all remainderpersons and reversioners must agree to the sale or get court approval. a. No banker will lend to someone with only a life estate as security. b. Another problem is waste: taking any action on the property could constitute waste, entitling the remainderpersons to an injunction or damages. c. Meanwhile, the life tenant must pay taxes and maintain the property to the extent income from the property is enough to do so. d. Personal property also need management and the application of law in this area is uncertain, leaving remainderpersons without definitive recourse. e. If the life tenant goes into debt, the creditor may seize and sell the property if the debtor is a remainderperson, the creditor may be able to take the remainder and sell it both cases could result in a very low sale. f. Many other problems arise: trespassers, eminent domain, third party injuries, etc. 91

(ii) Equitable Life Estates (i.e., a trust) 1. All the forgoing difficulties are resolved or mitigated by using a trust. a. The trustee usually has broad enough powers to deal with all of these issues third parties only need deal with the trustee as well. b. Also, the trust property can be put out of the reach of creditors c. Thus, a trust is always preferable to a legal life estate. b) Intent to Create a Trust. i) No words are necessary to create a trust. The sole question is whether the grantor manifested an intention to create a trust relationship. UTC Sec. 402(a)(2) (2000). (1) Thus, a person unfamiliar with trust law can intend to create a trust; the focus is function rather than form. (2) A transfer of property to an individual to hold for the use and benefit of another person manifests an intention to create a trust. ii) Lux v. Lux, 288 A.2d 701 (R.I. 1972) (1) Anysaid residue shall be maintained for the benefit of said grandchildren and shall not be sold until the youngest of said grandchildren has reached twenty-one years of age. (a) Based on this language, the court deemed this to be a trust. (i) There is no fixed formula as to when a testamentary disposition should be classified as an outright gift or a trust. The result reached depends on the circumstances of each particular case. (ii) *A+ trust is created when legal title to property is held by one person for the benefit of anotherno particular words are required to create a trust A trust never fails for lack of a trustee. (iii) It is clear from the nature of this bequest and the age of the beneficiaries *that she] intended that someone would hold and manage the property until they were of sufficient age to do so themselves. 1. The terms shall be maintained and shall not be sold until is a strong indication of the testatrixs intent that the property be retained and managed for some considerable time in the future for the benefit of her sons children. (iv) We name the executor the position of trustee, as is the general rule where it is not improper and no trustee has been named. iii) Jimenez v. Lee, 547 P.2d 126 (1976) (1) Material Facts: Plaintiff sued her father to compel accounting for assets which she claimed were held by her father, as trustee for her. Her grandmother had given her $1,500.00. It is undisputed that this gift was made for the benefit of the plaintiffs educational needs and it was deposited in a savings account in the name of the defendant. Defendant used the funds to buy certain stock and named himself custodian of those funds. (2) Issue: Is this gift a trust, making defendant trustee and subject to fiduciary duties, or are the newly acquired stocks held by defendant as custodian? 92

(3) Holding: The gift was intended to be a trust and thus defendant must make his accounting known to plaintiff. (4) Reasoning: (a) It is enough to create a trust if the transfer of property is made with the intent to vest the beneficial ownership in a third person. The gift was for the educational needs for the defendant and nothing about the gift suggested that defendant could use it as he wished to only pay out of his own funds similar amounts for the plaintiffs educational needs. (b) Defendants attempt to broaden his powers over the trust estate by investing the trust funds as custodian violated his duty to the beneficiary to administer the trust solely in the interest of the beneficiary. (c) Defendant also has a duty as trustee to identify expenditures made on the beneficiaries behalf specifically, and prove they were made for the trust purpose, before he can be credited for any expenditure. (i) A trustees duty to maintain and render accurate accounts is a strict one. (ii) In determining whether the defendant has met the strict burden of proof for accounting, the trial court must adhere to the rule that all doubts are resolved against a trustee who maintains an inadequate accounting system. (5) Notes: (a) Defendant in this case had identified the gift as a trust in a letter to his mother he was also an attorney and judge. To use this language in the non-legal sense did not make sense to the court. (b) The stakes in disputes over a transferors intent to create a trust usually boil down to the applicability of the fiduciary obligation. (c) Precatory Language (i) When the testator expresses a wish, hope, or recommendation that property be used by the devisee in some particular manner it can be unclear whether there is an intention to make a trust or merely an unenforceable moral obligation. 1. The later of this is called precatory language, or a precatory trust meaning unenforceable dispositions of this sort. (ii) The lesson: Be clear in your drafting I wish, but do not legally require, that C permit D to live on the land. (d) Equitable Charge (i) If a testator devises property to a person, subject to the payment of a certain sum of money to a third person, the testator creates an equitable charge, not a trust 1. This creates a security interest in the transferred property there is no fiduciary relationship 2. The relationship between holder of the charge and the beneficiary is more in the nature of a debtor and secured creditor. iv) The Hebrew University Association v. Nye, 169 A.2d 641 (1961) Supreme Court of CT 93

(1) Material Facts: Owner of a rare collection of books (library) sought to give the library to a charity. Before delivery of the books, she died. The charity sought to define the gift as a trust, the owner as trustee, so that a lack of delivery would be immaterial. (2) Issue: Is this a trust or an inter vivos gift? (3) Holding: This is not a trust (4) Reasoning: (a) In order for a trust to exist there needs to be a manifestation of such intent. Moreover, while he need not use the term trustee, nor even manifest an understanding of its technical meaning or the technical meaning of the word trust, he must manifest an intention to impose upon himself enforceable duties of a trust nature (i) There is no evidence that the owner had this intention. (b) A gift which is imperfect for lack of a delivery will not be turned into a declaration of trust for no better reason than that it is imperfect for lack of a delivery. Courts do not supply conveyances where there are none. This is true, even though the intended donee is a charity. (5) Notes: (a) Case remanded for a new trial at which the plaintiff could present its case on other theories other than a declaration of trust. v) The Hebrew University Association v. Nye, 223 A.2d 397 (1966) Superior Court of CT. (1) For an inter vivos gift based on constructive or symbolic delivery the donor must do that which, under the circumstances, will in reason be equivalent to an actual delivery. (a) It must be as nearly perfect and complete as the nature of the property and the circumstances will permit. (b) The gift may be perfected when the donor places in the hands of the donee the means of obtaining possession of the contemplated gift, accompanied with acts and declarations clearly showing an intention to give and to divert himself of all dominion over the property. (c) It is not necessary that the method adopted be the only possible one. It is sufficient if manual delivery is impractical or inconvenient. (i) i.e., delivery of keys to a safe deposit box, point out a hiding place where money is hidden, informal memorandum, etc. (2) The delivery of the memorandum coupled with the decedents acts and declarations, which clearly show an intention to give and to divest herself of any ownership of the library, was sufficient to complete the gift. (a) Forms and formalities do serve a useful and sometimes an essential purpose, but an excessive regard for formalism should not be allowed to defeat the ends of justice. (3) For a court of equity to permit the decedents wishes to be doubly frustrated for no better reason than that the rules so provide makes no sense whatsoever. The plastic remedies of the chancery are molded to the needs of justice. (4) Notes:

94

(a) The Scott treatise, favoring doctrinal purity, disapproves of cases where the intention to make a gift seems plain and the gift fails for lack of delivery however, they would recast the gift as a declaration of trust in order to save it, as this court did not. (b) Rst. (Third) of Property: Wills and Other Donative Transfers Sec. 6.2, cmt. Yy (2003) provides that a gift of personal property can be perfected on the basis of doantive intent alone if the donors intent to make a gift is established by clear and convincing evidence. Thus, noncompliance with the formality of delivery can be excused as harmless error, similar to excusing noncompliance with will execution formalities. c) The Requirement of a Trust Res. (Necessity of Trust Property) i) Under traditional law, a trust cannot exist without trust property, often called the res. (1) The sole exception to the property requirement is under legislation such as the Uniform Testamentary Additions to Trusts act for an inter vivos trust that is to be funded by a pour-over will. ii) The trust res can be any amount or any interest in property that can be transferred. RST. (Third) of Trusts Sec. 40, cmt. B. (2003). iii) The critical question is whether a court will call the particular claim property. iv) Uthank v. Rippstein, 386 S.W.2d 134 (1964) (1) Material Facts: Decedent, before his death penned a lengthy letter to a woman stating that he would send payments the first of every week of each month for the next five years. He said nothing of his eventual death. In the letter he stated that he hereby bind my estate to make the $200.00 monthly payments provided for on this Page One of this letter. The woman filed suit against the executor of his will to make the payments arguing that the letter was a voluntary trust in which the decedent bound his estate to the extent of the promised payments; before and after his death. (2) Issue: Is this a trust? (3) Holding: No. (4) Reasoning: (a) It is manifest that the decedent did not expressly declare that all of his property, or any specific portion of the assets which he owned at such time, would constitute the corpus or res of a trust for the benefit of the woman; and inferences may not be drawn from the language used sufficient for a holding to such effect ot rest in implication. (b) The intention was to make monthly gifts to her, accompanied by an ineffectual attempt to bind his estate in the future the writing was no more than a promise to make similar gifts in the future and as such is unenforceable. (i) The promise to give cannot be tortured into a trust declarations under which decedent, during life as trustee, and his estate after his death, were under a legally enforceable obligation to pay her. (5) Notes: (a) What policies are served by refusing to give effect to the writing here? 95

(b) Trusts distinguished from debts (i) The requirement of an identifiable trust res distinguishes a trust from a debt. (ii) A debt involves a personal obligation to pay a sum of money to another. The crucial factor in distinguishing between a trust relationship and an ordinary debt is whether the recipient of the assets is entitled to use them as his own and commingle them with his own assets. (c) Resulting trusts (i) In Unthank, the woman argued that the decedent, after transferring all his property into trust, had a resulting trust in the amount of his property not required to meet the payment to her. (ii) A resulting trust is an equitable reversionary interest that arises in two situations: 1. Where an express trust fails or makes an incomplete disposition (RST. (Third) of Trusts Sec. 8), or 2. Where one person pays the purchase price for property and causes title to the property to be taken in the name of another person who is not a natural object of the bounty of the purchaser (RST. (Third) of Trusts Sec. 9). This is often called a purchase money resulting trust. (iii) In both situations, the transferee is not entitled to the beneficial interests, so the interest is said to result (that is, it reverts) to the transferor or to the transferors estate or other successor(s) in interest. RST Sec. 7. (iv) Once a resulting trust is found, the trustee must convey the property to the beneficial owner upon demand. v) Brainard v. Commissioner, 91 F.2d 880 (7th Cir. 1937) (1) Brainard orally stated that he decalred a trust on his expected profits from stock trading in 1928 for the benefit of his family. He agreed to assume personally any losses. He made a profit in 1928, gave himself his fee and distributed the remaining profits on his books for the benefit of his beneficiaries. (2) The court held that the trust did not arise until after the profits were credited on Brainards books on the grounds that there was no res at the time of the declaration of trust. (a) When there is no res at the time of a declaration of trust, the settlor must manifest anew his intent to create a trust when the res comes into being. During the intervening time, Brainard is the sole owner of the profits and should be taxed as such. vi) Speelman v. Pascal, 178 N.E.2d 723 (1961) (1) Material Facts: Gabriel Pascal wrote, signed, and delivered a document to the plaintiff, Miss Kingman, promising to pay her a certain percentage of profits from the stage production and movie based on the Pygmalion Musical stage. The stage production did not appear until after Pascals death.

96

(2) Issue: Whether a document purporting to make a gift of a percentage of profits from a stage production and movie, which were not in existence at the time the document, was made, constitutes a valid gift. (3) Holding: The gift to the plaintiff is valid because the letter shows intent to make an irrevocable present transfer. The donor did not need to make any further steps to make the gift effective. (4) Reasoning: (a) The mere fact that a gift refers to the future does not make the gift uncertain. Therefore the court only requires that the donor make a present intent to transfer even though the property itself may not yet exist. (b) It is true that at the time of the delivery of the letter there was no musical stage or film play in existence but Pascal could grant to another a share of the moneys to accrue from the use of those rights by others. (c) In every such case the question must be as to whether there was a completed delivery of a kind appropriate to the subject property. In this case, there was nothing left to do in order to make an irrevocable transfer to plaintiff of part of Pascals right to receive royalties from the productions. (5) Notes: (a) The subject of a valid gift does not have to be in present physical existence and in the possession of the donor but a document purporting to make such a transfer must express intent to make an irrevocable present transfer. (b) Whats the difference between the two cases? (i) An expectation or hope of receiving property in the future, or an interest that has not come into existence or has ceased to exist, does not qualify as a res sufficient to create a trust. RST Sec. 41. 1. However, a person may assign future earnings from an existing contract. 2. The theory is that the future yield of an existing property right can be transferred even though property to be acquired in the future cannot be. a. In Speelman, Pascal had exclusive rights (a license) from the Shaw estate to make a musical version of Pygmalion. d) The Necessity of a Beneficiary to whom the Trustee Owes Fiduciary Duties. i) A trust must have one or more ascertainable beneficiaries. UTC Sec. 402(a)(3) (2000). The reason: There must be someone to whom the trustee owes fiduciary duties, someone who can call the trustee to account. ii) Exceptions: (1) A charitable trust (2) The beneficiaries of a private trust may be unborn or unascertained when the trust is created. (a) Note: If at the time the trust becomes effective the beneficiaries are to indefinite, the trust fails for want of ascertainable beneficiaries. 8) WILL SUBSTITUTES a) Introduction. 97

i)

These modes of nonprobate transfer, taken together, function as a private system of succession that runs in parallel indeed, competes with the probate system. ii) John H. Langbein, The Nonprobate Revolution and the Future of the Law of Succession, 97 Harv. L. Rev. 1108 (1984) (1) Probate, our court operated system for transferring wealth at death, is declining in importance. Will substitutes are functioning as free-market competitors of the probate system and enabling property to pass on death without probate and without will. (2) We pretend that these will substitutes are lifetime transfers. In truth, will substitutes are simply nonprobate wills wills that need not comply with the Wills Act (3) The Will Substitutes (a) When properly created, each is functionally indistinguishable from a will each reserves to the owner complete lifetime dominion, including the power to name and to change beneficiaries until death. The four pure will substitutes: (i) Life Insurance 1. The lable aside, life insurance is functionally indistinguishable from a will, for it satisfies the twin elements of the definition of a will. A will is revocable until the death of the testator and that interests of the devisees are ambulatory that is, nonexistent until the testators death. Unless specifically restricted by contract, the life insurance beneficiary designation operates the same way. 2. Just as the will is ambulatory, taking effect only on death, so the beneficiary designation can be changed until death. (ii) Pension Accounts (iii) Bank, Brokerage, and Mutual Fund Accounts (iv) The Revocable Inter Vivos Trust (v) Imperfect Will substitutes 1. Most prominent is the common-law joint tenancy. Because they ordinarily effect lifetime transfers, joint tenancies are imperfect rather than pure will substitutes. (b) The Hidden causes of the Nonprobate Revolution (i) Most will substitutes but not all are asset-specific (ii) Property that passes through a will substitute avoids probate (iii) The formal requirements of the Wills Act do not govern will substitutes are not complied with 1. Of these differences, only probate avoidance is a significant advantage that transferors might consciously seek. (4) Notes: (a) What lessons for probate reform can be drawn from the success of will substitutes? (b) Given the broad proliferation of will substitutes, how can the law facilitate a unified estate plan? (i) The combined use of revocable trust and pour-over will has become the technique of choice. 98

b) Will Substitutes and the Wills Act. i) Validity of Revocable Inter Vivos Trusts: Revocable Trusts (1) The revocable trust is the most flexible of all will substitutes because the donor can draft both the dispositive and the administrative provisions precisely to the donors liking. (a) An inter vivos trust can be revocable or irrevocable (testamentary trust, created by will, are by definition irrevocable) (2) A revocable trust may be created by a deed of trust, whereby the settlor transfers the property to be held in trust to the trustee. At the settlors death, the trust property is then distributed or held in further trust, depending on the terms of the trust instrument. (a) All jurisdictions now allow a revocable trust created by deed of trust to effect a nonprobate transfer on death. (3) A revocable trust may also be created by a declaration of trust where the settlor simply declares himself to be trustee of certain property for the benefit of himself during his life, with the remainder to pass to others at his death. (a) Since the settlor in this case retains the power to revoke the trust and the right ot the rust income, and as trustee he also controls the management of the trust property, the question arises if the declaration of trust be effective to transfer property at the settlors death without Wills Act formalities? (4) Farkas v. Williams, 125 N.E.2d 600 (1955) (a) Material Facts: Farkas bought several stocks and created revocable inter vivos instruments (which he called trusts) for the benefit of himself while alive and Williams, a friend, when Farkas died. Farkas was trustee-Settlor and beneficiary during life. At death, the stocks would be given to Williams. Farkas died and his family appealed saying that these were not valid trusts and that his stocks should go through intestate succession. (b) Issue: Are the trust instruments created valid inter vivos trusts effective to give the purported beneficiary title to the stock in question after the death of the settlortrustee? (c) Holding: These are revocable inter vivos trusts (d) Reasoning: (i) To make the determination we must consider: 1. Whether upon execution of the so-called trust instruments defendant acquired an interest in the subject matter of the trusts, the stock of defendant 2. Whether the decedent, as settlor-trustee, retained such control over the subject matter of the trusts as to render said trust instruments attempted testamentary dispositions. (ii) Here, when he executed these instruments, he could not deal with the stock in the same was as if he owned the property absolutely, but only in accordance with the terms of the instrument he intended to take on those trustee obligations implied by law. In addition he intended to bind himself to having this 99

property pass upon his death to Williams, unless he revoked it a true owner does not have to jump through these hoops thus, Williams had this present interest (however, it is difficult to name what that interest is exactly). 1. While the instrument did revoke if Williams predeceased, it does not mean he does not have an interest a. This is just another contingency which would serve to terminate the trust b. The RST of the Laws of Trust refer to the interest of a beneficiary under a trust who must survive the settlor as a contingent equitable interest in remainder. (iii) The fact that Farkas was settlor-trustee gave him great power. However, if we look to the revocable trust, the ability to revoke by itself is considerable power over the trustee regarding administration of the trust. Any trustee power can easily be assumed by the settlor with his power to revoke. He still had his duties as trustee. It is not a valid objection to this that Williams would never question Farkas since he would just revoke: that possibility exists whether he is trustee or not. 1. There are certain situations where Williams could indeed have a claim against Farkas contrasted with the rights of legatee or devisee under a will (they have no action to how the testator spends his ownership rights). (iv) Finally, the problems in which the formalities seek to fix are not at issue here he signed four different stock documents in which he made Williams the beneficiary. This was enough of a solemn and formal manner for the issue here. (e) Notes: (i) 2000 UTC Sec. 603: Settlors Powers of Withdrawal 1. (a) While a trust is revocable [and the settlor has capacity to revoke the trust], rights of the beneficiaries are subject to the control of, and the duties of the trustee are owed exclusively to, the settlor. 2. (b) During the period the power may be exercised, the holder of a power of withdrawal has the rights of a settlor of a revocable trust under this section to the extent of the property subject to the power. ii) Payable On Death Contracts. (1) In re Estate of Atkinson, 175 N.E.2d 548 (1961) (a) Material Facts: Testator left certificates of deposit at a bank which were payable at death to his daughters. In his will he specifically left out his wife/widow that she should not share in his estate. The widow took the forced share (spousal share) and upon learning about the certificates claimed they should have been part of testators estate for her percentage. (b) Issue: Does adding the words payable on death to a certificate of deposit is not invalid as being contrary to the statue of wills? (c) Holding: These instruments are an ineffectual attempt at a testamentary disposition of the deposits involved. 100

(d) Reasoning: (i) There was no present interest of any kind created in the decedents daughters by the language used in the certificate of deposit. The words pay on death are clearly testamentary. (e) Notes: (2) Estate of Hillowitz, 238 N.E.2d 723 (1968) (a) Material Facts: Testator had been a partner as an investment club which would transfer the shares owned by any partner at death to his wife. The executor claimed that this was an invalid testamentary disposition and should be included in the testators estate. The widow claimed that this was no different than any other valid and enforceable contract. (b) Issue: Is this a contract or an invalid testamentary disposition? (c) Holding: This is a valid contract (d) Reasoning: (i) A partnership agreement which provides that upon death the partners shares shall pass onto the remaining partners is valid. So too, is a contract that specifies these shares go to the wife. (ii) These partnership undertakings are nothing more or less than third-party beneficiary contracts, performable at death. Like many other contracts performable at death, they need not conform to the Statue of Wills. (e) Notes: (i) Whats the difference between these two cases? iii) John H. Langbein, The Nonprobate Revolution and the Future of the Law of Succession, 97 Harv. L. Rev. 1108 (1984) (1) Will substitutes fall on the will side of the will/gift line. Nevertheless the case law treats them as lifetime transfers (at least the ones that have legitimized them). (a) The main point is to identify some sort of present interest in the transferee, acquired during the lifetime of the transferor, which makes the transferee a donee and distinguishes the will substitute from a will. (i) This is a legal fiction courts make the right results for the wrong reasons, insisting that we treat will-like transfers as gift-like incidents. (ii) Of the pure will-substitutes only the transparently labeled P.O.D. account has persistently failed the present-interest test and has had to depend for the most part upon statutory validation. iv) The modern law no longer denies the will-like nature of the will substitutes and has validated them expressly. UPC validates authorized POD designations in all contracts, all but a few states have followed suit. Securities are also allowed to be transferred on death (TOD). These too are now allowed in nearly all states. v) 1990 UPC Sec. 6-101. Nonprobate Transfers on Death A provision for a nonprobate transfer on death in an insurance policy, contract of employment, bond, mortgage, promissory note, certificated or uncertificated security, account agreement, custodial 101

agreement, deposit agreement, compensation plan, pension plan, individual retirement plan, employee benefit plan, trust, conveyance, deed of gift, marital property agreement, or other written instrument of a similar nature is nontestamentary. This subsection includes a written provision that: (1) money or other benefits due to, controlled by, or owned by a decedent before death must be paid after the decedent's death to a person whom the decedent designates either in the instrument or in a separate writing, including a will, executed either before or at the same time as the instrument, or later; (2) money due or to become due under the instrument ceases to be payable in the event of death of the promisee or the promisor before payment or demand; or (3) any property controlled by or owned by the decedent before death which is the subject of the instrument passes to a person the decedent designates either in the instrument or in a separate writing, including a will, executed either before or at the same time as the instrument, or later. c) Pour-Over Wills. i) Introduction (1) The revocable trust allows the settlor to consolidate under one instrument the dispositive plan for all her property, probate and nonprobate. To achieve this goal, the settlor designates the trustee of her revocable trust as the beneficiary of all her will substitutes and names the trustee as the beneficiary under he will this is call a pourover will. (a) To change her estate plan later, all the settlor need to do is amend her revocable trust. ii) Norman Dacey and Avoiding Probate (1) This man wrote a book saying to do what Farkas did in the above referenced case. He also was a swindler and tax avoider his book did make many states rethink their probate system; many adopted the UPC, particularly its informal probate and small estate procedures. (a) Even so, revocable trusts are still advantageous in that they are relatively less costly, less time-consuming, and not public. (b) Note: The rise of lay industry to meet consumer demand for living trusts to avoid probate contains an obvious lesson: Reform probate and the law of wills so that a will can have the attributes of a revocable trust. If this were done, the living trust business would collapse. But until it is done, suppressing nonlawyers who are giving the public what its wants, while some lawyers are not, is unlikely to succeed. iii) Consequences During Life of Settlor (1) Property management by fiduciary (a) A third party trustee may be selected to manage a funded revocable trust, with duties under modern law that run only to the settlor. This is desirable if the settlor wants to be relieved of that duty. On the other hand, issues may arise when property is put in trust since third parties, such as banks, may want to see a copy of the trust instrument to verify the trustees power. (2) Keeping title clear

102

(a) A revocable trust is useful in keeping separate property that a husband or wife or both do not want commingled with their other assets (avoid issues at death or divorce). (b) Spouses who move from a community property state to a separate property state sometimes create a revocable trust for their community property to preserve their ability to qualify for a stepped-up income tax basis on all the property when one spouse dies. (3) Income and gift taxes (a) A well drafted will can save just as much money as a well drafted revocable trust (4) Dealing with incompetency (a) A revocable trust can be used to plan for this occurrence. The settlor may be a cotrustee, with the trust instrument providing that either co-trustee alone may at with respect to the trust property. Or the trust instrument can say that the co-trustee will shall act as sole trustee if the settlor becomes incompetent. The trust instrument should also include a mechanism for determining if the settlors incompetence. (i) Such provisions reduce the risk of litigation and allow the settlor in advance to select the successor fiduciary. (ii) A revocable trust continues during the settlors incapacity and can provide for disposition or continued management of the trust property at the settlors death 1. This is an advantage over a custodianship and durable power of attorney. iv) Consequences at Death of Settlor: Avoidance of Probate (1) Costs (a) Fees, pour-over documents, cost of drafting, and costs of transferring property (2) Delays (a) Under a revocable trust, income and principal can be disbursed to the beneficiaries more quickly. (b) Also, since the rules governing trustees are more liberal, it makes it easier for trustees to deal with an ongoing business in the form of a partnership or sole proprietorship, and for a trustee to exercise options, borrow money, and participate in reorganizations. (3) Creditors (a) In many states there is no similar short term limitation for creditors on revocable trusts as there is on wills. This is one place where probate may hold an advantage over a revocable trust (4) Publicity (a) Revocable trusts are private wills are not. (5) Ancillary probate (a) Transferring property to a revocable inter vivos trust can avoid ancillary probate at the owners death. (6) Avoiding restrictions protecting family members (a) It is one way to avoid the elective share or write out a child born out of wedlock 103

(7) Avoiding restrictions on testamentary trusts (a) Testamentary trusts often involve court order and court supervision even after the estate has closed. Also, in most states a testamentary trust has a duty to account to the court. These are often time-consuming and expensive procedures. (8) Choosing the law of another jurisdiction to govern (a) As a general matter, the settlor may choose what state law governs the revocable trust (if the trust asset is land, the law of the state where the land is located governs). The settlor may name his state, any beneficiaries state or where the trust is administered. (b) A testamentary trust cannot do this this is normally where the settlor is domiciled (c) Note: UPC Sec. 2-703 (1990) changes the old law and provides that the testator may select the state law to govern the meaning and the legal effect of his will, including trusts created by the will, unless that law is contrary to the domiciliary states law protecting the surviving spouse or any other public policy of the domiciliary state. (9) Lack of certainty in the law (a) The law may be less certain in discerning an issue that arises than it would be in the case of a will. The subsidiary law of wills dealing with divorce, adoption, lapse, ademption, simultaneous death, apportionment of death taxes, and creditors rights may or may not apply to revocable trusts, depending on the rule and the jurisdiction. (i) Of course, many of these issues can be dealt with in drafting, if you can foresee the issues. (10) Avoiding will contests (a) It is harder to set aside a funded revocable trust on incompetence and also in general if the trust has been ongoing with statements and income for several months. (11) Estate taxation (a) No real advantage here (12) Controlling surviving spouses disposition (13) Perceived complexity (a) Some clients have trouble thinking that someone else is in control of their property, even if they can revoke it at anytime this can be a deal breaker. v) Pour-Over Wills (1) O sets up a revocable inter vivos trust naming X as trustee. O then executes a will devising the residue of his estate to X, as trustee, to hold under the terms of the trust. The pour-over by will of probate assets into an inter vivos trust allows O to establish an inter vivos trust that can serve as a single receptacle for all the settlors probate and nonprobate property. i.e., Pilafas and Reiser (See below) (2) Two theories on how this law was developed (a) Incorporation by reference (i) Requires that the trust instrument be in existence at the time the will is executed 104

(b) Independent significance (i) Requires that the inter vivos trust have some property transferred to it during life, which the trust disposes of (3) Neither permits a pour-over into an unfunded inter vivos trust, whether drafted before or after the execution of the will. (a) Because of the limitations and uncertainties of these doctrines and frequent embarrassing errors by lawyers, estate planners sought the enactment of legislation permitting a will to pour over probate assets into an unfunded inter vivos trust. (4) UTATA, in its original 1960 form, validated a pour-over of probate assets into an inter vivos trust only if the trust instrument was executed (signed) before or concurrently with the will. (5) The original Uniform Act did not require that some property be transferred to the inter vivos trust during life. If the trust instrument was executed before or concurrently with the will, the probate assets could be poured over into the inter vivos trust as subsequently amended. The trust funded at death by the pour-over was treated as an inter vivos trust that is, as having come into existence before the testators death. The gave pour-over trusts the advantages of inter vivos trusts. (a) The 1990 UPC deleted the original acts requirement that the trust instrument be executed before or concurrently with the will. 1990 UPC Sec. 2-511(b), and the conforming 1991 UTATA, permits the trust instrument to be executed or amended after the will. 1990 UPC Sec. 2-511: Testamentary Additions to Trusts

(a) A will may validly devise property to the trustee of a trust established or to be established (i) during the testator's lifetime by the testator, by the testator and some other person, or by some other person, including a funded or unfunded life insurance trust, although the settlor has reserved any or all rights of ownership of the insurance contracts, or (ii) at the testator's death by the testator's devise to the trustee, if the trust is identified in the testator's will and its terms are set forth in a written instrument, other than a will, executed before, concurrently with, or after the execution of the testator's will or in another individual's will if that other individual has predeceased the testator, regardless of the existence, size, or character of the corpus of the trust. The devise is not invalid because the trust is amendable or revocable, or because the trust was amended after the execution of the will or the testator's death. (b) Unless the testator's will provides otherwise, property devised to a trust described in subsection (a) is not held under a testamentary trust of the testator, but it becomes a part of the trust to which it is devised, and must be administered and disposed of in accordance with the provisions of the governing instrument setting forth the terms of the trust, including any amendments thereto made before or after the testator's death.
d) Wills Substitutes and the Subsidiary Law of Wills. i) Impact on beneficiaries and on a unified estate plan. (1) Introduction (a) Restatement (Third) of Property: Wills and Other Donative Transfers (2003) 105

(b) Sec. 7.2 Application of Will Doctrines to Will Substitutes (i) Although a will substitute need not be executed in compliance wit the statutory formalities required for a will, such an arrangement is, to the extent appropriate, subject to substantive restrictions on testation and to rules of construction and other rules applicable to testamentary dispositions (ii) Comment 1. a. Rationale. [A] will substitute serves the function of a will. It shifts the right to possession or enjoyment to the donee at the donors death. In this sense, a will substitute is in reality a nonprobate will See page 413. (2) Revocable Trusts (a) The revocable trust is the most will-like of all the will substitutes. Thus, it is not surprising that the courts have applied to revocable trusts subsidiary rules from the law of will such as abatement or ademption when there is not enough trust property to satisfy the provisions calling for distribution on the death of the settlor (abatement) or the rust does not include a specific item of property that is to be distributed to a particular beneficiary (ademption). (b) Ire Estate and Trust of Pilafas, 836 P.2d 420 (Ariz. App. 1992) (i) Material Facts: Testator executed a will and a revocable inter vivos trust but at his death no one could find either document. The issue arose whether a presumption of revocation apply to one or both instruments. (ii) Issue: Does revocation apply to both instruments? (iii) Holding: The will is revoked by the trust is not. (iv) Reasoning: 1. Unlike the execution of a will, the creation of a trust involves the present transfer of property interests in the trust corpus to the beneficiaries. These interests cannot be taken from the beneficiaries except in accordance with a provision of the trust instrument, or by their own acts, or by a decree of a court. 2. Even a revocable trust vests the trust beneficiary with a legal right to enforce the terms of the trust. The terms of the trust also limit the powers of the settlor and trustee over the trust corpus, even when the settlor declares himself trustee for the benefit of himself and others. 3. The trust declared that the settlor may revoke only by writing delivered to the trustee. Even though the settlor wore both hats here, he still needed to revoke according the terms of his trust. 4. Since the trust transferred present interests to the trust beneficiaries, this trust is not applicable to subsidiary rules of wills even if doing so would be practical. (v) Notes: ii) Impact on creditors. (1) State Street Bank and Trust Co. v. Reiser, 389 N.E.2d 768 (1979)

106

(a) Material Facts: Dunnebier had almost all of his assets in a revocable inter vivos trust, which he was settlor, trustee and beneficiary (as were others). He then applied for a loan which he got. He soon after died in an accident and the residue of his will was not enough to pay the debt. (b) Issue: Can the bank reach the assets of an inter vivos trust in order to pay a debt to the bank owed by the estate of the settlor of the trust? (c) Holding: Yes. (d) Reasoning: (i) When a person creates for his own benefit a trust for support or a discretionary trust, his creditors can reach the maximum amount which the trustee, under terms of the trust, could pay to him or apply for his benefit. 1. This is so even if the trust contains spendthrift provisions. (ii) Under the terms of Dunnebiers trust, all the income and principal were at his disposal while he lived. However, at death, the trust vested in other beneficiaries (iii) Nonetheless, it is against public policy that a settlor to have an estate to live on, but not an estate to pay his debts with. The settlor of a trust who retains administrative powers, power to revoke or power to control beneficial enjoyment owns that trust property and provides that it shall be included in the settlors personal estate. (iv) Thus, where a person places property in trust and reserves the right to amend and revoke, or to direct disposition of principal and income, the settlors creditors may, following the death of the settlor, reach in satisfaction of the settlors debts to them, to the extent not satisfied by the settlors estate, those assets owned by the trust over which h the settlor had such control at the time of his death as would have enabled the settlor to use the trust assets for his own benefit. (e) Notes: (i) This case is the prevailing view: UTC Sec. 505(a)(3)(2000) are in accord and RST. (Third) of Trusts Sec. 25, cmt. e (2003) (ii) Creditors of a joint tenant cannot reach the jointly held property after the joint tenants death because the decedents interest is treated as having vanished. Life insurance proceeds are usually exempt if payable to a spouse or child, and the same is generally true of retirement benefits. U.S. savings bonds and POD beneficiary may also be exempt. 1. UPC Sec. 6-102 (1998) permits the decedents creditors to reach nonprobate transfers (except joint tenancies in real estate), such as revocable inter vivos trusts and joint bank accounts, if the probate estate is insufficient to pay the debts. (iii) Lapse and antilapse statutes to interests in trusts have been the most controversial of all the subsidiary rules from the law of wills (not sure if we covered this). 107

iii) Standing of beneficiaries. (1) Linthicum v. Rudi, 148 P.3d 746 (2006) (a) Material Facts: Cobb executed a revocable inter vivos trust naming beneficiaries. Later he changed the beneficiaries to someone else who shortly thereafter attempted to have guardianship of Cobb. This was dropped, and a court ordered another as guardian. The original beneficiaries sought to contest the amended trust under undue influence/incapacity. A motion to dismiss was filed claiming they did not have standing for a suit. (b) Issue: Do revocable inter vivos trust beneficiaries have the right to challenge amendments to the trust, when made by the settlor during settlors lifetime? (c) Holding: Because a beneficiarys interest in the revocable inter vivos trust is contingent at most, we hold that, generally, these beneficiaries lack standing to challenge the settlors lifetime amendments. (d) Reasoning: (i) Since this is a contingent trust that has not vested, they are not interested persons. As long as the settlor has the right to revoke or modify and other indicia of retained ownership during her lifetime, the beneficiaries did not have an absolute entitlement to any portion of the trust while the settlor was alive. (ii) Thus, they lack standing while he is still alive. (iii) As to their worries of capacity, those issues should be addressed under guardianship laws of the state. (e) Notes: (i) How is this case and Farkas reconcilable? (ii) UTC Sec. 604 (2000) provides that a person may bring suit to challenge a revocable trust, but only after the trust become irrevocable by reason of the settlors death. e) Pour-Over Wills and Revocable Trusts in Modern Estate Planning. i) Clymer v. Mayo, 473 N.E.2d 1084 (Mass. 1985) (1) Material Facts: Mayo executed a will where the bulk of her estate was to pour-over into a revocable trust. She named the appellee as the beneficiary of the revocable trust. She designated the trustees as the beneficiaries of her Boston University group life insurance policy and her TIAA-CREF pension retirement plans. After Mayo and the appellee divorced, she changed the beneficiary of her Boston University group life insurance to Marianne LaFrance. However she left the trustees beneficiary of her pension plans and the appellee was the main beneficiary of the trust. Mayo consolidated all of her property into her revocable trust. The revocable trust was unfunded during Mayos lifetime. (2) Issue: (a) Whether unfunded trusts that are intended to be funded by a pour-over will are valid?

108

(b) Whether a statute that revokes a former spouses gift and powers under a will can also be applied to that former spouses interests under a trust if the will and trust are a part of one testamentary scheme? (3) Holding: (a) Yes. A trust that receives property from a will is valid even though the trust had been unfunded during the settlors lifetime if the residuary clause of the testators will identified the trust and the terms of the trust were set out in a written instrument executed contemporaneously with the will. (b) Yes. A will and pour-over trust are a part of one testamentary scheme where the trust is a revocable pour-over trust that is funded entirely at the time of the decedents death. Because the trust and will were made as one scheme, the trust was also revoked as to the former spouse under the will statute. Furthermore, the legislature did not intend under the statute to allow the former spouse to receive a gift or power through a trust. (4) Reasoning: (a) In this case, the decedent left virtually all of her property to her former spouse. Because the legislature determined that former spouses should not be able to receive under their former spouses will, the court determined that it should also apply to testamentary trusts. (i) They used the legislatures intent to reach this conclusion. (ii) Transferors use will substitutes to avoid probate, not to avoid the subsidiary laws of wills. The subsidiary rules are the product of centuries of legal experience in attempting to discern transferors wishes and suppress litigation. These rules should be treated as presumptively correct for will substitutes as well as for wills. (iii) The court restricted their holding to the particular facts of this case. (5) Notes: (a) See 1990 UPC Sec. 2-511 above. f) Other Will Substitutes. i) Life insurance. (1) Whole life (a) This is a combination life insurance and savings plan. This eventually gets paid up or endowed, after which no further premiums are owed. (2) Term (a) No savings feature. It is a contract whereby the insurance company is obligated to pay the named beneficiary if the insured dies with the policys stated term of years. (3) Depending on the policy, the owner or beneficiary (after the insureds death), may select different settlement options for the receipt of death benefits. For modest estates, these settlement options can provide some of the flexibility that trusts provide for more substantial estates. (4) Cook v. Equitable Life Assurance Society, 428 N.E.2d 110 (Ind. App. 1981)

109

(a) Material Facts: Cook purchased a life insurance policy and named the appellant as the beneficiary. Approximately two years later, Cook and the appellant divorced. The parties agreed that the provisions of the divorce agreement would be full satisfaction of all claims by either of the parties against each other including alimony, support, and maintenance money. After the divorce, Cook stopped making payments on the life insurance policy. He was thereafter notified that his policy had changed to a paid-up term policy, and contained a provision specifying the procedure for changing a beneficiary. Cook could change the beneficiary by sending a written notice to the Equitable Life Assurance Society. Instead, Cook executed a holographic will in which he bequeathed his policy to the appellant. The will was admitted to probate and Doris Cook filed a claim with the Society for the insurance funds. The trial court ruled in favor of Doris Cook. (b) Issue: Whether a testator may change the beneficiary of his life insurance policy through a will even though it does not comply with the prescribed method in the insurance policy. (c) Holding: No. A testator must comply with the rules of the insurance policy to effect a change of beneficiary. Strict compliance with insurance policy requirements is necessary to change a beneficiary under the policy. The insurer, the insured, and beneficiary should be able to rely on the certainty that policy provisions relating to the naming and changing of beneficiaries will control (d) Reasoning: (i) Courts will protect the expectation interest of a beneficiary under a policy. Because the testator remarried, his first wife would not have known that he had changed her as the beneficiary because he changed it in his will and not with the Society. (ii) It is in the interest of insurance companies to require and to follow certain specified procedures in the change of beneficiaries of its policies so that they may pay over benefits to persons properly entitled to them without subjection to claims by others of whose rights they had no notice or knowledge. (iii) The insured had not done all that was within his powers or all that reasonably could have been expected of him to comply with the policy provisions respecting a change of beneficiary, but that through no fault of his own he was unable to achieve his goal. (e) Notes: (i) In many states, the statute that revokes a will provision for a divorced spouse does not apply to the designation of the divorced spouse in a life insurance contract or other nonprobate transfer. 1. In other states, it does revoke it. UPC Sec. 2-804 had Cook been in one of these states, he would have been fine. a. Regardless, an attorney dealing in divorce proceedings should handle these issues immediately.

110

b. Even so, most, but not all, courts have upheld retroactive application of the statute (insurances created before the enactment of the statute). c. Also, the application of statues such as UPC Sec. 2-804 to federally regulated pension accounts and employer-provided life insurance has been curtailed by Egelhoff v. Egelhoff (Below). (ii) A superwill? 1. These could revoke nonprobate transfers, if those nonprobate instruments said that a the will could do so however, this would essentially mean that those nonprobate transfers would be brought into probate, eliminating some of their advantages a. RST (Third) of Property Sec. 7.2 cmt. e (2003) endorses a modified superwill concept: see page 423. ii) Pension accounts and Retirement Accounts (1) Congress has long held that death beneficiaries could be permitted on various types of retirement plans. (2) John H. Langbein, The Twentieth-Century Revolution in Family Wealth Transmission, 86 Mich. L. Rev. 722 (1988) (a) There are three crucial advantages to conducting retirement savings through a taxqualified pension plan. (i) Most contributions are tax deferred (ii) The earnings on qualified plans investments accrue and compound on a taxdeferred basis. (iii) Distributions are taxed at a lower tax level since most people who begin to get these distributions are making less annual income than they did when they were working. (3) Defined Benefit Pension Plans (a) The employer promises to pay an annuity on retirement (i) This shifts the burden of living too long to a pension fund or insurance company. (ii) These remain common in government pension plans (4) Defined Contribution Pension Plans (a) The employer or employee or both make contributions to a specific pension account for the employee. Upon retirement the employee may withdraw from the account, subject to various distribution rules. (i) This operates in effect as a tax-advantaged savings account that concludes with a nonprobate transfer. (5) Does ERISA, the federal regulation of pension plans, preempt the state subsidiary law of wills? (a) Egelhoff v. Egelhoff, 532 U.S. 141 (2001) (i) Material Facts: The Petitioner was married to David A. Egelhoff. Egelhoff was employed by Boeing who gave him a life insurance policy and pension plan that were both governed by ERISA. The Petitioner was designated as a beneficiary under both plans. Egelhoff died in an automobile accident two months after the 111

couple divorced. Egelhoffs children of a previous marriage challenged her status as beneficiary because a state law revokes all nonprobate testamentary gifts to former spouses. (ii) Issue: Whether ERISA preempts a state statute that revokes the payment of a nonprobate asset to a former spouse? (iii) Holding: Yes. The federal Employee Retirement Income Security act preempts a state statute which revokes the payment of a non probate asset to a former spouse because the statute interferes with the statutes goal to administer a nationally uniform plan. The ERISA statute commands that a plan shall, specify the basis on which payments are made to and from the plan. If administrators are forced to act in accordance with the state statute, they will have to comply with the varying statutes of all 50 states and wait on litigation before processing a payment. This delay conflicts with the legislatures goal of minimizing the administrative and financial burdens placed on beneficiaries. (iv) Reasoning: 1. The state law directly interferes with the purpose of ERISA because it would cause beneficiaries to endure lengthy litigation before receiving their payments and the administrators would have to comply with several states with conflicting laws in order to distribute payments. (v) Notes: 1. Dissent: The state law imposes a mere administrative burden on the ERISA statute at the expense of other substantive state goals. This Court has held that the fact that state law poses some burden on the administration of ERISA plans does not necessarily require pre-emption. ERISAs ultimate goal is to protect employee benefits and the state law seeks to transfer an employees pension in the manner they wanted to receive them. In this case the Court permits a divorced wife to receive a windfall at the expense of the testators children. The logic of this Court would also extend to state cases involving slayer statues that prohibit a husband who kills a wife from receiving benefits as a result of a wrongful death. 2. iii) Multi-party bank and brokerage accounts. (1) Because banks and brokerage houses often give their customers a joint tenancy form without regard to the customers particular intention, courts are often left with the problem of discerning which type of account was actually intended. (2) Varela v. Bernachea, 917 So. 2d 295 (2005) (a) Material Facts: Man opens joint account with mistress. During his heart attack she basically empties the account. After he recovers he demands the money back and the mistress sues. (b) Issue: Does the mistress have a right to the money she withdrew? (c) Holding: Yes. (d) Reasoning: 112

(i) When a joint bank account is established with the funds of one person, a fit of the funds is presumed. The presumption can be rebutted only by clear and convincing evidence to the contrary. (ii) There is no evidence that he lacked donative intent and failed to rebut the presumption. (e) Notes: (i) If a convenience or POD account is intended, during the depositors life, the other tenant is not entitled to treat the funds as her own. If a convenience account is intended, the survivor is not entitled to the balance no the death of the depositor. For more see page 434 iv) Joint tenancies and tenancies by the entirety. (1) Joint tenancy and tenancy by the entirety are common and popular methods of avoiding the cost and delay of probate. (a) Upon the death of one joint tenant or tenant by the entirety, the survivor owns the property absolutely, freed of the decedents interest in the property. (i) The creation of a joint tenancy in land gives the joint tenants equal interests upon creation. (ii) A joint tenant cannot devise her share by will (iii) A creditor of a joint tenant generally must seize the joint tenants interest, if at all, during the joint tenants life. 1. In almost all states, the joint tenants interest vanishes at death, leaving nothing for the creditor to reach.

Formatted: Font: Italic

113

You might also like